SlideShare a Scribd company logo
1 of 129
Download to read offline
Dàch Vö To¡n Håc 
32 · thi tuyºn sinh v o lîp 10 
¤i håc KHTN H  Nëi 
(k±m theo ¡p ¡n) 
Mæn To¡n 
WWW.VNMATH.COM
About VnMath.Com 
vnMath.com 
Dàch vö To¡n håc 
info@vnmath.com 
¤i sè 
S¡ch 
Gi£i t½ch 
H¼nh håc 
C¡c lo¤i 
kh¡c 
Chuy¶n · 
To¡n 
Luy»n thi 
¤i håc 
Bçi d÷ïng 
HSG 
· thi 
¡p ¡n 
¤i håc 
Cao håc 
Gi¡o ¡n 
c¡c mæn 
Olympic 
Thi lîp 10 
1 
1T i li»u ÷ñc t¼m th§y tr¶n m¤ng v  khæng rã t¡c gi£.
Ch֓ng 1 
· thi tuyºn sinh lîp 10 
1.1 · thi tuyºn sinh lîp 10 nam 1989 
(cho måi thí sinh) 
Bài 1. Cho a thùc P(x) = ax2 + bx + c. 
Bi¸t r¬ng vîi måi giá trà nguyên cõa x, giá trà cõa a thùc P(x) ·u là 
nhúng sè chính ph÷ìng (nghia là b¬ng bình ph÷ìng cõa mët sè nguyên). 
Chùng minh r¬ng các h» sè a, b, c ·u là nhúng sè nguyên, và b là mët sè 
ch®n. 
Bài 2. Tìm giá trà bé nh§t cõa biºu thùc 
a2 + ab + b2 − 3a − 3b + 1989 
Giá trà bé nh§t ó ¤t ÷ñc t¤i giá trà nào cõa a và b? 
Bài 3. Chùng minh r¬ng trong 52 sè nguyên d÷ìng b§t ky luôn luôn có 
thº tìm ÷ñc 2 sè sao cho têng ho°c hi»u cõa 2 sè ó chia h¸t cho 100. 
Bài 4. Cho tam giác ABC. V· phía ngoài tam giác v³ các góc[BAx = 
[CAy = 21. H¤ BE vuông góc vîi Ax (E n¬m trên Ax), CF vuông góc vîi 
Ay (F n¬m trên Ay. M là trung iºm cõa BC. 
1. Chùng minh r¬ng tam giác MEF là tam giác cân 
2. Tính các góc cõa tam giác MEF. 
Bài 5. Có 9 håc sinh vøa lîp A vøa lîp B s­p thành mët hàng dåc, 
ùng cách ·u. Chùng minh r¬ng có ít nh§t 1 håc sinh ùng cách hai em 
cùng lîp vîi mình mët kho£ng cách nh÷ nhau. 
5 
www.vnmath.com
6 Ch÷ìng 1. · thi tuyºn sinh lîp 10 
1.2 · thi tuyºn sinh lîp 10 nam 1989 
(cho thí sinh thí sinh chuyên lý) 
Bài 1. Tìm t§t c£ nhúng giá trà nguyên cõa x º biºu thùc sau là sè nguyên 
−2x2 + x + 36 
2x + 3 
Bài 2. Tìm giá trà bé nh§t cõa biºu thùc 
a2 + ab + b2 − 3a − 3b + 3 
Giá trà bé nh§t ó ¤t ÷ñc t¤i giá trà nào cõa a và b? 
Bài 3. 
1. Chùng minh r¬ng vîi måi m nguyên d÷ìng, biºu thùc m2 + m + 1 
không ph£i là sè chính ph÷ìng (nghia là không thº b¬ng bình ph÷ìng 
cõa sè nguyên). 
2. Chùng minh r¬ng vîi måi m nguyên d÷ìng, m(m+1) không thº b¬ng 
tích cõa bèn sè nguyên liên ti¸p. 
Bài 4. Cho tam giác ABC vuông cân, góc A = 90. CM là trung tuy¸n 
(M n¬m trên AB). Tø A v³ ÷íng vuông góc vîi MC c­t BC ð H. Tính 
t sè BH 
HC . 
Bài 5. Có 6 thành phè, trong ó cù 3 thành phè b§t ky thì có ít nh§t 2 
thành phè liên l¤c ÷ñc vîi nhau. Chùng minh r¬ng trong 6 thành phè nói 
trên tçn t¤i 3 thành phè liên l¤c ÷ñc vîi nhau. 
1.3 · thi tuyºn sinh lîp 10 nam 1989 
(cho thí sinh chuyên toán - tin håc) 
Bài 1. Phân tích biºu thùc sau thành nhân tû 
a4 + b4 + c4 − 2a2b2 − ab2c2 − 2c2a2 
Bài 2. 
1. Cho bi¸t x 
x2+x+1 = −2 
3 . Hãy tính giá trà cõa biºu thùc 
x2 
x4 + x2 + 1 
www.vnmath.com
1.4. · thi tuyºn sinh lîp 10 nam 1991 (cho måi thí sinh) 7 
2. Tìm giá trà lîn nh§t cõa biºu thùc 
x2 
x4 + x2 + 1 
Giá trà lîn nh§t ó ¤t ÷ñc t¤i giá trà nào cõa x 
Bài 3. Cho biºu thùc P(n) = an + bn + c, trong ó a, b, c là nhúng 
sè nguyên d÷ìng. Chùng minh r¬ng n¸u vîi måi giá trà nguyên d÷ìng cõa 
n, P(n) luôn chia h¸t cho m (m là sè nguyên d÷ìng cè ành), thì b2 ph£i 
chia h¸t cho m. Vîi ví dö sau ây hãy chùng tä r¬ng không thº suy ra b 
chia h¸t cho m 
P(n) = 3n + 2n + 3 (xét khi m = 4) 
Bài 4. Cho a giác lçi sáu c¤nh ABCDEF.M, I,L,K,N,H l¦n l÷ñt là 
trung iºm cõa các c¤nh AB,BC,CD,DE,EF,FA. Chùng minh r¬ng các 
trång tâm cõa hai tam giác MNL và HIK trùng nhau. 
Bài 5. Gi£ sû trong mët tr÷íng có n lîp ta ký hi»u am là sè håc sinh 
cõa lîp thù m, dk là sè lîp trong ó méi lîp có ít nh§t k håc sinh, M là sè 
håc sinh cõa lîp ông nh§t. Chùng minh r¬ng: 
1. a1 + a2 + · · · + an = d1 + d2 + · · · + dM 
2. a21 
+a22 
+· · ·+a2 
n = d1+3d2+5d3+· · ·+(2k−1)dk +· · ·+(2M−1)dM 
1.4 · thi tuyºn sinh lîp 10 nam 1991 
(cho måi thí sinh) 
Bài 1. 
1. Gi£i và bi»n luªn ph÷ìng trình. 
p 
a + x + 
p 
a − x 
p 
a + x − 
p 
a − x 
= 
p 
b 
Trong ó a, b là các sè d÷ìng ã cho. 
2. Cho ph÷ìng trình x2 + ax + b+1 = 0. Trong ó a, b 2 Z và b6= −1. 
Chùng minh r¬ng n¸u ph÷ìng trình có hai nghi»m ·u là nhúng sè 
nguyên thì a2 + b2 là hñp sè. 
www.vnmath.com
8 Ch÷ìng 1. · thi tuyºn sinh lîp 10 
Bài 2. Cho a, b, c là các sè ôi mët khác nhau và khác 0. Gi£i h» 
8 
: 
a3x + a2y + az = 1 
b3x + b2y + bz = 1 
c3x + c2y + cz = 1 
Bài 3.Tìm nghi»m nguyên, d÷ìng cõa ph÷ìng trình 7x = 3.2y + 1. 
Bài 4. 
1. Cho hình thang ABCD(AB//CD). Gåi giao iºm cõa AD và BC là 
E, giao iºm cõa AC và BD là F. Chùng minh r¬ng ÷íng th¯ng EF 
i qua giao iºm cõa hai áy AB,CD. 
2. Cho tam giác ABC. M,N,P l¦n l÷ñt là các iºm trên các c¤nh 
BC,CA,AB. Nèi AM,BN,CP. Chùng minh r¬ng n¸u di»n tích cõa 
bèn tam giác g¤ch chéo b¬ng nhau thì các di»n tích cõa ba tù giác 
không g¤ch chéo cung b¬ng nhau. (Xem hình v³) 
Bài 5. Tçn t¤i hay không 1991 iºm trên m°t ph¯ng sao cho ba iºm 
b§t ky trong chúng là ba ¿nh cõa mët tam giác có mët góc tù? 
1.5 · thi tuyºn sinh lîp 10 nam 1991 
(cho thí sinh chuyên toán và chuyên tin) 
Bài 1. 
1. Rút gån biºu thùc 
A = 
q 
3 
p 
3 − 4 
2 
p 
2. 
q 
6 
p 
6 
44 + 16 
2. Phân tích biºu thùc sau thành nhân tû 
P = (x − y)5 + (y − z)5 + (z − x)5 
www.vnmath.com
1.6. · thi tuyºn sinh lîp 10 nam 1992 (cho måi thí sinh) 9 
Bài 2. 
1. Cho các sè a, b, c,
, 
 tho£ mãn các i·u ki»n 
8 
: 
a + b + c = 0 
 +
+ 
 = 0 
 
a +
b + 
 
c = 0 
Hãy tính giá trà cõa biºu thùc A = a2 +
b2 + 
c2 
2. Cho bèn sè a, b, c, d méi sè ·u không âm và nhä hìn ho°c b¬ng 1. 
Chùng minh r¬ng 
0  a + b + c + d − ab − bc − cd − da  2 
Khi nào thì d§u ¯ng thùc x£y ra? 
Bài 3. Cho tr÷îc a và d là nhúng sè nguyên d÷ìng. Xét t§t c£ các sè 
có d¤ng 
a, a + d, a + 2d, . . . , a + nd, . . . 
Chùng minh r¬ng trong các sè ó có ít nh§t mët sè mà 4 chú sè ¦u 
tiên cõa nó là 1991. 
Bài 4. Trong mët cuëc hëi th£o khoa håc có 100 ng÷íi tham dü. Gi£ 
sû méi ng÷íi ·u quen bi¸t vîi ít nh§t 67 ng÷íi. Chùng minh r¬ng có thº 
tìm ÷ñc mët nhóm 4 ng÷íi mà b§t ky 2 ng÷íi trong nhóm ó ·u quen 
bi¸t nhau. 
Bài 5. 
1. Cho hình vuông ABCD. L§y iºm M n¬m trong hình vuông sao cho 
MAB =MBA = 15. 
Chùng minh r¬ng tam giác MCD là tam giác ·u. 
2. Hãy xây düng mët tªp hñp gçm 8 iºm có tính ch§t: ÷íng trung 
trüc cõa o¤n nèi hai iºm b§t ky luôn i qua ít nh§t hai iºm cõa 
tªp hñp iºm ó. 
1.6 · thi tuyºn sinh lîp 10 nam 1992 
(cho måi thí sinh) 
Bài 1. 
www.vnmath.com
10 Ch÷ìng 1. · thi tuyºn sinh lîp 10 
1. Gi£i ph÷ìng trình 
q 
p 
2x − 5 + 
x+ 2 + 3 
q 
p 
2x − 5 = 2 
x − 2 − 3 
p 
2 
2. Gi£i h» ph÷ìng trình 
( 
xy2 − 2y + 3x2 = 0 
y2 + x2y + 2x = 0 
Bài 2. Tìm t§t c£ các c°p sè nguyên không âm (m, n) º ph÷ìng trình 
x2 − mnx + m + n = 0 
có nghi»m nguyên. 
Bài 3. Cho tam giác ABC có di»n tích S. Trên các c¤nh AB,BC,CA 
l¦n l÷ñt l§y C0,A0,B0 t÷ìng ùng, sao cho 
AC0 = C0B, 
BA0 
A0C 
= 
1 
2 
, 
CB0 
B0A 
= 
1 
3 
Gi£ sû AA0 c­t BB0 t¤i M, BB0 c­t CC0 t¤i N, CC0 c­t AA0 t¤i P. Tính 
di»n tích tam giác MNP theo S. 
Bài 4. Cho tam giác ABC nëi ti¸p trong mët ÷íng tròn. L§y mët iºm 
D trên cung BC (không chùa A) cõa ÷íng tròn ó. H¤ DH vuông góc vîi 
BC, DI vuông góc vîi CA và DK vuông góc vîi AB. Chùng minh r¬ng 
BC 
DH 
= 
AC 
DI 
+ 
AB 
DK 
Bài 5. Tìm t§t c£ các c°p sè nguyên d÷ìng (m, n) sao cho 2m+ 1 chia 
h¸t cho n và 2n + 1 chia h¸t cho m 
1.7 · thi tuyºn sinh lîp 10 nam 1992 
(cho thí sinh chuyên toán và chuyên tin) 
Bài 1. 
1. Tìm t§t c£ các sè nguyên n º n4 + 2n3 + 2n2 + n + 7 là sè chính 
ph֓ng. 
2. Cho a, b, c  0 và a + b + c  1. Chùng minh r¬ng 
1 
a2 + 2bc 
+ 
1 
b2 + 2ca 
+ 
1 
c2 + 2ab  9 
www.vnmath.com
1.8. · thi tuyºn sinh lîp 10 nam 1993 (cho måi thí sinh) 11 
Bài 2. Cho a là têng các chú sè cõa (29)1945, b là têng các chú sè cõa 
sè a. Tìm têng các chú sè cõa b. 
Bài 3. Cho tam giác ABC. Gi£ sû ÷íng phân giác trong và ngoài cõa 
góc A c­t ÷íng th¯ng BC t¤i D,K t÷ìng ùng. Chùng minh r¬ng n¸u 
AD = AK thì AB2+AC2 = 4R2, trong ó R là bán kính ÷íng tròn ngo¤i 
ti¸p tam giác ABC 
Bài 4. Trong m°t ph¯ng k´ 1992 ÷íng th¯ng sao cho không có 2 ÷íng 
nào song song và không có ba ÷íng nào çng quy. Tam giác t¤o bði ba 
÷íng th¯ng trong sè các ÷íng th¯ng ã cho gåi là tam giác xanh n¸u 
nó không bà ÷íng th¯ng nào trong sè các ÷íng th¯ng còn l¤i c­t. 
1. Chùng minh r¬ng sè tam giác xanh không ít hìn 664. 
2. Chùng minh k¸t luªn m¤nh hìn: Sè tam giác xanh không ít hìn 1328. 
Bài 5. Có 41 thành phè ÷ñc nèi vîi nhau b¬ng các ÷íng ch¿ i ÷ñc 
mët chi·u. Bi¸t r¬ng tø méi thành phè có úng 16 ÷íng ¸n các thành 
phè khác và úng 16 ÷íng tø các thành phè khác ¸n nó. Giúa hai thành 
phè b§t ky không có quá mët con ÷íng cõa m¤ng ÷íng nói trên. Chùng 
minh r¬ng tø mët thành phè b§t ky A ·u có thº i ¸n mët thành phè 
b§t ky B mà ch¿ i qua nhi·u nh§t hai thành phè trung gian. 
1.8 · thi tuyºn sinh lîp 10 nam 1993 
(cho måi thí sinh) 
Bài 1. 
1. Gi£i ph÷ìng trình 
x + 
s 
x + 
1 
2 
+ 
r 
x + 
1 
4 
= 2 
2. Gi£i h» ph÷ìng trình 
( 
x3 + 2xy2 + 12y = 0 
8y2 + x2 = 12 
Bài 2. Tìm giá trà lîn nh§t và bé nh§t cõa biºu thùc 
A = x2y(4 − x − y) 
khi x và y thay êi tho£ mãn i·u ki»n: x  0, y  0, x + y 6 6 
www.vnmath.com
12 Ch÷ìng 1. · thi tuyºn sinh lîp 10 
Bài 3. Cho hình thoi ABCD. Gåi R, r l¦n l÷ñt là bán kính các ÷íng 
tròn ngo¤i ti¸p các tam giác ABD,ABC và a là ë dài c¤nh hình thoi. 
Chùng minh r¬ng: 
1 
R2 + 
1 
r2 = 
4 
a2 
Bài 4. Cho tam giác ·u ABC nëi ti¸p ÷íng tròn tâm O bán kính R. 
Quay 4ABC mët góc 90 quanh tâm O ta ÷ñc 4A1B1C1. Tính di»n tích 
ph¦n chung cõa hai hình tam giác ABC và A1B1C1 theo R. 
Bài 5. Tìm t§t c£ các sè nguyên d÷ìng a, b, c ôi mët khác nhau sao 
cho biºu thùc 
A = 
1 
a 
+ 
1 
b 
+ 
1 
c 
+ 
1 
ab 
+ 
1 
ac 
+ 
1 
bc 
nhªn giá trà nguyên d÷ìng. 
1.9 · thi tuyºn sinh lîp 10 nam 1994 
(cho måi thí sinh) 
Bài 1. Gi£i các ph÷ìng trình sau: 
1. x4 − 2x3 − 6x2 + 16x − 8 = 0 
2. x2 + 2x + 4 = 3 
p 
x3 + 4x 
Bài 2. Xét các sè x, y, z, t  0 tho£ mãn h» thùc 
xy + 4zt + 2yz + 2xt = 9 
Tìm giá trà lîn nh§t cõa biºu thùc 
A = 
p 
zt 
p 
xy + 2 
Bài 3. Tìm t§t c£ các sè nguyên x, y, z, t tho£ mãn h» ph÷ìng trình 
( 
xy − 3zt = 1 
xz + yt = 2 
Bài 4. Cho tam giác cân ABC có AB = AC và H là trung iºm cõa 
c¤nh BC. Mët ÷íng tròn i qua A và ti¸p xúc vîi c¤nh BC t¤i B c­t 
AC,AH l¦n l÷ñt t¤i D và E. Bi¸t r¬ng D là trung iºm cõa AC và bán 
kính ÷íng tròn b¬ng R. Tính ë dài các dây cung AE,AD theo R. 
Bài 5. Cho tam giác ABC có BC  AC. Mët ÷íng th¯ng song song 
vîi c¤nh AB c­t các c¤nh BC và AC l¦n l÷ñt t¤i các iºm M và N. Chùng 
minh r¬ng BN  AM. 
www.vnmath.com
1.10. · thi tuyºn sinh lîp 10 nam 1994(cho thí sinh chuyên toán và chuyên tin)13 
1.10 · thi tuyºn sinh lîp 10 nam 1994 
(cho thí sinh chuyên toán và chuyên tin) 
Bài 1. Gi£i h» ph÷ìng trình 
8 
: 
(x + y)(y + z) = 4xy2z 
(y + z)(z + x) = 4yz2x 
(z + x)(x + y) = 4zx2y 
Bài 2. Tìm t§t c£ các c°p sè nguyên (x, y) tho£ mãn ph÷ìng trình 
12x2 + 6xy + 3y2 = 28(x + y) 
Bài 3. Xác ành các giá trà nguyên d÷ìng n(n  3) sao cho sè A = 
1, 2, 3 . . .n (tích cõa n sè nguyên d÷ìng ¦u tiên) chia h¸t cho sè B = 
1 + 2 + 3+· · · + n. 
Bài 4. Cho a, b, c  1. Chùng minh r¬ng 
1 
1 + a 
+ 
1 
1 + b 
+ 
1 
1 + c  1 
1 + 4 p 
ab3 
+ 
1 
1 + 4 p 
bc3 
+ 
1 
1 + 4 p 
ca3 
Bài 5. Cho 4ABC có AB = AC. 
1. Chùng minh r¬ng n¸u BAC = 20 thì luôn tìm ÷ñc các iºm D và 
K trên các c¤nh AB và AC sao cho AD = DK = KC = CB. 
2. Ng÷ñc l¤i, chùng minh r¬ng n¸u tçn t¤i các iºm D và K trên các 
c¤nh AB và AC sao cho AD = DK = KC = CB thì BAC = 20. 
1.11 · thi tuyºn sinh lîp 10 nam 1995 
(cho måi thí sinh) 
Bài 1. Gi£i h» ph÷ìng trình 
( 
2x2 − y2 = 1 
xy + x2 = 2 
Bài 2. Gi£i ph÷ìng trình 
p 
1 − x + 
p 
4 + x = 3 
www.vnmath.com
14 Ch÷ìng 1. · thi tuyºn sinh lîp 10 
Bài 3. Gi£ sû a, b là các sè nguyên d÷ìng sao cho: a+1 
b + b+1 
a là mët sè 
nguyên. Gåi d là ÷îc sè cõa a và b. Chùng minh r¬ng: d 6 
p 
a + b. 
Bài 4. Cho hai hình chú nhªt có cùng di»n tích. Hình chú nhªt thù nh§t 
có các kích th÷îc a và b (a  b). Hình chú nhªt thù hai có các kích th÷îc 
c và d (c  d). Chùng minh r¬ng: n¸u a  c thì chu vi cõa hình chú nhªt 
thù nh§t lîn hìn chu vi cõa hình chú nhªt thù hai. 
Bài 5. Cho ba iºm cè ành A,B,C th¯ng hàng theo thù tü §y. Gåi (
) 
là mët vòng tròn qua B và C. K´ tø A các ti¸p tuy¸n AE và AF ¸n vòng 
tròn (
). (E và F là các ti¸p iºm). Gåi O là tâm cõa vòng tròn (
), I là 
trung iºm cõa BC, N là trung iºm cõa EF. 
1. Chùng minh r¬ng: E và F n¬m trên mët vòng tròn cè ành khi vòng 
tròn (
) thay êi. 
2. ÷íng th¯ng FI c­t vòng tròn (
) t¤i E0. Chùng minh r¬ng EE0 song 
song vîi AB. 
3. Chùng minh r¬ng tâm vòng tròn ngo¤i ti¸p tam giác ONI n¬m trên 
mët ÷íng th¯ng cè ành khi vòng tròn (
) thay êi. 
1.12 · thi tuyºn sinh lîp 10 nam 1995 
(cho thí sinh chuyên toán và chuyên tin) 
Bài 1. Cho  
x + 
 
p 
x2 + 3 
y + 
 
= 3 
p 
y2 + 3 
Hãy tính giá trà cõa biºu thùc 
E = x + y 
Bài 2. Gi£i h» ph÷ìng trình 
8 
: 
x + xy + y = 1 
y + yz + z = 3 
z + zx + x = 1 
Bài 3. Cho x, y  0 và x2 + y2 = 1. Chùng minh r¬ng 
1 
p 
2 
6 x3 + y3 6 1 
Bài 4. Tìm sè nguyên có chín chú sè A = a1a2a3b1b2b3a1a2a3, trong 
24 
23 
22 
21 
ó a16= 0 và b1b2b3 = 2a1a2a3 çng thíi A có thº vi¸t ÷ñc d÷îi d¤ng 
A = ppppvîi p1, p2, p3, p4 là bèn sè nguyên khác nhau. 
www.vnmath.com
1.13. · thi tuyºn sinh lîp 10 nam 1996 (cho måi thí sinh) 15 
Bài 5. Cho vòng tròn (
), v³ hai dây cung AB và CD c­t nhau ð I (I 
n¬m trong vòng tròn). Gåi M là trung iºm cõa BD, MI kéo dài c­t AC 
ð N. Chùng minh r¬ng 
AN 
NC 
= 
AI2 
CI2 
1.13 · thi tuyºn sinh lîp 10 nam 1996 
(cho måi thí sinh) 
Bài 1. Cho x  0, hãy tìm giá trà nhä nh§t cõa biºu thùc 
P = 
 
x + 1 
x 
6 
− 
 
x6 + 1 
x6 
 
− 2 
 
x + 1 
x 
3 
+ x3 + 1 
x3 
Bài 2. Gi£i h» ph÷ìng trình 
8 
: 
p1 
x + 
q 
2 − 1 
y = 2 
1 p 
y + 
q 
2 − 1 
x = 2 
Bài 3. Chùng minh r¬ng vîi måi n nguyên d÷ìng ta có 
n3 + 5n 
...6 
Bài 4. Cho a, b, c  0, chùng minh r¬ng 
a3 
b 
+ 
b3 
c 
+ 
c3 
a  ab + bc + ca 
Bài 5. Cho hình vuông ABCD c¤nh b¬ng a. Gåi M,N,P,Q là các iºm 
b§t ky l¦n l÷ñt n¬m trên các c¤nh AB,BC,CD,DA. 
1. Chùng minh r¬ng 
2a2 6 MN2 + NP2 + PQ2 + QM2 6 4a2 
2. Gi£ sû M là mët iºm cè ành cho tr÷îc trên c¤nh AB. Hãy xác ành 
và trí cõa các iºm N, P,Q l¦n l÷ñt trên các c¤nh BC,CD,DA sao 
cho MNPQ là mët hình vuông. 
www.vnmath.com
16 Ch÷ìng 1. · thi tuyºn sinh lîp 10 
1.14 · thi tuyºn sinh lîp 10 nam 1996 
(cho thí sinh chuyên toán và chuyên tin) 
Ph¦n chung cho chuyên toán và chuyên tin 
Bài 1. Gi£i ph÷ìng trình 
( 
p 
x − 1 + 1)3 + 2 
p 
x − 1 = 2 − x 
Bài 2. Gi£i h» ph÷ìng trình 
8 
: 
x − 
p 
y = 1 
y − 
p 
z = 1 
z − 
p 
x = 1 
Bài 3. Cho x, y là nhúng sè nguyên d÷ìng thay êi tho£ mãn i·u ki»n 
x + y = 201 
Hãy tìm giá trà lîn nh§t và nhä nh§t cõa biºu thùc: P = x(x2+y)+y(y2+x). 
Bài 4. Cho o¤n th¯ng BC và ÷íng th¯ng (d) song song vîi BC. Bi¸t 
r¬ng kho£ng cách giúa ÷íng th¯ng (d) và ÷íng th¯ng i qua BC nhä hìn 
BC 
2 . Gi£ sû A là mët iºm thay êi trên ÷íng th¯ng (d). 
1. Hãy xác ành và trí cõa iºm A º bán kính vòng tròn ngo¤i ti¸p 
4ABC nhä nh§t 
2. Gåi ha, hb, hc là ë dài các ÷íng cao cõa 4ABC. Hãy xác ành và trí 
cõa iºm A º tích ha.hb.hc là lîn nh§t. 
Ph¦n dành cho chuyên toán 
Bài 5. Cho x, y, z  0 và x + y + z 6 3 
2. Chùng minh r¬ng: 
r 
x2 − 
1 
x2 + 
r 
y2 − 
1 
y2 + 
r 
z2 − 
1 
z2  3 
2 
p 
17 
Ph¦n dành cho chuyên tin 
Câu 5. Chia mët hình tròn thành 14 hình qu¤t b¬ng nhau. Trong méi 
hình qu¤t °t mët viên bi (xem hình v³). Gåi T là mët phép bi¸n êi: L§y 
hai hình qu¤t b§t ky có bi và chuyºn tø méi hình qu¤t ó mët viên bi sang 
hình qu¤t li·n k· nh÷ng theo hai chi·u ng÷ñc nhau (ví dö, n¸u viên bi ð 
mët hình qu¤t ÷ñc chuyºn theo chi·u kim çng hç thì viên bi ð hình qu¤t 
kia ÷ñc chuyºn theo chi·u ng÷ñc l¤i). Häi b¬ng vi»c thüc hi»n phép bi¸n 
êi trên, sau mët sè húu h¤n b÷îc ta có thº chuyºn ÷ñc t§t c£ các viên bi 
vào mët hình qu¤t ÷ñc không. N¸u có, hãy ch¿ ra quá trình bi¸n êi.N¸u 
không, hãy gi£i thích t¤i sao? 
www.vnmath.com
1.15. · thi tuyºn sinh lîp 10 nam 1997 (cho måi thí sinh) 17 
1.15 · thi tuyºn sinh lîp 10 nam 1997 
(cho måi thí sinh) 
Bài 1. Cho 
x = 
3 p 
p 
p 
p 
10 + 6 
3( 
3 − 1) 6 + 2 
p 
5 − 
p 
5 
Tính P = (x3 − 4x + 1)1997 
Bài 2. Gi£i ph÷ìng trình 
p 
x + 3+ 
p 
x + 8 = 5 
p 
x 
Bài 3. Gi£i h» ph÷ìng trình 
8 
: 
2xy = x + y + 1 
2yz = y + z + 7 
2xz = z + x + 2 
Bài 4. Tìm t§t c£ các sè tü nhiên n º 
2n + 15 
là sè chính ph÷ìng. 
Bài 5. Cho tam giác ·u ABC c¤nh l. Bên trong tam giác ta °t 2 
÷íng tròn (O,R) và (O0,R0) ti¸p xúc ngoài vîi nhau, sao cho mët trong 
hai ÷íng tròn ti¸p xúc vîi các c¤nh BC và BA, ÷íng tròn kia ti¸p xúc 
vîi các c¤nh BC và CA. 
1. Chùng minh r¬ng R + R0  
p 
3−1 
2 . 
2. Các bán kính R và R0 b¬ng bao nhiêu º têng di»n tích các hình tròn 
(O,R) và O0,R0 nhä nh§t và tính giá trà nhä nh§t ó. 
1.16 · thi tuyºn sinh lîp 10 nam 1997 
(cho thí sinh chuyên toán và chuyên tin) 
Bài 1. Gi£i h» ph÷ìng trình 
( 
y3 + y2x + 3x − 6y = 0 
x2 + xy = 3 
www.vnmath.com
18 Ch÷ìng 1. · thi tuyºn sinh lîp 10 
Bài 2. Có tçn t¤i hay không các sè nguyên x, y tho£ mãn i·u ki»n 
1992x1993 + 1993y1994 = 1995 
Bài 3. Sè 1997 vi¸t ÷ñc d÷îi d¤ng têng n hñp sè, nh÷ng không vi¸t 
÷ñc d÷îi d¤ng têng n + 1 hñp sè. Häi n b¬ng bao nhiêu? 
Bài 4. Cho các tam giác ABC ngo¤i ti¸p vòng tròn có bán kính b¬ng 
1. Gåi ha, hb, hc l¦n l÷ñt là ë dài các ÷íng cao h¤ tø ¿nh A,B,C tîi các 
c¤nh èi di»n. Hãy tìm giá trà lîn nh§t cõa biºu thùc. 
M = 
1 
ha + 2hb 
+ 
1 
hb + 2hc 
+ 
1 
hc + 2ha 
Bài 5. Trên ÷íng tròn cho 16 iºm và dùng 3 màu: xanh, ä, vàng º 
tô các iºm này (méi iºm tô b¬ng mët màu. Giúa méi c°p iºm nèi b¬ng 
mët o¤n th¯ng ÷ñc tô b¬ng màu tím ho°c màu nâu. 
Chùng minh r¬ng vîi måi cách tô màu trên các iºm (ch¿ dùng 3 màu: 
xanh, ä, vàng) và måi cách tô màu trên các o¤n th¯ng nèi giúa các c°p 
iºm (ch¿ dùng hai màu: tím ho°c nâu) ta ·u tìm ÷ñc trên hình v³ mët 
tam giác có ¿nh là các iºm ã cho, mà các ¿nh ÷ñc tô b¬ng cùng mët 
màu và các c¤nh cung ÷ñc tô b¬ng cùng mët màu (di nhiên khác màu tô 
trên ¿nh). 
1.17 · thi tuyºn sinh lîp 10 nam 1998 
(cho måi thí sinh) 
Bài 1. 
1. Gi£i ph÷ìng trình p 
2 − x2 + 
p 
x2 + 8 = 4 
2. Gi£i h» ph÷ìng trình 
( 
x2 + xy + y2 = 7 
x4 + x2y2 + y4 = 21 
Bài 2. Các sè a, b tho£ mãn i·u ki»n: 
( 
a3 − 3ab2 = 19 
b3 − 3a2b = 98 
Hãy tính giá trà cõa biºu thùc sau: P = a2 + b2. 
www.vnmath.com
1.18. · thi tuyºn sinh lîp 10 nam 1998(cho thí sinh chuyên toán và chuyên tin)19 
Bài 3. Cho các sè a, b, c 2 [0, 1]. Chùng minh r¬ng 
a + b2 + c3 − ab − bc − ca 6 1 
Bài 4. Cho ÷íng tròn () bán kính R. A và B là hai iºm cè ành trên 
÷íng tròn, (AB  2R). Gi£ sû M là mët iºm thay êi trên cung lîn AB 
cõa ÷íng tròn. 
1. K´ tø B ÷íng th¯ng vuông góc vîi AM, ÷íng th¯ng này c­t AM 
t¤i I và c­t ÷íng tròn () t¤i N. Gåi J là trung iºm cõa MN. 
Chùng minh r¬ng khi M thay êi trên ÷íng tròn thì méi iºm I, J 
·u n¬m trên mët ÷íng tròn cè ành. 
2. Xác ành và trí cõa iºm M º chu vi cõa 4AMB là lîn nh§t. 
Bài 5. 
1. Tìm t§t c£ các sè nguyên d÷ìng n sao cho méi sè n + 26 và n − 11 
·u là lªp ph÷ìng cõa mët sè nguyên d÷ìng. 
2. Cho các sè x, y, z thay êi tho£ mãn i·u ki»n: x2 + y2 + z2 = 1. 
Hãy tìm giá trà lîn nh§t cõa biºu thùc: 
P = xy + yz + zx + 
1 
2 
[x2(y − z)2 + y2(z − x)2 + z2(x − y)2] 
1.18 · thi tuyºn sinh lîp 10 nam 1998 
(cho thí sinh chuyên toán và chuyên tin) 
Bài 1. 
1. Gi£i h» ph÷ìng trình 
( 
x + x2 + x3 + x4 = y + y2 + y3 + y4 
x2 + y2 = 1 
2. Vîi nhúng giá trà nào cõa a thì ph÷ìng trình sau ây có nghi»m 
p 
1 − x + 
p 
1 + x = |1 − a| + |1 + a| 
Bài 2. Tìm nghi»m nguyên cõa ph÷ìng trình 
19x3 − 98y2 = 1998 
Bài 3. 
www.vnmath.com
20 Ch÷ìng 1. · thi tuyºn sinh lîp 10 
1. Cho a, b, c là các sè tho£ mãn hai i·u ki»n sau 
i) 0 a  b 
ii) Ph÷ìng trình ax2 + bx + c = 0 vô nghi»m. Chùng minh r¬ng 
a + b + c 
b − a 
 3 
2. Cho x, y, z  0. Hãy tìm giá trà nhä nh§t cõa biºu thùc 
P = 
x2 
x2 + 2yz 
+ 
y2 
y2 + 2zx 
+ 
z2 
z2 + 2xy 
Bài 4. Cho b£ng ô vuông kích th÷îc 1998 ×2000 (b£ng gçm 1998 hàng 
và 2000 cët) 
Ký hi»u (m, n) là ô vuông n¬m ð giao cõa hàng thù m (tính tø trên 
xuèng d÷îi)và cët thù n (tính tø trái qua ph£i). 
Cho các sè nguyên p, q vîi 1 6 p 6 1993 và 1 6 q 6 1995; 
Tô màu các ô vuông con cõa b£ng theo quy t­c: L¦n thù nh§t tô màu 
nam ô: (p, q); (p + 1, q + 1); (p + 2, q + 2); (p + 3, q + 3); (p + 4, q + 4). L¦n 
thù hai trð i, méi l¦n tô nam ô ch÷a có màu n¬m liên ti¸p trong cùng mët 
hàng ho°c cùng mët cët. 
Häi b¬ng cách ó ta có thº tô màu h¸t t§t c£ các ô vuông con cõa b£ng 
hay không? Vì sao? 
Bài 5. Cho tam giác ·u ABC. 
Trong 4ABC, v³ ba vòng tròn 1, 2, 3 có bán kính b¬ng nhau, ti¸p 
xúc ngoài l¨n nhau và méi vòng tròn ·u ti¸p xúc vîi hai c¤nh cõa tam 
giác. 
Gåi  là vòng tròn ti¸p xúc ngoài vîi c£ ba vòng tròn 1, 2, 3. Bi¸t bán 
kính cõa vòng tròn  là r, hãy tính ë dài c¤nh cõa 4ABC. 
1.19 · thi tuyºn sinh lîp 10 nam 1999 
(cho måi thí sinh) 
Bài 1. Cho các sè a, b, c tho£ mãn i·u ki»n 
( 
a + b + c = 0 
a2 + b2 + c2 = 14 
Hãy tính giá trà cõa biºu thùc: P = 1+a4 + b4 + c4 
Bài 2. 
www.vnmath.com
1.19. · thi tuyºn sinh lîp 10 nam 1999 (cho måi thí sinh) 21 
1. Gi£i ph÷ìng trình 
p 
x + 3 − 
p 
7 − x = 
p 
2x − 8 
2. Gi£i h» ph÷ìng trình 
( 
x + y + 1 
x + 1 
y = 9 
2 
xy + 1 
xy = 5 
2 
Bài 3. Tìm t§t c£ các sè nguyên d÷ìng n sao cho: n2 + 9n − 2 chia h¸t 
cho n + 11. 
Bài 4. Cho vòng tròn () và iºm I ð trong vòng tròn. Düng qua I hai 
dây cung b§t ky MIN và EIF. Gåi M0,N0,E0, F0 là các trung iºm cõa 
IM,IN,IE, IF. 
1. Chùng minh r¬ng tù giác M0E0N0F0 là tù giác nëi ti¸p. 
2. Gi£ sû I thay êi, các dây cung MIN,EIF thay êi. Chùng minh 
r¬ng vòng tròn ngo¤i ti¸p tù giác M0E0N0F0 có bán kính không êi. 
3. Gi£ sû I cè ành, các dây cung MIN,EIF thay êi nh÷ng luôn luôn 
vuông góc vîi nhau. Tìm và trí cõa các dây cung MIN và EIF sao 
cho tù giác M0E0N0F0 có di»n tích lîn nh§t. 
www.vnmath.com
22 Ch÷ìng 1. · thi tuyºn sinh lîp 10 
Bài 5. Các sè d÷ìng x và y thay êi tho£ mãn i·u ki»n: x + y = 1. 
Hãy tìm giá trà nhä nh§t cõa biºu thùc: 
P = 
 
x2 + 
1 
y2 
 
y2 + 
1 
x2 
 
Các thí sinh chuyên Sinh không ph£i làm bài 5 
1.20 · thi tuyºn sinh lîp 10 nam 1999 
(cho thí sinh chuyên toán và chuyên tin) 
Bài 1. Gi£i ph÷ìng trình 
r 
x + 7 
x + 1 
+ 8 = 2x2 + 
p 
2x − 1 
Bài 2. Các sè a1, a2, . . . ÷ñc xác ành bði công thùc 
ak = 
3k2 + 3k + 1 
(k2 + k)3 vîi måi k  1 
Hãy tính giá trà cõa têng: 1 + a1 + a2 + · · · + a9. 
Bài 3. Chùng minh r¬ng tçn t¤i mët sè chia h¸t cho 1999 và têng các 
chú sè cõa sè ó b¬ng1999. 
Bài 4. Cho vòng tròn tâm O bán kính R. Gi£ sû A và B là hai iºm cè 
p 
3. 
ành trên vòng tròn vîi AB = R 
1. Gi£ sû M là mët iºm thay êi trên cung lîn AB cõa ÷íng tròn. 
Vòng tròn nëi ti¸p 4MAB ti¸p xúc vîi MA t¤i E và ti¸p xúc vîi 
MB t¤i F. Chùng minh r¬ng ÷íng th¯ng EF luôn ti¸p xúc vîi mët 
÷íng tròn cè ành khi M thay êi. 
2. Tìm tªp hñp t§t c£ các iºm P sao cho ÷íng th¯ng 4 vuông góc vîi 
OP t¤i P c­t o¤n th¯ng AB. 
Bài 5. Cho hình tròn (C) bán kính b¬ng 1. Gi£ sû A1,A2, . . . ,A8 là 8 
iºm b§t ky n¬m tròn hình tròn (kº c£ biên). Chùng minh r¬ng trong các 
iºm ã cho luôn tçn t¤i hai iºm ma kho£ng cách giúa chúng nhä hìn 1. 
1.21 · thi tuyºn sinh lîp 10 nam 2000 
(cho måi thí sinh) 
Bài 1. 
www.vnmath.com
1.22. · thi tuyºn sinh lîp 10 nam 2000(cho thí sinh chuyên toán và chuyên tin)23 
1. Tính 
S = 
1 
1.2 
+ 
1 
2.3 
+ · · · + 
1 
1999.2000 
2. Gi£i h» ph÷ìng trình 
( 
x2 + 1 
y2 + x 
y = 3 
x + 1 
y + x 
y = 3 
Bài 2. 
1. Gi£i ph÷ìng trình 
p 
x − 1 + 
p 
x3 + x2 + x+ 1 = 1+ 
p 
x4 − 1 
2. Tìm t§t c£ các giá trà cõa a (a là sè thüc) º ph÷ìng trình 
2x2 − 
 
4a + 
11 
2 
 
x + 4a2 + 7 = 0 
có ít nh§t mët nghi»m nguyên. 
Bài 3. Cho ÷íng tròn tâm O nëi ti¸p trong hình thang ABCD(AB//CD), 
ti¸p xúc vîi c¤nh AB t¤i E và vîi c¤nh CD t¤i F (nh÷ hình v³) 
1. Chùng minh r¬ng 
BE 
AE 
= 
DF 
CF 
2. Cho bi¸t AB = a,CB = b, (a  b),BE = 2AE. Tính di»n tích hình 
thang ABCD. 
Bài 4. Cho x, y là hai sè thüc b§t ky khác không. Chùng minh r¬ng 
4x2y2 
(x2 + y2)2 + 
x2 
y2 + 
y2 
x2  3 
D§u ¯ng thùc x£y ra khi nào? 
1.22 · thi tuyºn sinh lîp 10 nam 2000 
(cho thí sinh chuyên toán và chuyên tin) 
Bài 1. 
www.vnmath.com
24 Ch÷ìng 1. · thi tuyºn sinh lîp 10 
1. Tìm t§t c£ các c°p sè nguyên (x, y) tho£ mãn ¯ng thùc y(x − 1) = 
x2 + 2 
2. Cho c°p sè (x, y) tho£ mãn các i·u ki»n 
−1 6 x + y 6 1, −1 6 xy + x + y 6 1 
Chùng minh r¬ng |x| 6 2, |y| 6 2 
Bài 2. 
1. Gi£i ph÷ìng trình 
1 
x 
+ 
r 
x − 
1 
x 
= x + 
r 
2x − 
5 
x 
2. Cho f(x) = ax2 + bx+c có tính ch§t f(1), f(4) và f(9) là các sè húu 
t. Chùng minh r¬ng khi ó a, b, c là các sè húu t. 
Bài 3. 
1. Cho tù giác lçi ABCD. Chùng minh r¬ng n¸u các góc B và D cõa tù 
giác là vuông ho°c tù thì AC  BD 
2. Cho o¤n th¯ng AC cè ành và iºm B di ëng. Hãy tìm tªp hñp t§t 
c£ các iºm B º tam giác ABC là tam giác không tù và góc [BAC 
là góc bé nh§t cõa tam giác ABC. 
Bài 4. Trên m°t ph¯ng cho 6 iºm sao cho không có ba iºm nào th¯ng 
hàng và kho£ng cách giúa các c°p iºm là các sè khác nhau. Ta nèi méi c°p 
iºm bði mët o¤n th¯ng. Chùng minh r¬ng trong các o¤n th¯ng thu ÷ñc 
có mët o¤n th¯ng là c¤nh bé nh§t cõa mët tam giác có 3 ¿nh là 3 trong 
6 iºm ã cho çng thíi là c¤nh lîn nh§t cõa mët tam giác khác cung có 3 
¿nh là 3 trong 6 iºm ã cho. 
1.23 · thi tuyºn sinh lîp 10 nam 2001 
(cho måi thí sinh) 
Bài 1. Tìm các giá trà nguyên x, y tho£ mãn ¯ng thùc 
(y + 2)x2 + 1 = y2 
Bài 2. 
www.vnmath.com
1.24. · thi tuyºn sinh lîp 10 nam 2001(cho thí sinh chuyên toán và chuyên tin)25 
1. Gi£i ph÷ìng trình 
p 
x(3x + 1) − 
p 
p 
x2 
x(x − 1) = 2 
2. Gi£i h» ph÷ìng trình 
( 
x2 + xy + 2 = 3x + y 
x2 + y2 = 2 
Bài 3. Cho nûa vòng tròn ÷íng kính AB = 2a. Trên o¤n AB l§y 
iºm M. Trong nûa m°t ph¯ng bí AB chùa nûa vòng tròn, ta k´ hai tia 
Mx và My sao choAMx =BMx = 300. Tia Mx c­t nûa vòng tròn ð E, 
tia My c­t nûa vòng tròn ð F. K´ EE0, FF0 vuông góc xuèng AB. 
1. Cho AM = a 
2 , tính di»n tích hình thang vuông EE0F0F theo a. 
2. Khi iºm M di ëng trên AB, chùng minh r¬ng ÷íng th¯ng EF luôn 
ti¸p xúc vîi mët vòng tròn cè ành. 
Bài 4. Gi£ sû x, y, z là các sè thüc khác không tho£ mãn h» ¯ng thùc: 
( 
x 
 
1 
y + 1 
z 
 
+ y 
 
1 
z + 1 
x 
 
+ z 
 
1 
x + 1 
y 
 
= −2 
x3 + y3 + z3 = 1 
Hãy tính giá trà cõa biºu thùc 
P = 
1 
x 
+ 
1 
y 
+ 
1 
z 
Bài 5. Vîi x, y, z là nhúng sè thüc d÷ìng, hãy tìm giá trà lîn nh§t cõa 
biºu thùc 
M = 
xyz 
(x + y)(y + z)(z + x) 
1.24 · thi tuyºn sinh lîp 10 nam 2001 
(cho thí sinh chuyên toán và chuyên tin) 
Bài 1. 
1. Cho f(x) = ax2 + bx + c có tính ch§t f(x) nhªn giá trà nguyên khi x 
là sè nguyên. Häi các h» sè a, b, c có nh§t thi¸t ph£i là các sè nguyên 
hay không? T¤i sao? 
www.vnmath.com
26 Ch÷ìng 1. · thi tuyºn sinh lîp 10 
2. Tìm các sè nguyên không âm x, y tho£ mãn ¯ng thùc: 
x2 = y2 + 
p 
y + 1 
Bài 2. Gi£i ph÷ìng trình 
p 
x + 1 = x2 − 5x + 14 
4 
Bài 3. Cho các sè thüc a, b, x, y tho£ mãn h» 
8 
: 
ax + by = 3 
ax2 + by2 = 5 
ax3 + by3 = 9 
ax4 + by4 = 17 
Hãy tính giá trà cõa biºu thùc 
A = ax5 + by5 
B = ax2001 + by2001 
Bài 4. Cho o¤n th¯ng AB có trung iºm là O. Gåi d1, d2 là các ÷íng 
th¯ng vuông góc vîi AB t÷ìng ùng t¤i A và B. Mët góc vuông ¿nh O có 
mët c¤nh c­t d1 ð M, còn c¤nh kia c­t d2 ð N. K´ OH vuông góc xuèng 
MN. Vòng tròn ngo¤i ti¸p tam giác MHB c­t d1 ð iºm thù hai E khác 
M, MB c­t NA ð I, ÷íng th¯ng HI c­t EB ð K. Chùng minh r¬ng K 
n¬m trên mët vòng tròn cè ành khi góc vuông quay xung quanh ¿nh O. 
Bài 5. Cho 2001 çng ti·n, méi çng ti·n ÷ñc sìn mët m°t b¬ng màu 
ä và m°t kia b¬ng màu xanh. X¸p 2001 çng ti·n ó theo mët vòng tròn 
sao cho t§t c£ các çng ti·n ·u có m°t xanh ngûa lên phía trên. Cho phép 
méi l¦n êi m°t çng thíi 5 çng ti·n liên ti¸p c¤nh nhau. Häi vîi cách làm 
nh÷ th¸, sau mët sè húu h¤n l¦n ta có thº làm cho t§t c£ các çng ti·n ·u 
có m°t ä ngûa lên phía trên ÷ñc hay không? T¤i sao? 
1.25 · thi tuyºn sinh lîp 10 nam 2002 
(cho måi thí sinh) 
Bài 1. 
1. Gi£i ph÷ìng trình: 
p 
8 + 
p 
x + 
p 
5 − 
p 
x = 5 
www.vnmath.com
1.26. · thi tuyºn sinh lîp 10 nam 2002(cho thí sinh chuyên toán và chuyên tin)27 
2. Gi£i h» ph÷ìng trình 
( 
(x + 1)(y + 1) = 8 
x(x+ 1)+y(y + 1) + xy = 17 
Bài 2. Cho a, b, c là ë dài ba c¤nh cõa mët tam giác. Chùng minh r¬ng 
ph÷ìng trình x2 + (a + b + c)x + ab + bc + ca = 0 vô nghi»m. 
Bài 3. Tìm t§t c£ các sè nguyên n sao cho n2 + 2002 là mët sè chính 
ph֓ng. 
Bài 4. Tìm giá trà nhä nh§t cõa biºu thùc 
P = 
1 
1 + xy 
+ 
1 
1 + yz 
+ 
1 
1 + zx 
trong ó x, y, z là các sè d÷ìng thay êi tho£ mãn i·u ki»n x2+y2+z2 6 3. 
Bài 5. Cho hình vuông ABCD, M là iºm thay êi trên c¤nh BC (M 
không trùng vîi B) và N là iºm thay êi trên c¤nh CD (N không trùng 
vîi D) sao cho: 
MAN =MAB +NAD 
1. BD c­t AN và AM t÷ìng ùng t¤i P và Q. Chùng minh r¬ng nam 
iºm P,Q,M,C,N cùng n¬m trên mët ÷íng tròn. 
2. Chùng minh r¬ng ÷íng th¯ng MN luôn ti¸p xúc vîi mët ÷íng tròn 
cè ành khi M và N thay êi. 
3. Ký hi»u di»n tích cõa tam giác APQ là S1 là di»n tích cõa tù giác 
PQMN là S2. Chùng minh r¬ng t¿ sè S1 
S2 
không êi khi M và N thay 
êi. 
1.26 · thi tuyºn sinh lîp 10 nam 2002 
(cho thí sinh chuyên toán và chuyên tin) 
Bài 1. 
1. Gi£i ph÷ìng trình: 
p 
x2 − 3x + 2+ 
p 
x + 3 = 
p 
x − 2+ 
p 
x2 + 2x − 3 
2. Tìm nghi»m nguyên cõa ph÷ìng trình 
x + xy + y = 9 
www.vnmath.com
28 Ch÷ìng 1. · thi tuyºn sinh lîp 10 
Bài 2. Gi£i h» ph÷ìng trình 
( 
x2 + y2 + xy = 1 
x3 + y3 = x+ 3y 
Bài 3. Cho m÷íi sè nguyên d÷ìng 1, 2, . . . , 10. S­p x¸p m÷íi sè ó mët 
cách tuy ý thành mët hàng. Cëng méi sè vîi sè thù tü cõa nó trong hàng, 
ta ÷ñc m÷íi têng. Chùng minh r¬ng trong m÷íi têng ó tçn t¤i ít nh§t 
hai têng có chú sè tªn cùng gièng nhau. 
Bài 4. Tìm giá trà nhä nh§t cõa biºu thùc 
P = 
4a 
b + c − a 
+ 
9b 
a + c − b 
+ 
16c 
a + b − c 
trong ó a, b, c là ë dài ba c¤nh cõa mët tam giác. 
Bài 5. ÷íng tròn (C) tâm I nëi ti¸p tam giác ABC ti¸p xúc vîi các 
c¤nh BC,CA,AB t÷ìng ùng t¤i các iºm A0,B0,C0. 
1. Gåi các giao iºm cõa ÷íng tròn (C) vîi các o¤n IA,IB,IC l¦n 
l÷ñt là M,N,P. Chùng minh r¬ng các ÷íng th¯ng A0M,B0N,C0P 
çng quy. 
2. Kéo dài o¤n AI c­t ÷íng tròn ngo¤i ti¸p tam giác ABC t¤i D (khác 
A). Chùng minh r¬ng IB.IC 
ID = 2r trong ó r là bán kính ÷íng tròn 
(C). 
1.27 · thi tuyºn sinh lîp 10 nam 2003 
(cho måi thí sinh) 
Bài 1. Gi£i ph÷ìng trình 
( 
p 
x + 5 − 
p 
x + 2)(1 + 
p 
x2 + 7x + 10 = 3 
Bài 2. Gi£i h» ph÷ìng trình 
( 
2x3 + 3x2y = 5 
y3 + 6xy2 = 7 
Bài 3. Tìm các sè nguyên x, y tho£ mãn ¯ng thùc 
2y2x + x + y + 1 = x2 + 2y2 + xy 
Bài 4. Cho nûa ÷íng tròn (O) ÷íng kính AB = 2R (R là mët ë 
dài cho tr÷îc), M,N là hai iºm trên nûa ÷íng tròn (O) sao cho M thuëc 
cung AN và têng các kho£ng cách tø A,B ¸n ÷íng th¯ng MN b¬ng 
R 
p 
3. 
www.vnmath.com
1.28. · thi tuyºn sinh lîp 10 nam 2003(cho thí sinh chuyên toán và chuyên tin)29 
1. Tính ë dài o¤n MN theo R. 
2. Gåi giao iºm cõa hai dây AN và BM là I, giao iºm cõa các ÷íng 
th¯ng AM và BN là K. Chùng minh r¬ng bèn iºm M,N, I,K cùng 
n¬m trên mët ÷íng tròn. Tính bán kính cõa ÷íng tròn ó theo R. 
3. Tìm giá trà lîn nh§t cõa di»n tích tam giác KAB theo R khi M,N 
thay êi nh÷ng v¨n tho£ mãn gi£ thi¸t cõa bài toán. 
Bài 5. x, y, z là các sè thüc tho£ mãn i·u ki»n 
x + y + z + xy + yz + zx = 6 
Chùng minh r¬ng: x2 + y2 + z2  3. 
1.28 · thi tuyºn sinh lîp 10 nam 2003 
(cho thí sinh chuyên toán và chuyên tin) 
Bài 1. Cho ph÷ìng trình 
x4 + 2mx2 + 4 = 0 
Tìm giá trà cõa tham sè m º ph÷ìng trình có 4 nghi»m phân bi»t x1, x2, x3, x4 
tho£ mãn 
x41 
+ x42 
+ x43 
+ x44 
= 32 
Bài 2. Gi£i h» ph÷ìng trình 
( 
2x2 + xy − y2 − 5x + y + 2 = 0 
x2 + y2 + x + y −4 = 0 
Bài 3. Tìm các sè nguyên x, y tho£ mãn ¯ng thùc 
x2 + xy + y2 = x2y2 
Bài 4. Cho ÷íng tròn tâm O nëi ti¸p tam giác ABC ti¸p xúc vîi các 
c¤nh BC,CA,AB t÷ìng ùng t¤i các iºm D,E,F. ÷íng tròn tâm O0 bàng 
ti¸p trong góc [BAC cõa tam giác ABC ti¸p xúc vîi c¤nh BC và ph¦n kéo 
dài cõa các c¤nh AB,AC t÷ìng ùng t¤i các iºm P,M,N. 
1. Chùng minh r¬ng: BP = CD. 
2. Trên ÷íng th¯ngMN ta l§y các iºm I và K sao cho CK//AB,BI//AC. 
Chùng minh r¬ng các tù giác BICE và BKCF là các hình binh hành. 
www.vnmath.com
30 Ch÷ìng 1. · thi tuyºn sinh lîp 10 
3. Gåi (S) là ÷íng tròn i qua ba iºm I,K,P. Chùng minh r¬ng (S) 
ti¸p xúc vîi các ÷íng th¯ng BC,BI,CK. 
Bài 5. Sè thüc x thay êi và tho£ mãn i·u ki»n x2+(3−x)2  5. Tìm 
giá trà nhä nh§t cõa biºu thùc 
p = x4 + (3 − x)4 + 6x2(3 − x)2 
1.29 · thi tuyºn sinh lîp 10 nam 2004 
(cho måi thí sinh) 
Bài 1. 
1. Gi£i ph÷ìng trình 
|x + 1| + |x − 1| = 1+|x2 − 1| 
2. Tìm nghi»m nguyên cõa h» 
( 
2y2 − x2 − xy + 2y − 2x = 7 
x3 + y3 + x − y = 8 
Bài 2. Cho các sè thüc d÷ìng a và b tho£ mãn 
a100 + b100 = a101 + b101 = a102 + b102 
Hãy tính giá trà cõa biºu thùc 
P = a2004 + b2004 
Bài 3. Cho 4ABC có AB = 3cm,BC = 4cm,CA = 5cm. ÷íng cao, 
÷íng phân giác, ÷íng trung tuy¸n cõa tam giác k´ tø ¿nh B chia tam 
giác thành 4 ph¦n. Tính di»n tích méi ph¦n. 
Bài 4. Cho tù giác ABCD nëi ti¸p trong ÷íng tròn có hai ÷íng chéo 
AC và BD vuông góc vîi nhau t¤i H (H không trùng vîi tâm cõa ÷íng 
tròn). Gåi M và N l¦n l÷ñt là chân các ÷íng vuông góc h¤ tø H xuèng các 
÷íng th¯ng AB và BC; P và Q l¦n l÷ñt là giao iºm cõa ÷íng th¯ng MH 
và NH vîi các ÷íng th¯ng CD và DA. Chùng minh r¬ng ÷íng th¯ng 
PQ song song vîi ÷íng th¯ng AC và bèn iºm M,N,P,Q n¬m trên cùng 
mët ÷íng tròn. 
Bài 5. Tìm giá trà nhä nh§t cõa biºu thùc 
Q = 
1 
2 
x10 
y2 + 
y10 
x2 
 
+ 
1 
4 
(x16 + y16) − (1 + x2y2)2 
www.vnmath.com
1.30. · thi tuyºn sinh lîp 10 nam 2004(cho thí sinh chuyên toán và chuyên tin)31 
1.30 · thi tuyºn sinh lîp 10 nam 2004 
(cho thí sinh chuyên toán và chuyên tin) 
Bài 1. Gi£i ph÷ìng trình 
p 
x+ 3+ 
p 
x − 1 = 2 
Bài 2. Gi£i h» ph÷ìng trình 
(x + y)(x2 + y2) = 15(x − y)(x2 − y2) = 3 
Bài 3. Tìm giá trà nhä nh§t cõa biºu thùc 
P = 
(x3 + y3) − (x2 + y2) 
(x − 1)(y − 1) 
trong ó, x, y là nhúng sè thüc lîn hìn 1. 
Bài 4. Cho hình vuông ABCD và iºm M n¬m trong hình vuông. 
1. Tìm t§t c£ các và trí cõa iºm M sao choMAB =MBC =MCD = 
MDA 
2. Xét iºm M n¬m trên ÷íng chéo AC. Gåi N là chân ÷íng vuông 
góc h¤ tø iºmM xuèng AB và O là trung iºm cõa o¤n AM. Chùng 
minh r¬ng t sè OB 
CN có giá trà không êi khi M di chuyºn trên ÷íng 
chéo AC. 
3. Vîi gi£ thi¸t M n¬m trên ÷íng chéo AC, xét các ÷íng tròn (S1) và 
(S2) có ÷íng kính t÷ìng ùng là AM và CN. Hai ti¸p tuy¸n chung 
cõa (S1) và (S2) ti¸p xúc vîi (S2) t¤i P và Q. Chùng minh r¬ng ÷íng 
th¯ng PQ ti¸p xúc vîi (S1) 
Bài 5. Vîi sè thüc a, ta ành nghia ph¦n nguyên cõa sè a là sè nguyên 
lîn nh§t không v÷ñt quá a và ký hi»u là [a]. Dãy các sè x0, x1, x2, . . . ,xn, . . . 
÷ñc xác ành bði công thùc 
xn = 
hn + 1 
p 
2 
i 
− 
h n 
p 
2 
i 
Häi trong 200 sè {x0, x1, . . . ,x199} có bao nhiêu sè khác 0? (Cho bi¸t 
1, 41  
p 
2  1, 42). 
www.vnmath.com
32 Ch÷ìng 1. · thi tuyºn sinh lîp 10 
1.31 · thi tuyºn sinh lîp 10 nam 2005 
(cho måi thí sinh) 
Bài 1. Gi£i h» ph÷ìng trình 
( 
x + y + xy = 3 
x2 + y2 = 2 
Bài 2. Gi£i ph÷ìng trình 
p 
x + 3 + 2 
x + 4 
p 
3 − 2x = 11 
Bài 3. Tìm nghi»m nguyên cõa ph÷ìng trình 
x2 + 17y2 + 34xy + 51(x + y) = 1740 
Bài 4. Cho ÷íng tròn (O), (O0) n¬m ngoài nhau có tâm t÷ìng ùng là 
O và O0. Mët ti¸p tuy¸n chung ngoài cõa hai ÷íng tròn ti¸p xúc vîi (O) 
t¤i A và (O0) t¤i B. Mët ti¸p tuy¸n chung trong cõa hai ÷íng tròn c­t AB 
t¤i I, ti¸p xúc vîi (O) t¤i C và (O0) t¤i D. Bi¸t C n¬m giúa I và D. 
1. Hai ÷íng th¯ng OC,O0B c­t nhau t¤i M. Chùng minh r¬ng OM  
O0M. 
2. Ký hi»u (S) là ÷íng tròn i qua A,C,B và (S0) là ÷íng tròn i 
qua A,D,B. ÷íng th¯ng CD c­t (S) t¤i E khác C và c­t (S0) t¤i 
F khác D. Chùng minh r¬ng AF vuông góc vîi BE. 
Bài 5. Gi£ sû x, y, z là các sè d÷ìng thay êi và tho£ mãn i·u ki»n 
xy2z2 + x2z + y = 3z2. Hãy tìm giá trà lîn nh§t cõa biºu thùc 
P = 
z4 
1 + z4(x4 + y4) 
1.32 · thi tuyºn sinh lîp 10 nam 2005 
(cho thí sinh chuyên toán và chuyên tin) 
Bài 1. Gi£i ph÷ìng trình 
p 
2 − x + 
p 
2 + x + 
p 
4 − x2 = 2 
Bài 2. Gi£i h» ph÷ìng trình 
( 
x3 + y3 − xy2 = 1 
4x4 + y4 = 4x + y 
Bài 3. Gi£ sû x, y là nhúng sè không âm tho£ mãn i·u ki»n x2+y2 = 1 
www.vnmath.com
1.32. · thi tuyºn sinh lîp 10 nam 2005(cho thí sinh chuyên toán và chuyên tin)33 
1. Chùng minh r¬ng 1 6 x + y 6 
p 
2 
2. Tìm giá trà lîn nh§t và giá trà nhä nh§t cõa biºu thùc 
P = 
p 
1 + 2x + 
p 
1 + 2y 
Bài 4. Cho hinh vuông ABCD và iºm P n¬m trong tam giác ABC. 
1. Gi£ sû gócBPC = 1350. Chùng minh r¬ng 2PB2 + PC2 = PA2. 
2. Các ÷íng th¯ng AP và CP c­t các c¤nh BC và BA t÷ìng ùng t¤i 
các iºm M và N. Gåi Q là iºm èi xùng vîi B qua trung iºm cõa 
o¤n MN. Chùng minh r¬ng khi P thay êi trong 4ABC, ÷íng 
th¯ng PQ luôn i qua D. 
Bài 5. 
1. Cho a giác ·u (H) có 14 ¿nh. Chùng minh r¬ng trong 6 ¿nh b§t 
ky cõa (H) luôn có 4 ¿nh là các ¿nh cõa mët hình thang 
2. Có bao nhiêu phân sè tèi gi£n m 
n lîn hìn 1 (m, n là các sè nguyên 
d÷ìng) tho£ mãn m.n = 13860. 
www.vnmath.com
Ch֓ng 2 
áp án tuyºn sinh 
2.1 áp án tuyºn sinh lîp 10 nam 1989 
(cho måi thí sinh) 
Bài 1. Gåi tªp hñp các sè chính ph÷ìng là . P(x) = ax2 + bx + c Ta có 
P(0) = c 2  ! c = c21 
, c1 2 Z 
P(1) = a + b + c 2  
P(−1) = a − b + c 2  ! 
( 
a + b 2 Z 
a − b 2 Z 
! 
( 
2a = a1 2 Z 
2a = b1 2 Z 
P(4) = 16a + 4b + c21 
= k2, k 2 Z ! 8a1 + 2b1 = k2 − c21 
Do 8a1 + 2b1 ch®n nên k2 − c21 
ch®n hay k và c1 cùng tính ch®n, l´ nên 
k2 − c21 ... 
4 ! b1 
... 
2 ! b = 
b1 
2 
2 Z 
Do a − b 2 Z nên tø ây ta có a 2 Z 
P(2) = 4a+ 2b + c21 
= t2, t2 Z ! 4a+2b = t2 − c21 
! t2 − c21 
là ch®n 
suy ra t và c1 cùng tính ch®n, l´ nên ta có t2 − c21 
.. .4 ! b là ch®n 
Vªy a, b, c 2 Z và b ch®n. 
Bài 2. °t 
P = a2 + ab + b2 − 3a − 3b + 1989 
! 4P = a2 − 2ab + b2 + 3(a2 + b2 + 4 + 2ab − 4a − 4b) + 4.1989 − 12 
= (a − b)2 + 3(a + b − 2)2 + 4.1986  4.1986 
Suy ra P  1986, d§u = ¤t ÷ñc khi và ch¿ khi 
( 
a − b = 0 
a + b − 2 = 0 
! a = b = 1 
34 
www.vnmath.com
2.1. áp án tuyºn sinh lîp 10 nam 1989 (cho måi thí sinh) 35 
Vªy P ¤t giá trà bé nh§t b¬ng 1986, ¤t ÷ñc khi a = b = 1 
Bài 3. Gåi 52 sè nguyên d÷ìng b§t ky ã cho là a1, a2, . . . ,a52. Méi sè ai 
·u có d¤ng ai = 100bi +ci, trong ó bi, ci 2 N và 0  ci  99, (i = 1, 52). 
N¸u trong sè c1, c2, . . . , c52 có hai sè b¬ng nhau, gi£ sû ci = ck ! ai−ak = 
100(bi − bk) 
.. .100. 
N¸u t§t c£ c1, c2, . . . , c52 ôi mët khác nhau thì có ít nh§t 51 sè khác 
50, gi£ sû ó là c1, c2, . . . , c51. Khi ó ta °t di = 100 − ci thì d1, d2, . . . ,d51 
là các sè nguyên khác nhau và 1  di  100. Nh÷ vªy 102 sè c1, c2, . . . , c52, 
d1, d2, . . . ,d51 ch¿ nhªn không quá 101 giá trà (tø 0 ¸n 100) và do ó có 2 sè 
trong chúng b¬ng nhau. Do các sè c1, c2, . . . , c51 khác nhau và d1, d2, . . . ,d51 
khác nhau nên hai sè b¬ng nhau là ci và dk nào ó suy ra ci = dk = 100 − 
ck ! ci+ck = 100, ð ây i6= k vì ci6= 50 ! ai+ak = 100(bi+bk)+100 
... 
100. 
Bài 4 Kéo dài BE,CF các o¤n EI = BE và FK = CF. Khi ó 
4ABI,4ACK cân ð A vàBAK =CAK = 30. 
1. N¸u [BAC = 150 thì B,A,K th¯ng hàng, C,A,K th¯ng hàng và 
BK = BA = AK = IA + AC = IC. 
Do E,M, F là các trung iºm cõa IB,BC,CK nên EM = 1 
2IC = 
1 
2BK = MF )4MEF cân ð M 
2. Gåi giao iºm cõa IC và BK là O thì trong måi tr÷íng hñp ta ·u có 
A,B,O, I cùng n¬m trên mët ÷íng tròn và góc giúa hai tia BK,CI 
b¬ng 150. Tø ó ta cóMEF =MFE = 15. 
Bài 5. Gi£ sû theo thù tü 9 b¤n håc sinh là a1, a2, a3, a4, a5, a6, a7, a8, a9. 
Ta chùng minh bài toán b¬ng ph£n chùng. 
Gi£ sû ng÷ñc l¤i: Không có b¤n nào ùng cách ·u hai b¤n cùng lîp (1). 
Không m§t têng quát gi£ sû a5 là håc sinh lîp A, khi ó a4 và a6 không 
thº cùng thuëc lîp A. Vì vªy có hai kh£ nang sau: 
1. a4 và a6 cùng thuëc lîp B. Khi ó do a4 cách ·u a2 và a6, còn a6 
cách ·u a4 và a8 nên a2 và a8 thuëc lîp A suy ra a5 ùng cách ·u 
hai b¤n cùng lîp là a2 và a8, trái vîi gi£ thi¸t (1). 
2. a4 và a6 thuëc hai lîp khác nhau, không m§t têng quát gi£ sû a4 thuëc 
lîp A còn a6 thuëc lîp B. Do a4 cách ·u a3 và a5, nên a3 thuëc lîp 
B. Do a6 cách ·u a3 và a9 nên a9 thuëc lîp A. Do a5 cách ·u a1 và 
a9 nên a1 thuëc lîp B. Do a2 cách ·u a1, a3 nên a2 thuëc lîp A. Do 
a5 cách ·u a2, a8 nên a8 thuëc lîp B. Do a6, a8 thuëc lîp B nên a7 
thuëc lîp A. Nh÷ vªy a7 ùng cách ·u hai b¤n cùng lîp A là a5 và 
a9, trái vîi gi£ thi¸t (1). 
Vªy c£ hai kh£ nang a) và b) ·u d¨n ¸n vô lý nên i·u gi£ sû (1) là 
sai. 
www.vnmath.com
36 Ch÷ìng 2. áp án tuyºn sinh 
2.2 áp án tuyºn sinh lîp 10 nam 1989 
(cho thí sinh chuyên lý) 
Bài 1. 
f(x) = 
−2x2 + x + 36 
2x + 3 
= −x + 2+ 
30 
2x + 3 
Vîi x nguyên f(x) là sè nguyên khi và ch¿ khi 2x + 3 là ÷îc sè cõa 
30. Do 2x + 3 là l´ nên 2x + 3 ch¿ có thº nhªn mët trong tám giá trà là 
±1,±3,±5,±15, tø ó thu ÷ñc 8 giá trà c¦n tìm cõa x là −9,−4,−3,−2,−1, 0, 1, 6. 
Bài 2. °t P = a2 + ab + b2 − 3a − 3b + 3 ta có 
4P = a2 − 2ab + b2 + 3(a2 + b2 + 4 + 2ab − 4a − 4b) 
= (a − b)2 + 3(a + b − 2)2  0 
Suy ra P  0, d§u = ¤t ÷ñc khi và ch¿ khi 
( 
a − b = 0 
a + b − 2 = 0 
) a = b = 1 
Vªy P ¤t giá trà bé nh§t b¬ng 0, ¤t ÷ñc khi và ch¿ khi a = b = 1 
Bài 3 
1. Vîi m nguyên d÷ìng ta có m2  m2 + m + 1  (m + 1)2. Do ó 
m 
p 
m2 + m + 1  m+1 ! 
p 
m2 + m + 1 không là sè nguyên hay 
m2 + m + 1 không là sè chính ph÷ìng. 
2. Gi£ sû ng÷ñc l¤i: m(m+1) b¬ng tích cõa 4 sè nguyên liên ti¸p, tùc là 
tçn t¤i a 2 Z mà m(m+1) = a(a+1)(a +2)(a+3) = (a2 +3a)(a2 + 
3a + 2) = n(n + 2), vîi n = a2 + 3a 2 Z. 
Vªy m(m+1) = n(n+2) suy ra m2 +m+1 = n2 +2n+1 = (n+1)2 
hay m2 +m+1 là sè chính ph÷ìng, trái vîi k¸t luªn câu 1) nên i·u 
gi£ sû m(m+ 1) b¬ng tích cõa 4 sè nguyên liên ti¸p là sai. 
Bài 4. 
www.vnmath.com
2.3. áp án tuyºn sinh lîp 10 nam 1989 (cho thí sinh chuyên toán - tin håc) 37 
Gi£ sû AH c­t MC ð I. Gåi trung iºm cõa BH là K thì MK//AM. 
D¹ th§y ba tam giác vuông AMC, IAC và IMA çng d¤ng mà AC = 2AM 
nên IC = 2IA = 4IM suy ra 
MK 
MC 
= 
IM 
IC 
= 
1 
4 
! 
BH 
HC 
= 
2HK 
HC 
= 
1 
2 
Vªy 
BH 
HC 
= 
1 
2 
Bài 5. Gåi 6 thành phè ã cho là A,B,C,D,E,F. 
Xét thành phè A. Trong 5 thành phè còn l¤i thì có ít nh§t 3 thành phè 
liên l¤c ÷ñc vîi A ho°c có ít nh§t ba thành phè không liên l¤c ÷ñc vîi A 
(vì n¸u sè thành phè liên l¤c ÷ñc vîi A không v÷ñt quá 2 và sè thành phè 
không liên l¤c ÷ñc vîi A cung không v÷ñt quá 2 thì ngoài A, sè thành phè 
còn l¤i không v÷ñt quá 4). Ta xét c£ hai kh£ nang. 
a) Sè thành phè liên l¤c ÷ñc vîi A không ít hìn 3, gi£ sû B,C,D liên 
l¤c ÷ñc vîi A. Theo gi£ thi¸t, trong 3 thành phè B,C,D có hai thành phè 
liên l¤c ÷ñc vîi nhau, khi ó hai thành phè này cùng vîi A là ba thành 
phè (ôi mët) liên l¤c ÷ñc vîi nhau. 
b) Sè thành phè không liên l¤c ÷ñc vîi A không ít hìn 3, gi£ sû ba 
thành phè không liên l¤c ÷ñc vîi A là D,E,F. Khi ó trong bë ba thành 
phè (A,D,E) thì D và E liên l¤c ÷ñc vîi nhau (vì D,E không liên l¤c 
÷ñc vîi A). 
T÷ìng tü, trong các bë ba (A,E, F), (A,F,D) thì E và F liên l¤c ÷ñc 
vîi nhau, F và D liên l¤c ÷ñc vîi nhau và nh÷ vªy D,E,F là ba thành 
phè (ôi mët) liên l¤c ÷ñc vîi nhau. 
2.3 áp án tuyºn sinh lîp 10 nam 1989 
(cho thí sinh chuyên toán - tin håc) 
Bài 1. 
a4 + b4 + c4 − 2a2b2 − 2b2c2 − 2c2a2 = 
= (a2 + b2 − c2)2 − 4a2b2 
= (a2 + b2 − c2 + 2ab)(a2 + b2 − c2 − 2ab) 
= [(a + b)2 − c2][(a− b)2 − c2] 
= (a + b + c)(a + b − c)(a − b + c)(a − b − c) 
Bài 2. 
www.vnmath.com
38 Ch÷ìng 2. áp án tuyºn sinh 
1. 
x 
x2 + x+ 1 
= − 
2 
3 
) 
x2 + x + 1 
x2 = − 
3 
2 
)x + 
1 
x 
+ 1 = − 
3 
2 
) x + 
1 
x 
= − 
5 
2 
) 
x4 + x2 + 1 
x2 = x2 + 
1 
x2 + 1 = 
 
x + 
1 
x 
2 
−1 = 
25 
4 
−1 = 
21 
4 
Vªy 
x2 
x4 + x2 + 1 
= 
4 
21 
Chú ý: Có thº gi£i ph÷ìng trình x 
x2+x+1 = −2 
3, thu ÷ñc hai nghi»m 
2 . Tø ó ta có: x2 
x4+x2+1 = 4 
21. 
x1 = 2 và x2 = 1 
2. 
P(x) = 
x2 
x4 + x2 + 1 
Ta có P(x)  0 vîi måi x6= 0 và P(0) = 0. 
Vîi x6= 0 ta có 1 
P(x) = x2 + 1 
x2 + 1 3 (vì x2 + 1 
x2  2) 
D§u = ¤t ÷ñc khi và ch¿ khi x2 = 1 ! x = ±1. Vªy P(x)  1 
3 
hay P(x) ¤t giá trà lîn nh§t b¬ng 1 
3, ¤t ÷ñc khi và ch¿ khi x = ±1. 
Bài 3. 
P(n) = an + bn + c ) 
8 
: 
P(1) = a + b + c 
P(2) = a2 + 2b + c 
P(3) = a3 + 3b + c 
Tø ó ta có 
P(2) − P(1) 
... 
.. .m 
m , a(a − 1) + b 
! 2ba(a − 1) + 2b2... 
m (1) 
và 
[a(a − 1) + b]2... 
m , a2(a − 1)2 + 2ba(a − 1) + b2.. .m (2) 
P(1) + P(3) − 2P(2) 
... 
.. .m ) a(a − 1)2... 
m , a3 − 2a2 + a 
m ) a2(a − 1)2.. .m 
www.vnmath.com
2.3. áp án tuyºn sinh lîp 10 nam 1989 (cho thí sinh chuyên toán - tin håc) 39 
Do ó, tø (2) ta có 
2ba(a − 1) + b2... 
m (3) 
Tø (1) và (3) ta suy ra b2... 
m. 
Có thº th§y r¬ng tø gi£ thi¸t cõa bài toán không suy ra ÷ñc b 
.. .m. Thªt 
vªy, vîi a = 3, b = 2, c = 3 thì P(n) = 3n + 2n + 3. Chån m = 4 khi ó: 
.. .4, do ó P(n) = 3n + 1 + 2(n + 1) 
Vîi n l´ thì d¹ th§y 3n + 1 
... 
4 
... 
Vîi n ch®n thì d¹ th§y 3n + 3 
4, do ó P(n) = 3n + 3 + 2n 
... 
4 
Vªy vîi måi sè nguyên d÷ìng n ta có P(n) 
... 
4, nh÷ng b = 2 không chia 
h¸t cho 4. 
Bài 4. Gåi trung iºm cõa CF là O thì MIOH và KLON là các hình 
bình hành. Suy ra hai o¤n MO, IH có chung trung iºm P và hai o¤n 
KO,LN có chung trung iºm Q. Có hai kh£ nang: 
1. M,O,K không th¯ng hàng. Khi ó MQ là ÷íng trung tuy¸n chung 
cõa 4MLN và 4MOK. Do vªy trång tâm G cõa 4MLN cung là 
trång tâm 4MOK. T÷ìng tü, KP là ÷íng trung tuy¸n chung cõa 
4MOK và 4IKH nên trång tâm G cõa 4MOK cung là trång tâm 
4IKH. Vªy trång tâm hai tam giác MLN và IKH trùng nhau. 
2. M,O,K th¯ng hàng. Gåi G và G0 là trång tâm 4MLN và 4IKH 
t÷ìng ùng. °t MP = PO = a,OQ = QK = b. 
Khi ó 
MG = 
2 
3 
MQ = 
2 
3 
(2a + b) = 
4 
3 
a + 
2 
3 
b 
KG0 = 
2 
3 
KP = 
2 
3 
(2b + a) = 
4 
3 
b + 
2 
3 
a 
) MG0 = MK − KG0 = 2a + 2b − 
4 
3 
b + 
2 
3 
 
= 
a 
4 
3 
a + 
2 
3 
b = MG 
Do G,G0 cùng thuëc o¤n MK nên tø ây ta có G0  G 
Bài 5. Gåi têng sè håc sinh là T thì T = a1 + a3 + · · · + an. Gåi sè lîp 
có úng i håc sinh là pi (i = 1,M). 
1. D¹ th§y T = 1p1 + 2p2 + 3p3 + · · · + MpM. M°t khác do dk là sè lîp 
www.vnmath.com
40 Ch÷ìng 2. áp án tuyºn sinh 
mà trong méi lîp ó có sè håc sinh không ít hìn k nên 
d1 = p1 + p2 + p3 + · · · + pM−1 + pM (1) 
d2 = p2 + p3 + · · · + pM−1 + pM (2) 
d3 = p3 + · · · + pM−1 + pM (3) 
. . . . . . 
dM−1 = pM−1 + pM (M-1) 
dM = pM (M) 
Cëng các ¯ng thùc trên l¤i ta ÷ñc 
d1+d2+· · ·+dM = 1p1+2p2+3p3+· · ·+MpM = T = a1+a2+· · ·+an 
2. Ta có a21 
+a22 
+· · · +a2 
n = p112 + p222 + · · ·+pMM2. M°t khác, nhân 
các ¯ng thùc (1), (2), (3), . . . , (M) ð câu 1) vîi 1, 3, 5, . . . , (2M − 1) 
t÷ìng ùng rçi cëng l¤i ta ÷ñc 
d1 + 3d2+5d3 + · · · + (2M − 1)dM = 
= p1 + (1 + 3)p2 + (1+3 + 5)p3 + · · ·+ 
+ (1 + 3 + 5+· · · + (2M − 1))pM 
= 12P1 + 22p2 + 33p3 + · · · +M2pM = a21 
+ a22 
+ · · · + a2 
n 
(Chú ý r¬ng 1+3 + 5 + (2k − 1) = k2) 
2.4 áp án tuyºn sinh lîp 10 nam 1991 
(cho måi thí sinh) 
Bài 1. 
1. p 
a + x + 
p 
a − x 
p 
a + x − 
p 
a − x 
= 
p 
b (1) 
º các can có nghia ta ph£i có −a  x  a. Do v¸ ph£i d÷ìng nên p 
p 
a + x − 
a −x  0 suy ra x  0. 
Vªy i·u ki»n èi vîi x là: 0  x  a. Vîi i·u ki»n ó (1) t÷ìng 
÷ìng vîi 
p 
a2 − x2 
2a + 2 
p 
a2 − x2 
2a − 2 
= b ) 
2a 
1 − 
p 
a2 − x2 
−1 = b 
2a 
b + 1 
= a − 
p 
a2 − x2 ) 
p 
a2 − x2 = 
a(b − 1) 
b + 1 
(2) 
www.vnmath.com
2.4. áp án tuyºn sinh lîp 10 nam 1991 (cho måi thí sinh) 41 
Do ó 
N¸u b  1 thì (2) vô nghi»m do ó (1) vô nghi»m 
N·u b  1 thì (2) t÷ìng ÷ìng vîi 
a2 − x2 = 
a2(b − 1)2 
(b+ 1)2 
) x2 = a2 4b 
(b + 1)2 
) x = ± 
2a 
b + 1 
p 
b 
Lo¤i nghi»m âm ta ÷ñc nghi»m x = 2a 
b+1 
p 
b tho£ mãn i·u ki»n 
p 
b). 
0  x  a (vì b + 1  2 
p 
b 
b+1 
Vªy (1) có nghi»m khi và ch¿ khi b  1 và nghi»m ó là x = 2a 
2. 
x2 + ax + b + 1 = 0 (1) 
Gi£ sû x1, x2 là hai nghi»m nguyên cõa (1). Do b6= −1 nên x16= 0 và 
x26= 0. Theo ành lý Viét ta có 
( 
x1 + x2 = −a 
x1x2 = b + 1 
! a2 + b2 = (x1 + x2)2 + (x1x2 − 1)2 = x21 
+ x22 
+ x21 
x22 
+ 1 
= (x21 
+ 1)(x22 
+ 1) 
Do x1, x2 là các sè nguyên khác 0 nên x21 
+1 và x22 
+1 là các sè nguyên 
không bé hìn 2 và nh÷ vªy a2 + b2 là hñp sè. 
Bài 2. 8 
: 
a3x + a2y + az = 1 
b3x + b2y + bz = 1 
c3x + c2y + cz = 1 
Nhân (1) vîi b và (2) vîi a rçi trø tøng v¸ cho nhau, sau ó chia cho 
a − b6= 0 ta ÷ñc ph÷ìng trình 
ab(a + b)x + aby = −1 (4) 
T÷ìng tü, nhân (1) vîi c và (3) vîi a rçi trø tøng v¸ cho nhau, sau ó chia 
cho a − c6= 0 ta ÷ñc 
ac(a + c)x + acy = −1 (5) 
www.vnmath.com
42 Ch÷ìng 2. áp án tuyºn sinh 
Khi ó h» ã cho t÷ìng ÷ìng vîi 
8 
: 
a3x + a2y + az = 1 
ab(a + b)x + aby = −1 
ac(a + c)x + acy = −1 
Nhân (4) vîi c, (5) vîi b rçi trø tøng v¸ cho nhau, sau ó chia cho b−c6= 0 
ta ֖c abcx = 1 hay x = 1 
abc . Thay x = 1 
abc vào (4) thu ÷ñc y = −a+b+c 
abc . 
Thay x = 1 
abc, y = −a+b+c 
abc vào (1) thu ÷ñc z = ab+bc+ca 
abc . Vªy nghi»m cõa 
h» ã cho là 
x = 
1 
abc 
, y= − 
a + b + c 
abc 
, z= 
ab + bc + ca 
abc 
Bài 3 D¹ th§y 7x chia 4 d÷ 3 n¸u x l´ và d÷ 1 n¸u x ch®n. Ph÷ìng trình 
ã cho t÷ìng ÷ìng vîi 
7x − 1 = 3.2y (1) 
N¸u x l´ thì 7x − 1 chia 4 d÷ 2 còn vîi y  2 thì 3.2y... 
4 do ó y ch¿ có 
thº là 1. Vîi y = 1 ta ÷ñc nghi»m là x = 1, y = 1. 
N¸u x ch®n tùc là x = 2z (z nguyên d÷ìng) ph÷ìng trình (1) có d¤ng 
(7z + 1)(7z − 1) = 3.2y (2) 
Vì 2,3 là các sè nguyên tè, nên (2) là d¤ng phân tích cõa (7z + 1)(7z − 1) 
thành tích các thøa sè nguyên tè. 
Do 7z +1 chia 3 d÷ 2 nên 7z +1 = 2n.(3) vîi n là sè nguyên d÷ìng nào 
ó. Tø ó ta có 7z −1 = 2n − 2 suy ra (2) có d¤ng 
2n(2n − 2) = 3.2y ) 2n+1(2n−1 − 1) = 3.2y 
Do ó 2n−1 −1 không chia h¸t cho 2 nên 2n−1 −1 = 3 hay n = 3. Thay vào 
(3) ta thu ÷ñc z = 1 suy ra x = 2. Thay x = 2 vào (1) ta ÷ñc nghi»m là 
x = 2, y = 4. 
Vªy ph÷ìng trình ã cho có hai nghi»m nguyên d÷ìng là x = 1, y = 1 
và x = 2, y = 4 
Bài 4. 
1. Gåi các giao iºm cõa EF vîi AB,CD t÷ìng ùng là I,K. Qua F k´ 
÷íng th¯ng song song vîi AB và CD c­t AD ð M, c­t BC ð N. 
Ta có 
MF 
DC 
= 
AM 
AD 
= 
BN 
BC 
= 
FN 
DC 
) MF = FN 
www.vnmath.com
2.4. áp án tuyºn sinh lîp 10 nam 1991 (cho måi thí sinh) 43 
Do AB//MN,CD//MN nên ba ÷íng th¯ng çng quy EC,ED,EK 
c­t các ÷íng th¯ng AB,MN,DC thành các o¤n th¯ng t l» 
IA 
IB 
= 
KD 
KC 
= 
FM 
FN 
= 1 
Suy ra EF i qua các trung iºm I,K cõa AB và CD 
2. 
SIJK = SNJC ) SICK = SICN ) KN//IC 
Khi ó theo (1) thì AJ i qua trung iºm E cõa KN suy ra SAKJ = 
SANJ mà SAKP = SCJN nên 
SAPJ = SAIC ) PJ = JC ) SBPJ = SBJC 
Do SBIM = SIJK nên tø ây ta có SBIKP = SCJIM. T÷ìng tü ta chùng 
minh ÷ñc SCJIM = SAKJN. Vªy di»n tích cõa ba tù giác không g¤ch 
chéo b¬ng nhau. 
Bài 5. Trên nûa ÷íng tròn ÷íng kính AB ta l§y 1991 iºm khác 
nhau và khác A,B : A1,A2,A3, . . . ,A1991. Gi£ sû Ai,Aj,Ak là ba iºm b§t 
www.vnmath.com
44 Ch÷ìng 2. áp án tuyºn sinh 
ky trong chúng, khi ó Ai,Aj,Ak không th¯ng hàng. Gi£ sû trên nûa ÷íng 
tròn ã cho AJ n¬m giúa Ai và Ak (tùc Aj thuëc cung nhä AjAk) khi ó 
AiAjAk  90 và 4AiAjAk là tam giác tù 
Vªy trên m°t ph¯ng tçn t¤i 1991 iºm mà ba iºm b§t ky trong chúng 
là ba ¿nh cõa mët tam giác tù. 
2.5 áp án tuyºn sinh lîp 10 nam 1991 
(cho thí sinh chuyên toán và chuyên tin) 
Bài 1. 
1. Rút gån 
A = 
q 
3 
p 
3 − 4 
2 
p 
2 
q 
6 
44 + 16 
p 
6 
= 
q 
3 
p 
3 − 4 
2 
q 
(2 
p 
2 6 
p 
3 + 4 
p 
2)2 
q 
= 3 
(2 
p 
3 − 4 
p 
2)(2 
p 
3 + 4 
p 
2) = − 3 p 
20 
2. P = (x − y)5 + (y − z)5 + (z − x)5 
°t a = x − y, b = y − z, z − x = −(a + b) Khi ó 
P = a5 + b5 − (a + b)5 = (a + b)[a4 − a3b + a2b2 − ab3 + b4 − (a + b)4] 
= (a + b)(a4 − a3b + a2b2 − ab3 + b4 − a4 − 4a3b − 6a2b2 − 4ab3 − b4) 
= (a + b)[−5(a3b + a2b2 + ab3)] = −5(a + b)ab(a2 + ab + b2) 
= 5(x − y)(y − z)(z − x)[(x − y)2 + (x − y)(y − z) + (y − z)2] 
= 5(x − y)(y − z)(z − x)(x2 + y2 + z2 − xy − yz − zx) 
Bài 2. 
1. 8 
: 
a + b + c = 0 
 +
+ 
 = 0 
 
a +
b + 
 
c = 0 
www.vnmath.com
2.5. áp án tuyºn sinh lîp 10 nam 1991(cho thí sinh chuyên toán và chuyên tin)45 
Tø ph÷ìng trình 1 và 2 ta có c = −(a+b), 
= −(+
). Thay vào 
ph÷ìng trình 3 ta ÷ñc 
 
a 
+
b 
+ 

 
c 
= 0 
! (b +
a)(a+ b) + ( +
)ab = 0 
! b2 +
a2 + 2ab( +
) = 0 
! b2 +
a2 − 2ab
 = 0 (1) 
T÷ìng tü
c2 + 
b2 − 2bc = 0 (2) 

a2 + c2 − 2ca
= 0 (3) 
Cëng (1), (2), (3) ta ÷ñc 
(
+ 
)a2 + (
 + )b2 + ( +
)c2 − 2(bc + ca
+ ab
) = 0 
!−(a2 +
b2 + 
c2) = 2(bc + ca
+ ab
) = 
 
a 
= 2abc 
+
b 
+ 

 
c 
 
= 0 
Vªy A = a2 +
+
b2 + 
c2 = 0 
2. 0  a, b, c, d  1. °t P = a + b + c + d − ab − bc − cd − da 
a) Ta có 
a − ab = a(1 − b)  0 d§u = ¤t ÷ñc , a = 0 ho°c b = 1 
(1) 
b − bc  0 d§u = ¤t ÷ñc , b = 0 ho°c c = 1 
(2) 
c − cd  0 d§u = ¤t ÷ñc , c = 0 ho°c d = 1 
(3) 
d − da  0 d§u = ¤t ÷ñc , d = 0 ho°c a = 1 
(4) 
Cëng bèn b§t ¯ng thùc trên ta ÷ñc P  0 
Gi£ sû d§u = ¤t ÷ñc, khi ó c£ bèn b§t ¯ng thùc ð (1), (2), (3), 
(4) ph£i là ¯ng thùc. Tø (1) ta có a = 0 ho°c b = 1. 
N¸u a = 0 thì tø (4) ta có d = 0, do ó tø (3) ta có c = 0 và tø (2) 
suy ra b = 0. Vªy a = b = c = d = 0 
www.vnmath.com
46 Ch÷ìng 2. áp án tuyºn sinh 
N¸u b = 1 thì tø (2) ta có c = 1, tø (3) ta có d = 1 và tø (4) suy ra 
a = 1. Vªy a = b = c = d = 1 
Ng÷ñc l¤i d¹ th§y vîi a = b = c = d = 0 ho°c a = b = c = d = 1 thì 
p = 0 
Tóm l¤i: P  0, P = 0 khi và ch¿ khi a = b = c = d = 0 ho°c 
a = b = c = d = 1 
b) Tø (1 − a)(a − b)  0 ! 1 − a − b + ab  0 ! a + b − ab  1. 
Vªy 
a + b − ab  1, d§u = ¤t ÷ñc , a = 1 ho°c b = 1 (5) 
T÷ìng tü 
b + c − bc  1, d§u = ¤t ÷ñc , b = 1 ho°c c = 1 (6) 
c + d − cd  1, d§u = ¤t ÷ñc , c = 1 ho°c d = 1 (7) 
d + a − da  1, d§u = ¤t ÷ñc , d = 1 ho°c a = 1 (8) 
M°t khác 
(a + c)(b + d)  0 ! ab + bc + cd + da  0 
!−(ab + bc + cd + da)  0 
D§u = ¤t ÷ñc , a + c = 0 ho°c b + d = 0 
! a = c = 0 ho°c b = d = 0 (9) 
Cëng các b§t ¯ng thùc ð (5),(6),(7),(8) và (9) ta ÷ñc 2p  4 hay 
p  2 
N¸u d§u = ¤t ÷ñc thì ð c£ 5 b§t ¯ng thùc ·u có d§u =, nh÷ 
vªy, tø (9) ta có a = c = 0 ho°c b = d = 0. 
Vîi a = c = 0 thì tø (5), (7) suy ra b = d = 1. 
Vîi b = d = 0 thì tø (5), (7) suy ra a = c = 1 
Ng÷ñc l¤i, vîi a = c = 0, b = d = 1 ho°c b = d = 0, a = c = 1 thì 
P = 2. 
Tóm l¤i P  2, P = 2 khi và ch¿ khi a = c = 0, b = d = 1 ho°c 
b = d = 0, a = c = 1 
Chú ý: Có thº gi£i cách khác nh÷ sau: 
 
P = (a + c)(1 − b − d) + b + d 
P = (b + d)(1 − a − c) + a + c 
! 
2P = (a + c)(2 − b − d) + (b + d)(2 − a − c) 
 
1 
4 
[(a + c + 2 − b − d)2 + (b + d + 2 − a − c)2] 
www.vnmath.com
2.5. áp án tuyºn sinh lîp 10 nam 1991(cho thí sinh chuyên toán và chuyên tin)47 
Do a + c + 2 − b − d  0, b+ d + 2 − a − c  0 nên tø ây ta có. 
2P  
1 
4 
[(a + c + 2 − b − d) + (b + d + 2 − a − c)]2 = 4 hay P  2 
D§u = ¤t ÷ñc khi và ch¿ khi 
 
a + c + 2 − b − d = 0 
b + d + 2 − a − c = 0 
! 
 
a = c = 0, b= d = 1 
b = d = 0, a = c = 1 
(vì 0  a, b, c, d  1) 
Bài 3. Gi£ sû k là sè tü nhiên tho£ mãn a  10k, d  10k. °t 
an = a + nd và xn = an 
1991 = a 
1991 + n d 
1991 
Do a 
1991  10k và d 
1991  0 nên tçn t¤i sè tü nhiên m mà 
xm−1  10k  xm (1) 
M°t khác 
xm = xm−1 + 
d 
1991 
 xm−1 + 
10k 
1991 
 10k + 
10k 
1991 
= 
1992.10k 
1991 
(2) 
Tø (1) và (2) ta có 
10k  xm  
1992.10k 
1991 
! 1991.10k  1991.xm  1992.10k 
Hay 
1991.10k  am  1992.10k 
Do ó am = a + md là sè có bèn chú sè ¦u tiên là 1991. 
Bài 4. Gåi 100 ng÷íi dü hëi th£o là a1, a2, a3, . . . ,a99, a100. Gi£ sû a1 
quen bi¸t vîi 67 ng÷íi là a2, a3, . . . ,a68. 
Khi ó không kº a1 và 32 ng÷íi là a69, a70, . . . ,a100 thì a2 quen bi¸t 
vîi ít nh§t 34 ng÷íi trong sè a3, a4, . . . ,a68. Không m§t têng quát, gi£ 
sû a2 quen bi¸t vîi a3, a4, . . . ,a36. T÷ìng tü, không kº a1, a2 và 64 ng÷íi là 
a37, a38, . . . ,a100 thì a3 quen bi¸t vîi ít nh§t mët ng÷íi trong sè a4, a5, . . . ,a36, 
ch¯ng h¤n a3 quen bi¸t vîi a4. 
Tø ó ta có 4 ng÷íi là a1, a2, a3, a4 ôi mët quen bi¸t nhau. 
Bài 5. 1) Cùng phía vîi hình vuông èi vîi CD düng tam giác ·u 
M0CD. Khi ó 4ADM cân ð D và 4BCM cân ð C nên ta có 
ADM0 =BCM0 = 90 − 60 = 30 !DAM0 =CBM0 = 75 
www.vnmath.com
48 Ch÷ìng 2. áp án tuyºn sinh 
M°t khác, d¹ th§y M0 n¬m cùng phía vîi hình vuông ABCD èi vîi AB. 
Theo gi£ thi¸t thì M n¬m trong hình vuông vàABM =BAM = 15 suy 
ra M0  M hay 4MCD ·u 
Chú ý: Có thº chùng minh b¬ng ph£n chùng nh÷ sau: 
Gi£ sû 4MCD không ·u. D¹ th§y 4AMD = 4BMC (c-g-c) suy ra 
MC = MD,MCD =MDC. Có hai kh£ nang 
a) MDC = MCD  60 suy ra MD  CD và ADM  30. Mà 
DAM = 75 nênDMA  75 ! AD  MD  CD vô lý (Vì ABCD là 
hình vuông). 
b)MDC =MCD  60, lý luªn t÷ìng tü d¨n ¸n i·u vô lý, do ó gi£ 
thi¸t 4MCD không ·u là sai. 
2) Trong hình vuông ABCD düng bèn iºm M,N,P,Q tho£ mãnMAB = 
MBA =NBC =NCB =PCD =PDC =QDA =QAD = 15 thì theo 
chùng minh trên, bèn tam giác MCD,NDA,PAB và QBC là ·u. 
Ta chùng minh tªp hñp 8 iºm {A,B,C,D,M,N, P,Q} tho£ mãn i·u 
ki»n bài toán. 
Có t§t c£ là 8.7 
2 = 28 o¤n th¯ng nèi hai trong tám iºm trên. Ta chia 
chúng thành 6 nhóm sau: 
a) 4 c¤nh AB,BC,CD,DA cõa hình vuông ABCD 
b) 2 ÷íng chéo AC,BD cõa hình vuông ABCD 
c) 4 c¤nh MN,NP,PQ,QM cõa hình vuông MNPQ (d¹ th§y MNPQ 
là hình vuông) 
d) 2 ÷íng chéo MP,NQ cõa hình vuông MNPQ 
www.vnmath.com
2.6. áp án tuyºn sinh lîp 10 nam 1992 (cho måi thí sinh) 49 
e) 8 o¤n MA,MB,NB,NC,PC,PD,QD,QA 
f) 8 o¤n MC,MD,ND,NA, PA, PB,QB,QC 
Ta chùng minh các o¤n ð nhóm e) và f) tho£ mãn i·u ki»n bài toán. 
(Vi»c chùng minh các o¤n ð các nhóm còn l¤i tho£ mãn i·u ki»n bài 
toán ìn gi£n hìn, b¤n åc tü chùng minh) 
Do 4AMD cân ð D vàADQ =MDQ = 15 nên DQ là trung trüc cõa 
AM hay trung trüc cõa AM i qua hai iºm D,Q, èi vîi các o¤n khác ð 
nhóm e) chùng minh t÷ìng tü. 
Do 4MCD ·u còn 4MQD cân ð Q nên trung trüc cõa MD i qua 
hai iºm C,Q. èi vîi các o¤n khác ð nhóm f) chùng minh t÷ìng tü. 
2.6 áp án tuyºn sinh lîp 10 nam 1992 
(cho måi thí sinh) 
Bài 1. 
1. q 
p 
2x − 5 + 
x + 2 + 3 
q 
p 
2x − 5 = 2 
x − 2 − 3 
p 
2 (1) 
i·u ki»n: x  5 
2 
Nhân hai v¸ vîi 
p 
2 ta có (1) t÷ìng ÷ìng vîi 
q 
p 
2x − 5 + 9+ 
(2x − 5) + 6 
q 
p 
2x − 5+1 = 4 
(2x − 5) − 2 
q 
) 
( 
p 
2x − 5 + 3)2 + 
q 
( 
p 
2x − 5 − 1)2 = 4 
p 
2x − 5 + 3+| 
) 
p 
2x − 5 − 1| = 4 (2) 
Vîi x  3 thì (2) có d¤ng 
p 
2x − 5 + 3+ 
p 
2x − 5 −1 = 4 ) 
p 
2x − 5 = 1 
)2x − 5 = 1 ) x = 3 (tho£ mãn x  3) 
2 6 x  3 thì (2) có d¤ng 
Vîi 5 
p 
2x − 5 + 3 + 1 − 
p 
2x −5 = 4 luôn 
tho£ mãn. 
Vªy nghi»m cõa (1) là 5 
2 6 x 6 3. 
2. ( 
xy2 − 2y + 3x2 = 0 
y2 + x2y + 2x = 0 
) 
( 
xy2 − 2y = −3x2 (1) 
x2y + 2x = −y2 (2) 
www.vnmath.com
50 Ch÷ìng 2. áp án tuyºn sinh 
D¹ th§y x = 0, y = 0 là mët nghi»m cõa h» và ng÷ñc l¤i n¸u (x, y) là 
nghi»m mà x ho°c y b¬ng 0 thì sè kia cung b¬ng 0. Ta tìm nghi»m 
tho£ mãn x6= 0, y6= 0. 
Vîi i·u ki»n x6= 0, y6= 0, chia (1) cho y2 và (2) cho x2 ta ÷ñc h» 
(1), (2) t÷ìng ÷ìng vîi 
( 
x − 2 
y = −3x2 
y2 (3) 
x = −y2 
x2 (4) 
y + 2 
Nhân hai ph÷ìng trình vîi nhau ta ÷ñc h» (3), (4) t÷ìng ÷ìng vîi 
( 
x − 2 
y = −3x2 
 y2 
x − 2 
y 
 
y + 2 
x 
 
= 3 
) 
( 
xy2 − 2y = −3x2 (1) 
xy − 4 
xy (5) 
Ta có (5) t÷ìng ÷ìng vîi 
(xy)2 − 3xy − 4 = 0 ) xy = 
 
−1 
4 
Thay xy = −1 hay y = −1 
x vào (1) ta ÷ñc 
1 
x 
+ 
2 
x 
= −3x2 ) x3 = −1 ) x = −1 ) y = 1 
Thay xy = 4 hay y = 4 
x vào (1) ta ÷ñc 
16 
x 
− 
8 
x 
= −3x2 ) x3 = − 
8 
3 
) x = − 
2 
3 p 
3 
) y = −2 
[ 
3 
]3 
Vªy h» ã cho có ba nghi»m là x = y = 0;x = −1, y = 1 và x = 
− 2 
3 p 
3 
, y = −2 3 p 
3 
Bài 2. 
x2 − mnx + m + n = 0 (1) 
Vîi m = 0 ph÷ìng trình (1) có d¤ng x2 + n = 0. N¸u ph÷ìng trình có 
nghi»m nguyên a vîi n  0 nào ó thì a2 + n = 0 ) n = −a2  0 do ó 
n = 0 và nghi»m là x = a = 0. Lý luªn t÷ìng tü èi vîi tr÷íng hñp n = 0. 
Nh÷ vªy n¸u mët trong hai sè m, n b¬ng 0 thì (1) có nghi»m nguyên suy 
ra sè còn l¤i cung b¬ng 0 và nghi»m là x = 0. 
www.vnmath.com
2.6. áp án tuyºn sinh lîp 10 nam 1992 (cho måi thí sinh) 51 
Ta xét tr÷íng hñp m, n  1. Khi ó gi£ sû a, b là các nghi»m cõa (1), 
theo ành lý Viét ta có ( 
a + b = mn 
ab = m + n 
(2) 
Do m, n  1 nên tø ây suy ra a, b  0 ) a, b  1 ) (a−1)(b−1)  0 hay 
ab − a − b + 1  0 ) m + n − mn+ 1  0 ) mn − m − n + 1 6 2 
) (m − 1)(n − 1) 6 2 (3) 
1. Vîi m = 1 thì (1) có d¤ng 
x2 − nx + 1+n = 0 (2) 
x2 + 1 = n(x − 1) ) n = 
x2 + 1 
x − 1 
) n = x+ 1+ 
2 
x − 1 
N¸u ph÷ìng trình có nghi»m nguyên x thì x − 1 là ÷îc cõa 2, do ó 
x − 1 2 {1,−1, 2,−2}. 
Vîi x−1 = 1 tùc x = 2 thì n = 5, ph÷ìng trình (2) có d¤ng x2−5x+ 
6 = 0 (có hai nghi»m x = 2, x = 3). 
Vîi x −1 = −1 tùc x = 0 thì n = −1 (lo¤i). 
Vîi x − 1 = 2 tùc x = 3 thì n = 5, ph÷ìng trình x = 2, x = 3. 
Vîi x −1 = −2 tùc x = −1 thì n = −1 (lo¤i). 
Vªy vîi m = 1 thì ph÷ìng trình (1) có nghi»m nguyên khi n = 5. 
Lý luªn t÷ìng tü ta có vîi n = 1 thì ph÷ìng trình (1) có nghi»m 
nguyên khi m = 5. Ta xét tr÷íng hñp m, n  2. 
2. Vîi m = 2 thì tø (3) ta có n = 2 ho°c n = 3. 
Khi n = 2 thì (1) có d¤ng x2 − 4x + 4 = 0 có nghi»m nguyên x = 2. 
Khi n = 3 thì (1) có d¤ng x2 −6x+5 = 0 có nghi»m nguyên là x = 1 
và x = 5. 
Nh÷ vªy vîi m = 2 thì (1) có nghi»m nguyên khi n = 2 ho°c n = 3. 
Lý luªn t÷ìng tü ta có vîi n = 2 thì (1) có nghi»m nguyên khi m = 2 
ho°c m = 3. 
3. Vîi m, n  3 ¯ng thùc (3) không tho£ mãn. 
Tóm l¤i: Ph÷ìng trình (1) có nghi»m nguyên (vîi m, n không âm) khi 
(m, n) là mët trong các c°p sè sau: (0, 0); (1, 5); (5, 1); (2, 2); (2, 3); (3, 2). 
www.vnmath.com
52 Ch÷ìng 2. áp án tuyºn sinh 
Bài 3. 
SMNP = S − SABM − SBCN − SCAP 
Qua A0 k´ ÷íng th¯ng song song vîi BB0 c­t CB0 t¤i I, ta có: 
AM 
MA0 = 
AB0 
B0I 
= 
AB0 
B0C 
. 
B0C 
B0I 
= 3. 
BC 
BA0 = 3.3 = 9 
AM 
AA0 = 
AM 
AM +MA0 = 
9 
10 
) 
SABM 
SABA0 
= 
AM 
AA0 = 
9 
10 
mà SABA0 = 
S 
3 
) SABM = 
9 
10 
SABA0 = 
3 
10 
S 
T÷ìng tü: Qua B0 k´ ÷íng th¯ng song song vîi CC0 c­t AC0 t¤i H, ta có 
BN 
NB0 = 
BC0 
C0K 
= 
BC0 
C0A 
. 
C0A 
C0K 
= 1. 
CA 
CB0 = 4 
BN 
BB0 = 
4 
5 
) 
SBCN 
SBCB0 
= 
BN 
BB0 = 
4 
5 
mà SBCB0 = 
S 
4 
) SBCN = 
4 
5 
SBCB0 = 
S 
5 
Hoàn toàn t÷ìng tü ta tính ÷ñc 
SCAP = 
2 
5 
S ) SMNP = S − 
 3 
10 
S + 
S 
5 
+ 
2 
5 
S 
 
= 
S 
10 
Bài 4. Trên c¤nh BC l§y iºm M sao cho BDM = ADC ) 
CDM = ADB. D¹ th§y 4BDM v 4ADC ) BM 
AC = DH 
DI (t sè hai 
÷íng cao t÷ìng ùng cõa hai tam giác çng d¤ng b¬ng t sè çng d¤ng), 
suy ra 
BM 
DH 
= 
AC 
DI 
(1) 
T÷ìng tü 
4CDM v 4ADB ) 
CM 
AB 
= 
DH 
DK 
) 
CM 
DH 
= 
AB 
DK 
(2) 
Cëng (1) và (2) ta ÷ñc 
BC 
DM 
= 
AC 
DI 
+ 
AB 
DK 
www.vnmath.com
2.6. áp án tuyºn sinh lîp 10 nam 1992 (cho måi thí sinh) 53 
Bài 5. Gi£ sû m, n nguyên d÷ìng và 
8 
: 
... 
2m + 1 
n 
... 
2n + 1 
m 
... 
) (2m + 1)(2n + 1) 
mn 
... 
) 2m + 2n + 1 
mn ) 2m + 2n + 1 = kmn (k 2 N) (1) 
Suy ra: k,m, n ·u l´. Vì 
m  1, n  1 ) (m − 1)(n − 1)  0 ) m+ n 6 mn + 1 
2m + 2n + 1 6 2mn + 3 6 5mn (2) 
Do ó tø (1) suy ra k 6 5. 
1. Vîi k = 5 ) 2m + 2n+1 = 5mn, tø (2) ta suy ra 3mn = 3 ) m = 
... 
n = 1. Rõ ràng m = n = 1 tho£ mãn 2m + 1 
...m. 
n và 2n + 1 
2. Vîi k = 3, gi£ sû m  n, ta có 3mn = 2m + 2n + 1 6 5m ) 3n 6 
5 ) n = 1 ) 3m = 2m + 3 ) m = 3. Rõ ràng n = 1,m = 3 tho£ 
mãn 2m + 1 
... 
... 
n và 2n + 1 
m. Vîi m 6 n ta ÷ñc n = 3,m = 1 tho£ 
mãn. 
3. Vîi k = 1, gi£ sû m  n, ta có mn = 2m + 2n + 1 6 5m ) n 6 5 ) 
n = 1 ho°c n = 3 ho°c n = 5. 
Vîi n = 1 ¯ng thùc mn = 2m + 2n + 1 trð thành m = 2m + 3 vô lý 
Vîi n = 3 ta có 3m = 2m + 6 + 1 ) m = 7. Rõ ràng n = 3,m = 7 
tho£ mãn i·u ki»n 2m + 1 
... 
... 
n; 2n + 1 
m. 
Vîi n = 5 ta có 5m = 2m + 10 + 1 ) m = 11 
3 (lo¤i). 
Vîi m 6 n lý luªn t÷ìng tü ta ÷ñc m = 3, n = 7 tho£ mãn i·u ki»n 
... 
2m + 1 
... 
n; 2n + 1 
m. 
Tóm l¤i: Có 5 c°p sè (m, n) tho£ mãn yêu c¦u bài toán là 
(1, 1); (1, 3); (3, 1); (3, 7); (7, 3) 
. 
www.vnmath.com
54 Ch÷ìng 2. áp án tuyºn sinh 
2.7 áp án tuyºn sinh lîp 10 nam 1992 
(cho thí sinh chuyên toán và chuyên tin) 
Bài 1. 
1. 
P = n4 + 2n3 + 2n2 + n + 7 = (n2 + n)2 + (n2 + n) + 7 
Do n nguyên nên n2+n  0. °t k = n2+n ta có P = k2+k+7  k2. 
Ta có: 
P (k + 3)2 (vì(k + 3)2 − P = 5k + 2  0) 
Do ó k2  P  (k + 3)2 ) P = (k + 1)2 ho°c p = (k + 2)2. 
Vîi P = (k + 1)2 tùc k2 + k + 7 = k2 + 2k + 1 ta thu ÷ñc k = 6 ) 
n2 + n = 6 ) n = 2 và n = −3. 
Vîi P = (k + 2)2 tùc k2 + k + 7 = k2 + 4k + 4 ta thu ÷ñc k = 1 ) 
n2 + n = 1 ) n = 2 không có nghi»m nguyên. 
Vªy P là sè chính ph÷ìng khi n = 2 và n = −3 (khi ó P = 49). 
2. 
A = 
1 
a2 + 2bc 
+ 
1 
b2 + 2ca 
+ 
1 
c2 + 2ab 
°t x = a2 + 2bc, y = b2 + 2ac, z = c2 + 2ab thì  
x, y, z  0 và 
x+y+z = (a+b+c)2 6 1, do (x+y+z)A = (x+y+z) 
1 
x+1 
y +1 
z 
 
 9 
mà x + y + z 6 1 nên A  9. 
Bài 2. Vîi n 2 N ta ký hi»u têng các chú sè cõa n là S(n). Ta có 
N = (29)1945 = (23)3.1945 = 85835  105835 
Nên N có không quá 5835 chú sè mà a = S(N) 6 5835.9 = 52515 suy ra 
a có không quá 5 chú sè, b = S(a) 6 5.9 = 45. Trong các sè tü nhiên tø 0 
¸n 45 thì sè có têng các chú sè lîn nh§t là 39 và têng các chú sè cõa nó là 
12. Suy ra S(b) 6 12. 
Ta bi¸t r¬ng vîi måi n 2 N thì S(n)  n (mod 9), do ó S(b)  b = 
S(a)  a = S(N) (mod 9). Mà N = 85835 suy ra N  −1 (mod 9) hay 
S(b) −1 (mod 9). Do S(b) 6 12 nên S(b) = 8. 
Vªy têng các chú sè cõa b là 8. 
Bài 3. Gi£ sû K n¬m ngoài o¤n BC v· phía C (tr÷íng hñp AB  AC), 
khi ó d¹ th§y ACK nhån và ACB tù suy ra cung AB không chùa C 
lîn hìn cung ACE. Trên cung AB không chùa C l§y iºm E sao cho cung 
BE b¬ng cung AC, khi ó ACBE là hình thang cân. 
www.vnmath.com
2.7. áp án tuyºn sinh lîp 10 nam 1992(cho thí sinh chuyên toán và chuyên tin)55 
Ta có AEB + EAB = EAC + EAB = 2(EAB + BAD) = 
2EAD = 90 suy ra ABE = 90 nên AE là ÷íng kính cõa ÷íng tròn 
(O) ngo¤i ti¸p 4ABC. Do ó 
AB2 + AC2 = AB2 + EB2 = AE2 = 4R2 
Chú ý: Có thº chùng minh b¬ng cách l§y iºm M trên cung AB không 
chùa C sao cho cung AM b¬ng cung AC, sau ó chùng minh BAM = 90, 
tø ó suy ra ¯ng thùc AB2 + AC2 = 4R2 
Bài 4. Gåi các ÷íng th¯ng ã cho là d1, d2, . . . ,d1992 và giao iºm cõa 
÷íng th¯ng di, dk là Aik ho°c Aki. 
a) Xét ÷íng th¯ng di b§t ky trong 1992 ÷íng th¯ng ã cho. Do không 
có 3 ÷íng th¯ng nào çng quy nên các giao iºm Akl cõa các c°p ÷íng 
th¯ng dk, dl(k6= i, l6= i) ·u n¬m ngoài di. Do sè giao iºm ó là húu h¤n 
nên có 1 giao iºm g¦n nó nh§t, gi£ sû ó là Akl. Ta chùng minh tam giác 
AklAkiAli là tam giác xanh. Thªt vªy n¸u tam giác ó bà ÷íng th¯ng dm 
nào ó trong 1989 ÷íng th¯ng còn l¤i c­t thì dm ph£i c­t mët trong hai 
o¤n AklAki ho°c AklAli, gi£ sû dm c­t o¤n AklAki t¤i Akm thì Akm g¦n di 
hìn Akl, trái vîi gi£ thi¸t Akl là iºm g¦n di nh§t. Nh÷ vªy vîi méi ÷íng 
th¯ng di luôn tçn t¤i mët tam giác xanh có c¤nh n¬m trên nó. Trên méi di 
ta chån mët c¤nh cõa mët tam giác xanh thì ta thu ÷ñc 1992 c¤nh khác 
nhau cõa các tam giác xanh. Tø ó suy ra sè tam giác xanh không ít hìn 
1992 : 3 = 664. 
b) Xét ÷íng th¯ng di trong sè 1992 ÷íng th¯ng ã cho. N¸u trong 
méi nûa m°t ph¯ng có bí là di ·u có các giao iºm cõa các c°p ÷íng 
th¯ng còn l¤i thì trong méi nûa m°t ph¯ng ta l§y giao iºm g¦n di nh§t và 
lý luªn nh÷ câu a) ta ÷ñc hai tam giác xanh n¬m v· hai phía cõa di. Hai 
tam giác ó có hai c¤nh n¬m trên di và hai c¤nh ó là khác nhau (không 
có ba ÷íng nào çng quy). 
Ta chùng minh r¬ng sè ÷íng th¯ng mà các giao iºm cõa các c°p ÷íng 
th¯ng còn l¤i n¬m v· cùng mët phía cõa nó không v÷ñt quá 2. Thªt vªy, gi£ 
sû có 3 ÷íng th¯ng nh÷ vªy, ch¯ng h¤n ó là di, dk, dl. Khi ó xét ÷íng 
th¯ng dn khác, dn c­t di, dk, dl t¤i 3 iºm phân bi»t Ani,Ank,Anl. Trong 3 
iºm ó có 1 iºm n¬m giúa hai iºm kia, gi£ sû Ank. Khi ó hai giao iºm 
Ani và Anl n¬m v· hai phía cõa dk trái vîi gi£ thi¸t. 
Vªy có ít nh§t 1990 ÷íng th¯ng mà v· hai phía cõa méi ÷íng ·u có 
các giao iºm cõa các ÷íng th¯ng còn l¤i. Theo lý luªn ð trên thì có hai 
tam giác xanh n¬m v· hai phía cõa méi ÷íng th¯ng ó và có hai c¤nh khác 
nhau n¬m trên nó. Trong hai ÷íng th¯ng còn l¤i, trên méi ÷íng th¯ng có 
ít nh§t mët c¤nh cõa mët tam giác xanh. Nh÷ vªy sè c¤nh khác nhau cõa 
các tam giác xanh không ít hìn 1990 × 2 + 2 = 3982 = 1327.3 +1. Suy ra 
sè tam giác xanh không ít hìn 1327 + 1 = 1328. 
Bài 5. 
www.vnmath.com
56 Ch÷ìng 2. áp án tuyºn sinh 
2.8 áp án tuyºn sinh lîp 10 nam 1993 
(cho måi thí sinh) 
Bài 1. 
1. Gi£i ph÷ìng trình 
x + 
s 
x + 
1 
2 
+ 
r 
x + 
1 
4 
= 2 (1) 
i·u ki»n: x  −1 
2 
(1) ) x + 
s 
r 
x + 
1 
4 
+ 
1 
2 
2 
= 2 ) x + 
r 
x + 
1 
4 
+ 
1 
2 
= 2 
) 
r 
x + 
1 
4 
+ 
1 
2 
2 
= 2 ) 
r 
x + 
1 
4 
= 
p 
2 − 
1 
2 
) x + 
1 
4 
= 2+ 
1 
4 
− 
p 
2 
) x = 2− 
p 
2(tho£ mãn i·u ki»n x  − 
1 
2 
) 
2. Gi£i h» ph÷ìng trình 
( 
x3 + 2xy2 + 12y = 0 
8y2 + x2 = 12 
) 
( 
x3 + 2xy2 + (8y2 + x2)y = 0 (1) 
8y2 + x2 = 12 (2) 
Ta có (1) t÷ìng ÷ìng vîi 
x3 + x2y + 2xy2 + 8y3 = 0 (3) 
D¹ th§y h» không có nghi»m vîi y = 0, vì n¸u y = 0 thì tø (3) suy ra 
x = 0 không tho£ mãn (2). 
Vîi y6= 0 t÷ìng ÷ìng vîi 
x 
y 
3 
+ 
x 
y 
2 
+ 2 
x 
y 
+ 8 = 0 (4) 
°t x 
y = t thì (4) có d¤ng 
t3 + t2 + 2t + 8 = 0 ) (t + 2)(t2 − t+4) = 0 ) t = −2 
Tø ó x 
y = −2 ) x = −2y. Thay vào (2) ta ÷ñc 
12y2 = 12 ) y = ±1 ) x = 2 
Vªy h» ã cho có hai nghi»m x = 2, y = −1 và x = −2, y = 1 
www.vnmath.com
2.8. áp án tuyºn sinh lîp 10 nam 1993 (cho måi thí sinh) 57 
Bài 2. Tr÷îc h¸t ta chùng minh r¬ng: Vîi a, b, c, d  0 thì 
abcd 6 
a + b + c + d 
4 
4 
(1) 
D§u = ¤t ÷ñc khi và ch¿ khi a = b = c = d. 
Ta có vîi a, b  0 thì ab 6 
 
a+b 
2 
2 
, d§u = ¤t ÷ñc khi và ch¿ khi 
a = b. Do ó vîi a, b, c, d  0 thì 
abcd 6 
a + b 
2 
2c + d 
2 
2 
= 
a + b 
2 
. 
c + d 
2 
2 
6 
ha + b + c + d 
4 
2i2 
= 
a + b + c + d 
4 
4 
) abcd 6 
a + b + c + d 
4 
4 
N¸u trong a, b, c, d có mët sè b¬ng 0 thì d§u = ¤t ÷ñc khi và ch¿ 
khi a = b = c = d = 0. 
N¸u c£ 4 sè a, b, c, d ·u d÷ìng thì d§u = ¤t ÷ñc khi và ch¿ khi 
8 
: 
a = b 
c = d 
a + b = c + d 
) a = b = c = d 
Tóm l¤i abcd 6 
 
a+b+c+d 
4 
4 
d§u = ¤t ÷ñc khi và ch¿ khi a = b = c = d. 
Xét 
A = x2y(4 − x − y) vîi x  0, y  0, x + y 6 6 (1) 
a) Giá trà lîn nh§t: 
Vîi x + y  4 thì A 6 0 
Vîi x +y  4 ta có 
A = 4. 
x 
2 
. 
x 
2 
 x 
.y(4 − x − y) 6 4 
2 + x 
2 + y + 4 − x − y 
4 
4 
= 4 
D§u = ¤t ÷ñc khi và ch¿ khi x 
2 = y = 4 − x − y ) x = 2, y = 1. 
Vªy A ¤t giá trà lîn nh§t b¬ng 4 ¤t ÷ñc khi x = 2, y = 1 (tho£ mãn 
i·u ki»n (1)). 
b) Giá trà bé nh§t. 
Vîi x + y 6 4 thì A  0. 
Vîi 4  x+ y 6 6 ta có 
− 
A 
4 
= 
x 
2 
. 
x 
2 
.y(x + y − 4) 6 
 x 
2 + x 
2 + y + x + y − 4 
4 
4 
= 
h2(x + y) − 4 
4 
i4 
6 
2.6 − 4 
4 
4 
= 16 
www.vnmath.com
58 Ch÷ìng 2. áp án tuyºn sinh 
Vì x+y 6 6 nên A  −64. D§u = ¤t ÷ñc khi và ch¿ khi x 
2 = y = x+y−4 
và x + y = 6 ) x = 4, y = 2. 
Vªy A ¤t giá trà bé nh§t b¬ng −64, ¤t ÷ñc khi x = 4, y = 2. 
Bài 3. K´ ÷íng trung trüc cõa AB c­t AC ð O1, c­t BD ð O2 thì 
O1,O2 là tâm các ÷íng tròn ngo¤i ti¸p 4ABD và 4ABC suy ra R = 
O1A, r = O2B. 
4AIO1 v 4AOB ) 
O1A 
AB 
= 
AI 
AO 
) R = O1A = 
AB.AI 
AO 
= 
a2 
AC 
) 
1 
R2 = 
AC2 
a4 
T÷ìng tü: 
1 
r2 = 
BD2 
a4 
) 
1 
R2 + 
1 
r2 = 
AC2 + BD2 
a4 = 
4AB2 
a4 = 
4 
a2 
Bài 4. Gi£ sû trên ÷íng tròn (O) chi·u i tø A ! B ! C ! A là 
ng÷ñc chi·u kim çng hç và gi£ sû quay 4ABC mët góc 90 thuªn chi·u 
kim çng hç quanh (O) ta thu ÷ñc 4A1B1C1. Khi ó A1,B1,C1 thuëc 
các cung nhä AC,AB,BC t÷ìng ùng. Do ó c¤nh A1B1 ph£i c­t các c¤nh 
AB,AC, gi£ sû l¦n l÷ñt t¤i M,N. T÷ìng tü ta có c¤nh A1C1 c­t các c¤nh 
AC,BC l¦n l÷ñt t¤i P,Q và c¤nh B1C1 c­t các c¤nh BC,BA l¦n l÷ñt t¤i 
T,K, suy ra ph¦n chung cõa hai hình tam giác ABC và A1B1C1 là löc giác 
MNPQTK. Gåi di»n tích löc giác ó là S thì 
S = SABC − SAMN − SBKT − SCPQ 
Ta có: 
SABC = 
p 
3 
4 
BC2 
= 
p 
3R2 
4 
3 
Khi quay mët góc 90 thì OA1?OA,OB1?OB,OC1?OC mà BC?OA nên 
OA1 k BC, t÷ìng tü: OB1 k CA,OC1 k AB. 
Gåi giao iºm cõa OA1 vîi AC là E. Do các cung AA1,BB1,CC1 có sè 
o b¬ng 90 nên AMN = 90 mà MAN = 60 nên suy ra AN = 2AM. 
D¹ th§y 4NEA1 cân ð E (các góc ð áy b¬ng 30). Do OE k BC còn 
AO b¬ng 2 
3 trung tuy¸n AA0 cõa 4ABC nên 
AE = 
2 
3 
AC = 
2 
p 
3 
R,OE = 
BC 
3 
= 
R 
p 
3 
) EN = EA1 = OA1 − OE = R − 
R 
p 
3 
) AN = AE − EN = 
2 
p 
3 
R − 
 
R − 
R 
p 
3 
 
= ( 
p 
3 − 1)R 
) SAMN = 
AM.MN 
2 
= 
1 
2 
AN 
2 
p 
3 
2 
AN 
= 
p 
3 
8 
AN2 = 
= 
p 
3 
p 
3(4 − 2 
8 
R2 = 
p 
3 − 3 
2 
4 
R2 
www.vnmath.com
2.8. áp án tuyºn sinh lîp 10 nam 1993 (cho måi thí sinh) 59 
T÷ìng tü ta có SBKT = SCPQ = 2 
p 
3−3 
4 R2. 
Vªy 
S = 
p 
3 
4 
9 − 3 
R2 
. 
Bài 5. 
A = 
1 
a 
+ 
1 
b 
+ 
1 
c 
+ 
1 
ab 
+ 
1 
bc 
+ 
1 
ca 
= 
ab + bc + ca + a + b + c 
abc 
(1) 
Ta chùng minh ba sè a, b, c cùng ch®n ho°c cùng l´. 
N¸u abc là l´ thì méi sè a, b, c ·u l´. 
N¸u abc là ch®n thì mët trong ba sè ph£i ch®n, ch¯ng h¤n a ch®n. Vì 
tû sè ð (1) chia h¸t cho abc nên tû sè ph£i ch®n suy ra bc + b + c ch®n hay 
(b + 1)(c + 1) − 1 ch®n ) (b + 1)(c + 1) l´. Vªy b + 1 và c+ 1 là l´ hay b, c 
ch®n. Vªy a, b, c cùng ch®n ho°c cùng l´. Vì a, b, c ôi mët khác nhau nên 
ta có thº gi£ sû r¬ng a  b  c. Khi ó, a 6 2, vì n¸u a  3 thì b  5, c  7, 
do ó 
A 6 1 
3 
+ 
1 
5 
+ 
1 
7 
+ 
1 
15 
+ 
1 
35 
+ 
1 
21 
 1 
Suy ra A không nguyên. 
a) Vîi a = 2 thì b  4, c  6 và 
A = 
1 
2 
+ 
1 
b 
+ 
1 
c 
+ 
1 
2b 
+ 
1 
2c 
+ 
1 
bc 6 1 
2 
+ 
1 
4 
+ 
1 
6 
+ 
1 
8 
+ 
1 
12 
+ 
+ 
1 
16 
+ 
1 
24 
= 
28 
24 
 2 
)A = 1. 
Vîi b  6 thì c  8 khi ó 
A 6 1 
2 
+ 
1 
6 
+ 
1 
8 
+ 
1 
12 
+ 
1 
16 
+ 
1 
48 
= 
46 
48 
 1 
)A không nguyên 
Do ó b = 4. Vªy a = 2 thì b = 4 suy ra 1 = 1 
2 + 1 
4 + 1 
c + 1 
8 + 1 
4c + 1 
2c . 
Tø ây ta thu ÷ñc c = 14. 
Vªy vîi a = 2 ta ÷ñc nghi»m là a = 2, b = 4, c = 14. 
Chú ý: Tø i·u ki»n a = 2 và A = 1 ta ÷ñc ph÷ìng trình 
1 = 
1 
2 
+ 
1 
b 
+ 
1 
c 
+ 
1 
2b 
+ 
1 
2c 
+ 
1 
bc 
) 
2b + 2c + b + c + 2 
2bc 
= 
1 
2 
)3b + 3c + 2 = bc ) (b − 3)(c − 3) = 11 ) 
( 
b − 3 = 1 
c −3 = 11 
) 
( 
b = 4 
c = 14 
www.vnmath.com
60 Ch÷ìng 2. áp án tuyºn sinh 
Do ó ta cung tìm ÷ñc nghi»m trên. 
b) a = 1, khi ó b  3, c  5 và 
A = 1+ 
2 
b 
+ 
2 
c 
+ 
1 
bc 
= 
32 
15 
 3 
mà A  1 ) A = 2 ) 2 
b + 2 
c + 1 
bc = 1 
Khi ó n¸u b  5 thì 2 
b + 2 
c + 1 
bc  1, do ó b ch¿ có thº là 3. Vîi b = 3 
ta ֖c 2 
3 + 2 
c + 1 
3c = 1 ) c = 7. 
Vªy vîi a = 1 ta ÷ñc nghi»m là a = 1, b = 3, c = 7. 
Tóm l¤i, vîi gi£ thi¸t a  b  c ta có hai nghi»m là (2, 4, 14) và (1, 3, 7). 
Thay êi vai trò a, b, c ta thu ÷ñc 12 nghi»m là các cách s­p thù tü cõa ba 
sè 2, 4, 14 và ba sè 1, 3, 7. 
2.9 áp án tuyºn sinh lîp 10 nam 1994 
(cho måi thí sinh) 
Bài 1. 
1. 
x4 − 2x3 − 6x2 + 16x − 8 = 0 (1) 
Phân tích v¸ trái thành các nhân tû ta ÷ñc (1) 
(x − 2)2(x2 + 2x − 2) = 0 , x = 
 
2 
−1 ± 
p 
3 
Vªy ph÷ìng trình có ba nghi»m: x = 1, x= −1 + 
p 
3, và x = 
−1 − 
p 
3 
2. 
x2 + 2x + 4 = 3 
p 
x3 + 4x 
p 
x2 + 4 + 2x − 3 
(x2 + 4)x = 0 
i·u ki»n: x  0 
°t: 
p 
x2 + 4 = u, 
p 
x = v thì ph÷ìng trình có d¤ng: 
u2 + 2v2 − 3uv = 0 
(u − v)(u − 2v) = 0 
u = 
 
v 
2v 
Vîi u = v ta ÷ñc 
p 
x2 + 4 = 
p 
x , x2 + 4 = x vô nghi»m. 
Vîi u = 2v ta ÷ñc 
p 
x2 + 4 = 2 
p 
x , x2 + 4 = 4x , x = 2. 
www.vnmath.com
2.9. áp án tuyºn sinh lîp 10 nam 1994 (cho måi thí sinh) 61 
Vªy ph÷ìng trình có nghi»m duy nh§t: x = 2. 
Bài 2. 
A = 
p 
zt 
p 
xy + 2 
p 
xyzt = xy + 4zt + 2(2 
A2 = xy + 4zt + 4 
p 
xt)  
p 
yz. 
6 xy + 4zt + 2yz + 2xt = 9 
Tø gi£ thi¸t suy ra A 6 3. 
D§u = ¤t ch¯ng h¤n khi x = y = z = t = 1, (tho£ mãn xy + 4zt + 
2yz + 2xt = 9). 
Vªy A ¤t giá trà lîn nh§t b¬ng 3. 
Bài 3. 
( 
xy − 3zt = 1 (1) 
xz + yt = 2 (2) 
) 
( 
x2y2 − 6xyzt+ 9z2t2 = 1 (3) 
x2z2 + 2xyzt + y2t2 = 4 (4) 
Nhân (4) vîi 3 và cëng tøng v¸ vîi (3) ta có 
x2y2 + 9z2t2 + 3x2z2 + 3y2t2 = 13 
Vì x, y, z, t là các sè nguyên mà têng các h» sè cõa v¸ trái là 16 nên n¸u 
x, y, z, t tho£ mãn h» ã cho thì ph£i có mët sè b¬ng 0. 
N¸u x = 0 ho°c y = 0 thì tø (1) ta có: −3zt = 1, vô lý. 
N¸u z = 0 h» có d¤ng ( 
xy = 1 
yt = 2 
H» này có hai nghi»m nguyên là: x = y = 1, t = 2 và x = y = −1, t = −2 
N¸u t = 0 h» có d¤ng ( 
xy = 1 
xz = 2 
H» này có hai nghi»m nguyên là: x = y = 1, z = 2 và x = y = −1, z = −2 
Vªy h» ã cho có 4 nghi»m nguyên là 
x = y = 1, z = 0, t = 2; x = y = −1, z = 0, t = −2 
x = y = 1, t = 0, z = 2; x = y = −1, t = 0, z = −2 
Bài 4. Gåi tâm ÷íng tròn ã cho là O và trung iºm cõa AB là I thì 
OI?AB và AI = BI = AD = DC. °t CD = x. 
www.vnmath.com
62 Ch÷ìng 2. áp án tuyºn sinh 
Do AH k OB (cùng k BC), nên OBI = BAH. Tø ó suy ra 
4OBI v 4BAH ) 
AI 
AH 
= 
OA 
AB 
) AH = 
AI.AB 
OA 
= 
2x2 
R 
(2.1) 
M°t khác do ÷íng tròn (O) ti¸p xúc vîi BC t¤i B nên 
CD.CA = BC2 = 4BH2 = 4(AB2 − AH2) = 16x2 − 4AH2 
) 2x2 = 16x2 − 4AH2 
) AH2 = 
7 
2 
x2 (2.2) 
Tø (2.1) và (2.2) ta suy ra 
4x4 
R2 = 
7 
2 
x2 ) x2 = 
7 
8 
R2 ) x = 
R 
2 
r 
7 
2 
Vªy 
AD = x = 
R 
2 
r 
7 
2 
(2.3) 
Tø (2.1) và (2.3) suy ra 
AH = 
7 
4 
R ) HB2 = AB2 − AH2 = 4x2 − 
49 
16 
R2 = 
28 
8 
R2 − 
49 
16 
R2 = 
7 
16 
R2 
Do HE.HA = HB2 nên ta có 
HE = 
HB2 
HA 
= 
R 
4 
) AE = AH − HE = 
3 
2 
R 
Bài 5. Do BC  AC nên BAC  ABC. Trong nûa m°t ph¯ng có 
bí là AB và chùa C k´ tia Bx sao cho ABx = BAC. Bx c­t ÷íng 
th¯ng MN t¤i P thì M n¬m giúa N và P (vì ABP  ABM). Khi 
ó ABPN là hình thang cân nên APN = BNP. Xét 4AMP ta có: 
AMP  ANM  BNM  BNP = APN  APM. 
Do ó: AM  AP = BN. 
2.10 áp án tuyºn sinh lîp 10 nam 1994 
(cho thí sinh chuyên toán và chuyên tin) 
Bài 1. 8 
: 
(x + y)(y + z) = 4xy2z (1) 
(y + z)(z + x) = 4yz2x (2) 
(z + x)(x + y) = 4zx2y (3) 
www.vnmath.com
2.10. áp án tuyºn sinh lîp 10 nam 1994(cho thí sinh chuyên toán và chuyên tin)63 
Rõ ràng x = y = z = 0 là mët nghi»m cõa h». Ng÷ñc lai, d¹ th§y n¸u 
(x, y, z) là nghi»m cõa h» mà môt trong ba sè x, y, z b¬ng 0 thì hai sè kia 
cung b¬ng 0. 
Ta tìm nghi»m tho£ mãn x6= 0, y6= 0, z6= 0. 
Ta chùng minh n¸u (x, y, z) là nghi»m mà x6= 0, y6= 0, z6= 0 thì 
x = y = z. Thªt vªy, n¸u (x, y, z) tho£ mãn (1), (2), (3) thì x + y6= 
0, y + z6= 0, z + x6= 0, do ó chia (1) cho (2) ta ÷ñc 
x + y 
z + x 
= 
y 
z 
) xz + yz = yz + xy ) x(y − z) = 0 ) y = z 
T÷ìng tü, chia (2) cho (3) ta thu ÷ñc z = x. 
Vªy vîi i·u ki»n x6= 0, y6= 0, z6= 0 h» (1),(2),(3) t÷ìng ÷ìng vîi 
( 
x = y = z 
4x2 = 4x4 
) x = y = z = ±1 
Vªy h» ã cho có ba nghi»m là 
(x, y, z) = 
2 
4 
(0, 0, 0) 
(1, 1, 1) 
(−1,−1,−1) 
Bài 2. 
12x2 + 6xy + 3y2 = 28(x + y) ) 3(4x2 + 2xy + y2) = 28(x + y) (1) 
Do 3 và 28 nguyên tè cùng nhau nên x + y 
.. .3 hay x + y = 3k vîi k 2 Z. 
Tø (1) suy ra 
3x2 + (x + y)2 = 28k ) 3x2 + 9k2 = 28k ) k 
.. .3 hay k = 3n(k 2 Z) 
)x2 + 3k2 = 28n mà k = 3n ) x2 + 27n2 = 28n ) x2 = n(28 − 27n)  0 
)n 
28 
27 
− n 
 
 0 ) 0 6 n 6 28 
27 
) n = 0 ho°c n = 1 
Vîi n = 0 ) k = 0 ) 
( 
x = 0 
x + y = 0 
) x = y = 0 
Vîi n = 1 ) k = 3 ) 
( 
x2 = 1 
x + y = 9 
) 
 
x = 1; y = 8 
x = −1; y = 10 
Vªy ph÷ìng trình ã cho có ba nghi»m nguyên là 
x = y = 0; x = 1, y = 8 và x = −1, y = 10 
www.vnmath.com
64 Ch÷ìng 2. áp án tuyºn sinh 
Bài 3. Ký hi»u A = 1.2.3 . . . n = n! (åc là n giai thøa). Ta có 
B = 1 + 2 + 3+· · · + n = 
n(n + 1) 
2 
(n  3) 
... 
Vîi n = 3 thì rõ ràng A = B = 6 suy ra A 
B. 
Ta xét n  4. Khi ó có hai kh£ nang sau: 
a) n+ 1 là sè nguyên tè. Ta chùng minh A không chia h¸t cho B. Thªt 
... 
vªy, n¸u A 
B thì 
n! = k 
n(n + 1) 
2 
) 2(n − 1)! = k(n + 1) 
i·u này vô lý vì n+1 là sè nguyên tè nên (n+1) và các sè 1, 2, . . . ,n −1 
là nguyên tè cùng nhau. 
b) n + 1 là hñp sè. Khi ó 
n + 1 = p.q (p, q 2 N, p, q  2) (1) 
Suy ra n + 1  2p hay p 6 n+1 
2 . 
Do n  3 ta có 2n  n+ 3 suy ra 2n − 2  n + 1 hay n+1 
2  n− 1 nên 
p  n− 1. T÷ìng tü q  n− 1. 
Do ó n¸u n +1 có thº vi¸t ÷ñc d÷îi d¤ng (1) vîi p6= q thì p, q là các 
sè tü nhiên nhä hìn n − 1 nên trong tích (n − 1)! = 1.2.3 . . . (n − 1) có hai 
thøa sè là p và q suy ra 
(n − 1)! 
... 
(p − q) = n + 1 ) n! 
.. .B 
.. .n(n − 1) ) A 
N¸u n+1 có d¤ng (1) vîi p = q tùc n+1 = p2(p  2) và p là hñp sè thì 
.. .B. 
n + 1 cung có d¤ng (1) vîi p6= q do ó A 
Ta xét tr÷íng hñp n + 1 = p2 vîi p sè nguyên tè. Khi ó, do n + 1  5 
nên p  3 suy ra p2  9 hay n  8. Ta chùng minh 
p2 = n + 1  
n − 1 
2 
2 
(2) 
Ta có (2) t÷ìng ÷ìng vîi 
4n + 4  n2 − 2n + 1 ) n2 − 6n − 3  0 ) (n2 − 8n) + (2n − 3)  0 
B§t ¯ng thùc này úng vì n  8, do vªy (2) úng. Tø (2) suy ra 
p  n−1 
2 . Khi ó (n − 1)! = 1.2 . . . p.(p + 1. . . (n − 1). Do (n − 1)  2p nên 
www.vnmath.com
2.10. áp án tuyºn sinh lîp 10 nam 1994(cho thí sinh chuyên toán và chuyên tin)65 
tích (p+1) . . . (n−1) có nhi·u hìn p thøa sè do ó có mët thøa sè chia h¸t 
cho p nên ta có 
(n − 1)! 
... 
p2 = n + 1 ) n! 
... 
.. .B 
n(n + 1) ) A 
K¸t hñp vîi tr÷íng hñp n = 3 ta có k¸t luªn: 
Vîi n + 1 là sè nguyên tè thì A không chia h¸t cho B. 
Vîi n + 1 là hñp sè thì A 
.. .B. 
Bài 4. Ta chùng minh r¬ng vîi x, y  1 ta có 
1 
1 + x 
+ 
1 
1 + y  1 
1 + 
p 
xy 
(1) 
Ta có (1) t÷ìng ÷ìng vîi 
(1 + y)(1 + 
p 
xy) + (1 + x)(1 + 
p 
xy) − 2(1 + x)(1 + y)  0 
)1 + 
p 
xy + y + y 
p 
xy + 1+ 
p 
xy + x + x 
p 
xy − 2 − 2x − 2y − 2xy  0 
)x 
p 
xy + y 
p 
xy − 2xy − x − y + 2 
p 
xy  0 
p 
xy( 
) 
p 
x − 
p 
y)2 − ( 
p 
x − 
p 
y)2  0 
)( 
p 
xy − 1)( 
p 
x − 
p 
y)2  0 
B§t ¯ng thùc này úng vì x, y  1 hay (1) úng (có thº th§y r¬ng d§u 
= ¤t ÷ñc khi và ch¿ khi x = y ho°c xy = 1). 
Áp döng vîi a, b, c  1 ta có 
1 
1 + a 
+ 
3 
1 + b 
= 
1 
1 + a 
+ 
1 
1 + b 
+ 
 1 
1 + 
2 
1 + b  2 
p 
ab 
+ 
1 
1 + b 
 
 4 
1 + 
p 
b 
p 
ab 
= 
4 
1 + 4 p 
ab3 
Vªy 
1 
1 + a 
+ 
3 
1 + b  4 
1 + 4 p 
ab3 
(2) 
T÷ìng tü 
1 
1 + b 
+ 
3 
1 + c  4 
1 + 4 p 
bc3 
(3) 
www.vnmath.com
66 Ch÷ìng 2. áp án tuyºn sinh 
và 
1 
1 + c 
+ 
3 
1 + a  4 
1 + 4 p 
ca3 
(4) 
Cëng (2), (3), (4) rçi chia cho 4 ta ÷ñc b§t ¯ng thùc ph£i chùng minh. 
Bài 5. 
1. Gi£ sû BAC = 20. Trên các c¤nh AB,AC l§y các iºm D,K t÷ìng 
ùng sao cho AD = KC = BC (chú ý AB = AC  BC). Ta chùng 
minh AD = DK = KC 
Phía trong 4ABC düng tam giác ·u BCI thì A, I n¬m trên trung 
trüc cõa BC suy ra AI là phân giác cõa góc BAC. Khi ó ACB = 
80 ) ACI = 20 = CAD, mà AD = BC = CI nên d¹ th§y 
ACID là hình thang cân (áy là AC và ID), tø ó ta có AC k 
ID ) DIA = IAC = IAD = 10 ) 4ADI cân ð D. Suy ra 
ID = AD = CK nên CIDK là hình bình hành suy ra DK = BC. 
Vªy AD = DK = KC = CB. 
2. Ng÷ñc l¤i, gi£ sû tçn t¤i các iºm D và K trên các c¤nh AB,AC 
t÷ìng ùng sao cho AD = DK = KC = CB. K´ o¤n th¯ng CI song 
song, cùng chi·u và b¬ng KD thì CKDI là hình bình hành và là hình 
thoi. Do CI = DK = AD và DAC = AKD = ACI nên ACID 
là hình thang cân. 
Vì CD là phân giác cõa ACI (CKDI là hình thoi) nên d¹ th§y AI 
là phân giác cõa góc DAC tø ó ta có AI là trung trüc cõa BC suy 
ra IB = IC = BC hay 4IBC ·u. °t BAC = x, ta có 
ABI = ACI = BAC = x ) IBC + ICB + 3x = 180 
hay 
120 + 3x = 180 ) BAC = x = 20 
2.11 áp án tuyºn sinh lîp 10 nam 1995 
(cho måi thí sinh) 
Bài 1. 
( 
2x2 − y2 = 1 
xy + x2 = 2 
) 
( 
2x2 − y2 = 1 (1) 
xy + x2 = 2(2x2 − y2) (2) 
www.vnmath.com
2.11. áp án tuyºn sinh lîp 10 nam 1995 (cho måi thí sinh) 67 
Tø (2) suy ra 
2y2 + xy − 3x2 = 0 ) (y − x)(2y + 3x) = 0 ) 
 
y = x 
y = −3 
2x 
Vîi y = x thay vào (1) ra ÷ñc ph÷ìng trình: x2 = 1 
Do ó h» có nghi»m là x = y = ±1 
Vîi y = −3 
2x thay vào (1) ta ÷ñc ph÷ìng trình: −x2 
2 = 1 (vô nghi»m) 
Vªy h» ã cho có hai nghi»m: x = y = 1 và x = y = −1. 
Bài 2. p 
1 − x + 
p 
4 + x = 3 (2.4) 
i·u ki»n: −4 6 x 6 1. Khi ó (2.4) t÷ìng ÷ìng vîi 
p 
1 − x + 4+x + 2 
(1 − x)(4 + x) = 9 
p 
4 − 3x − x2 = 2 
) 
)4 − 3x − x2 = 4 
)x2 + 3x = 0 ) 
 
x = 0 
x = −3 
C£ hai nghi»m ·u tho£ mãn i·u ki»n. 
Vªy ph÷ìng trình (2.4) có hai nghi»m: x = 0 và x = −3. 
Bài 3. 
a + 1 
b 
+ 
b + 1 
a 
= 
a2 + b2 + a + b 
ab 
= 
(a + b)2 + (a + b) 
ab 
− 2 
Do a+1 
b + b+1 
a 
2 N nên 
(a + b)2 + (a + b) 
ab 
2 N ) (a + b)2 + (a + b) = ka (1) 
vîi k 2 N. N¸u d  0 là ÷îc sè chung cõa a, b thì a = md, b = nd, (n,m 2 
N) ) a + b = (m + n)d, ab = mnd2. Do ó (1) có d¤ng 
(m+ n)2d2 + (m + n)d = kmnd2 
)m+ n = [kmn − (m + n)2]d = ld (l 2 N) 
p 
)a + b = ld2  d2 ) d 6 
a + b 
Bài 4. Gåi di»n tích cõa hai hình chú nhªt là S thì ta có: 
ab = cd = S ) b = 
S 
a 
, d = 
S 
c 
và 
a + b − (c + d) = a + 
S 
a 
− (c + 
S 
c 
) = a − c − ( 
S 
c 
− 
S 
a 
) = (a − c)(1 − 
S 
ac 
 0 
www.vnmath.com
68 Ch÷ìng 2. áp án tuyºn sinh 
(vì a  c  d nên a − c  0 và S 
ac  S 
dc = 1). 
Vªy a+b  c + d và chu vi hình chú nhªt thù nh§t lîn hìn chu vi hình 
chú nhªt thù hai. 
Bài 5. 
1. D¹ th§y 4ABE v 4AEC nên ta có: 
AE 
AB 
= 
AC 
AE 
) AE2 = AB.AC 
) AF = AE = 
p 
AB.AC không êi 
Vªy E,F luôn ch¤y trên ÷íng tròn cè ành tâm A, bán kính 
p 
AB.AC. 
2. Gi£ sû O /2 BC và ÷íng th©ng OI c­t cung BC không chùa F t¤i 
M. Khi ó do A,E, F,O, I cùng thuëc ÷íng tròn tâm AO nên trong 
måi tr÷íng hñp ta có 
EOM = EFI  EFE0 = 
1 
2EOE0 
Vªy OM là ÷íng phân giác cõa góc EOE0 suy ra OM?EE0 hay 
OI?EE0. Mà OI?BC nên EE0 k BC  AB. 
Tr÷íng hñp O 2 BC khi ó O  I thì FEE0 = 900 mà FE?BC 
nên EE0 k BC. 
3. Gi£ sû O6= I. Gåi giao iºm cõa BC và EF là P thì ÷íng tròn ngo¤i 
ti¸p 4ONI là ÷íng tròn ÷íng kính OP(PNO = PIO = 90). 
D¹ th§y AP.AI = AN.AO = AE2 = AB.AC ) AP = AB.AC 
AI không 
êi, mà P thuëc tia AB cè ành nên P cè ành. 
Gåi trung iºm cõa PI là K thì K cè ành và tâm O0 cõa ÷íng tròn 
÷íng kính OP (tùc ÷íng tròn ngo¤i ti¸p 4ONI) luôn n¬m trên 
÷íng th¯ng d cè ành vuông góc vîi BC ð K. 
2.12 áp án tuyºn sinh lîp 10 nam 1995 
(cho thí sinh chuyên toán và chuyên tin) 
Bài 1.  
x + 
 
p 
x2 + 3 
y + 
 
= 3 (1) 
p 
y2 + 3 
Ta có:  
x + 
p 
x2 + 3 
 
− x + 
 
= 3 (2) 
p 
x2 + 3 
 
y + 
p 
y2 + 3 
 
− y + 
 
= 3 (3) 
p 
y2 + 3 
www.vnmath.com
2.12. áp án tuyºn sinh lîp 10 nam 1995(cho thí sinh chuyên toán và chuyên tin)69 
Nhân (2) vîi (3) và chia cho (1) ta ÷ñc: 
 
− x + 
 
p 
x2 + 3 
− y + 
 
= 3 (4) 
p 
y2 + 3 
(1) ) xy + x 
p 
y2 + 3+y 
p 
x2 + 3+ 
p 
x2 + 3. 
p 
y2 + 3 = 3 (5) 
(4) ) xy − x 
p 
y2 + 3 − y 
p 
x2 + 3+ 
p 
x2 + 3. 
p 
y2 + 3 = 3 (6) 
Trø (5) cho (6) ta ÷ñc 
x 
p 
y2 + 3+y 
p 
x2 + 3 = 0 
Suy ra x, y trái d§u ho°c cùng b¬ng 0 và 
x 
p 
y2 + 3 = −y 
p 
x2 + 3 ) x2(y2 + 3) = y2(x2 + 3) 
) 3x2 = 3y2 ) |x| = |y| ) x = −y 
(vì x, y trái d§u ho°c cùng b¬ng 0) nên E = x + y = 0 
Bài 2. 
8 
: 
x + xy + y = 1 
y + yz + z = 3 
z + zx + x = 7 
) 
8 
: 
(x + 1)(y + 1) = 2 (1) 
(y + 1)(z + 1) = 4 (2) 
(z + 1)(x + 1) = 8 (3) 
8 
: 
(x + 1)(y+1) = 2 
(y + 1)(z + 1) = 4 
[(x + 1)(y + 1)(z + 1)]2 = 64 
) 
8 
: 
(x + 1)(y+1) = 2 
(y + 1)(z + 1) = 4 
(x + 1)(y + 1)(z + 1) = ±8 
Vîi (x + 1)(y + 1)(z+1) = 8 ta có: 
x + 1 = 2, y + 1 = 1, z + 1 = 4 ) x = 1, y = 0, z = 3 
Vîi (x + 1)(y + 1)(z + 1) = −8 ta có: 
x + 1 = −2, y + 1 = −1, z + 1 = −4 ) x = −3, y = −2, z = −5 
Vªy h» có hai nghi»m 
 
x = 1, y = 0, z = 3 
x = −3, y = −2, z = −5 
Chú ý: Có thº gi£i b¬ng cách nhân (1) vîi (3) rçi chia cho (2) ta ÷ñc 
(x + 1)2 = 4 ) x + 1 = ±2 tø ây d¹ dàng tìm ÷ñc các nghi»m. 
www.vnmath.com
70 Ch÷ìng 2. áp án tuyºn sinh 
Bài 3. Tø gi£ thi¸t x, y  0, x2 + y2 = 1 suy ra vîi 0 6 x, y 6 1 ta có 
( 
x3 6 x2 
y3 6 y2 
) x3 + y3 6 x2 + y2 = 1 
(D¹ th§y d§u = ¤t ÷ñc khi và ch¿ khi x = 1, y = 0 ho°c x = 0, y = 1) 
Ta có: 
1 = (x2 + y2)3 = x6 + 3x4y2 + 3x2y4 + y6 
2(x3 + y3)2 = 2x6 + 4x3y3 + 2y6 
Trø hai ¯ng thùc cuèi ta ÷ñc 
2(x3 + y3)2 −1 = x6 + y6 + 4x3y3 − 3x4y2 − 3x2y4 
= (x3 − y3)2 − 3x2y2(x − y)2 = (x − y)2[(x2 + xy + y2)2 − 3x2y2] 
= (x − y)2[x4 + y4 + 2x3y + 2xy3] 0 (vì x4 + y4 + 2x3y + 2xy3  0) 
Vªy 2(x3 + y3)2  1 hay x3 + y3  p1 
2 
. D§u = ¤t ÷ñc khi và ch¿ khi 
x = y = p1 
2 
). 
Chú ý: Có thº chùng minh b§t ¯ng thùc cuèi mët cách ng­n gån hìn 
(nh÷ng v÷ñt ngoài ch÷ìng trình!) b¬ng cách sû döng b§t ¯ng thùc Cauchy- 
Buniacovski nh÷ sau: Ta có: 
(x + y)2  2(x2 + y2)  2 ) x + y  
p 
2 
1 = (x2 + y2)2 = ( 
p 
x3 + 
p 
x 
p 
y 
p 
y3)2  (x + y)(x3 + y3) 
) x3 + y3  
1 
x + y 
 
1 
p 
2 
Bài 4. °t a = a1a2a3 thì b1b2b3 = 2a. Khi ó 
A = a1a2a3b1b2b3a1a2a3 = 106a + 2.103a + a = (103 + 1)2a = 10012.a 
= 72.112.132.a 
Vì A vi¸t ÷ñc d÷îi d¤ng A = p21 
p22 
p23 
p24 
, trong ó p1, p2, p3, p4 là bèn sè 
nguyên tè khác nhau, nên ba trong bèn sè p1, p2, p3 ph£i là 7, 11, 13 còn sè 
thù t÷ có bình ph÷ìng b¬ng a. Do ó a là bình ph÷ìng cõa mët sè nguyên 
tè khác 7, 11, 13. 
Chú ý r¬ng: 100  a = 1 
2b  1000 
2  500 suy ra a = 172 ho°c a = 192. 
Vªy có hai sè tho£ mãn i·u ki»n bài toán là 
A = 289.578.289, và A = 361.722.361 
www.vnmath.com
32 dedhkhtn hanoi1989-2005
32 dedhkhtn hanoi1989-2005
32 dedhkhtn hanoi1989-2005
32 dedhkhtn hanoi1989-2005
32 dedhkhtn hanoi1989-2005
32 dedhkhtn hanoi1989-2005
32 dedhkhtn hanoi1989-2005
32 dedhkhtn hanoi1989-2005
32 dedhkhtn hanoi1989-2005
32 dedhkhtn hanoi1989-2005
32 dedhkhtn hanoi1989-2005
32 dedhkhtn hanoi1989-2005
32 dedhkhtn hanoi1989-2005
32 dedhkhtn hanoi1989-2005
32 dedhkhtn hanoi1989-2005
32 dedhkhtn hanoi1989-2005
32 dedhkhtn hanoi1989-2005
32 dedhkhtn hanoi1989-2005
32 dedhkhtn hanoi1989-2005
32 dedhkhtn hanoi1989-2005
32 dedhkhtn hanoi1989-2005
32 dedhkhtn hanoi1989-2005
32 dedhkhtn hanoi1989-2005
32 dedhkhtn hanoi1989-2005
32 dedhkhtn hanoi1989-2005
32 dedhkhtn hanoi1989-2005
32 dedhkhtn hanoi1989-2005
32 dedhkhtn hanoi1989-2005
32 dedhkhtn hanoi1989-2005
32 dedhkhtn hanoi1989-2005
32 dedhkhtn hanoi1989-2005
32 dedhkhtn hanoi1989-2005
32 dedhkhtn hanoi1989-2005
32 dedhkhtn hanoi1989-2005
32 dedhkhtn hanoi1989-2005
32 dedhkhtn hanoi1989-2005
32 dedhkhtn hanoi1989-2005
32 dedhkhtn hanoi1989-2005

More Related Content

What's hot

đề Thi và đáp án trường chuyên ams truonghocso.com
đề Thi và đáp án trường chuyên ams   truonghocso.comđề Thi và đáp án trường chuyên ams   truonghocso.com
đề Thi và đáp án trường chuyên ams truonghocso.comThế Giới Tinh Hoa
 
Tuyen tap de thi vao lop 10
Tuyen tap de thi vao lop 10Tuyen tap de thi vao lop 10
Tuyen tap de thi vao lop 10Nguyen Van Tai
 
Tuyển chọn 50 đề thi học sinh giỏi toán 9
Tuyển chọn 50 đề thi học sinh giỏi toán 9Tuyển chọn 50 đề thi học sinh giỏi toán 9
Tuyển chọn 50 đề thi học sinh giỏi toán 9Nhập Vân Long
 
9 [htq] de thi hsg
9 [htq] de thi hsg9 [htq] de thi hsg
9 [htq] de thi hsgHồng Quang
 
TUYỂN TẬP ĐỀ THI VÀO CÁC TRƯỜNG CHUYÊN TP HÀ NỘI (Ams-NguyenHue-ChuVanAn)
TUYỂN TẬP ĐỀ THI VÀO CÁC TRƯỜNG CHUYÊN TP HÀ NỘI (Ams-NguyenHue-ChuVanAn)TUYỂN TẬP ĐỀ THI VÀO CÁC TRƯỜNG CHUYÊN TP HÀ NỘI (Ams-NguyenHue-ChuVanAn)
TUYỂN TẬP ĐỀ THI VÀO CÁC TRƯỜNG CHUYÊN TP HÀ NỘI (Ams-NguyenHue-ChuVanAn)Nhật Hiếu
 
30 đề toán luyện thi vào trương chuyên truonghocso.com
30 đề toán luyện thi vào trương chuyên   truonghocso.com30 đề toán luyện thi vào trương chuyên   truonghocso.com
30 đề toán luyện thi vào trương chuyên truonghocso.comThế Giới Tinh Hoa
 
10 de tang hsg quan huyen thay hong tri quang
10 de tang hsg quan   huyen thay hong tri quang10 de tang hsg quan   huyen thay hong tri quang
10 de tang hsg quan huyen thay hong tri quangHồng Quang
 
30 dechuyen2007
30 dechuyen200730 dechuyen2007
30 dechuyen2007Toan Isi
 
9 [htq] de thi hsg 2
9 [htq] de thi hsg 29 [htq] de thi hsg 2
9 [htq] de thi hsg 2Hồng Quang
 
9 [htq] de thi hsg 3 2 lopluyenthi
9 [htq] de thi hsg 3 2 lopluyenthi9 [htq] de thi hsg 3 2 lopluyenthi
9 [htq] de thi hsg 3 2 lopluyenthiHồng Quang
 
đề Cương ôn tập toán học kỳ i1 toán 7
đề Cương ôn tập  toán học kỳ i1 toán 7đề Cương ôn tập  toán học kỳ i1 toán 7
đề Cương ôn tập toán học kỳ i1 toán 7Dang thi thuha
 
9 03 de thi tet 2016
9 03 de thi tet 20169 03 de thi tet 2016
9 03 de thi tet 2016Hồng Quang
 
Bai tap menh de tap hop lop 10 co ban
Bai tap menh de tap hop lop 10 co banBai tap menh de tap hop lop 10 co ban
Bai tap menh de tap hop lop 10 co bandiemthic3
 
On tap toan 7 hoc ki ii va de thi hoc ki ii
On tap toan 7 hoc ki ii va de thi hoc ki iiOn tap toan 7 hoc ki ii va de thi hoc ki ii
On tap toan 7 hoc ki ii va de thi hoc ki iiTu Em
 
CHUYÊN ĐỀ BẤT ĐẲNG THỨC ÔN THI CHUYỂN CẤP LỚP 9
CHUYÊN ĐỀ BẤT ĐẲNG THỨC ÔN THI CHUYỂN CẤP LỚP 9CHUYÊN ĐỀ BẤT ĐẲNG THỨC ÔN THI CHUYỂN CẤP LỚP 9
CHUYÊN ĐỀ BẤT ĐẲNG THỨC ÔN THI CHUYỂN CẤP LỚP 9Hoàng Thái Việt
 
Đề thi vào lớp 10 THPT môn Toán tỉnh Hưng Yên năm 2015
Đề thi vào lớp 10 THPT môn Toán tỉnh Hưng Yên năm 2015Đề thi vào lớp 10 THPT môn Toán tỉnh Hưng Yên năm 2015
Đề thi vào lớp 10 THPT môn Toán tỉnh Hưng Yên năm 2015Linh Nguyễn
 
DE+DAP AN (minh hoa) Thi THPT QG mon Toan 2018-Bo GD
DE+DAP AN (minh hoa) Thi THPT QG mon Toan 2018-Bo GDDE+DAP AN (minh hoa) Thi THPT QG mon Toan 2018-Bo GD
DE+DAP AN (minh hoa) Thi THPT QG mon Toan 2018-Bo GDBẢO Hí
 

What's hot (20)

đề Thi và đáp án trường chuyên ams truonghocso.com
đề Thi và đáp án trường chuyên ams   truonghocso.comđề Thi và đáp án trường chuyên ams   truonghocso.com
đề Thi và đáp án trường chuyên ams truonghocso.com
 
Tuyen tap de thi vao lop 10
Tuyen tap de thi vao lop 10Tuyen tap de thi vao lop 10
Tuyen tap de thi vao lop 10
 
Tuyển chọn 50 đề thi học sinh giỏi toán 9
Tuyển chọn 50 đề thi học sinh giỏi toán 9Tuyển chọn 50 đề thi học sinh giỏi toán 9
Tuyển chọn 50 đề thi học sinh giỏi toán 9
 
9 [htq] de thi hsg
9 [htq] de thi hsg9 [htq] de thi hsg
9 [htq] de thi hsg
 
TUYỂN TẬP ĐỀ THI VÀO CÁC TRƯỜNG CHUYÊN TP HÀ NỘI (Ams-NguyenHue-ChuVanAn)
TUYỂN TẬP ĐỀ THI VÀO CÁC TRƯỜNG CHUYÊN TP HÀ NỘI (Ams-NguyenHue-ChuVanAn)TUYỂN TẬP ĐỀ THI VÀO CÁC TRƯỜNG CHUYÊN TP HÀ NỘI (Ams-NguyenHue-ChuVanAn)
TUYỂN TẬP ĐỀ THI VÀO CÁC TRƯỜNG CHUYÊN TP HÀ NỘI (Ams-NguyenHue-ChuVanAn)
 
30 đề toán luyện thi vào trương chuyên truonghocso.com
30 đề toán luyện thi vào trương chuyên   truonghocso.com30 đề toán luyện thi vào trương chuyên   truonghocso.com
30 đề toán luyện thi vào trương chuyên truonghocso.com
 
10 de tang hsg quan huyen thay hong tri quang
10 de tang hsg quan   huyen thay hong tri quang10 de tang hsg quan   huyen thay hong tri quang
10 de tang hsg quan huyen thay hong tri quang
 
30 dechuyen2007
30 dechuyen200730 dechuyen2007
30 dechuyen2007
 
9 [htq] de thi hsg 2
9 [htq] de thi hsg 29 [htq] de thi hsg 2
9 [htq] de thi hsg 2
 
9 [htq] de thi hsg 3 2 lopluyenthi
9 [htq] de thi hsg 3 2 lopluyenthi9 [htq] de thi hsg 3 2 lopluyenthi
9 [htq] de thi hsg 3 2 lopluyenthi
 
K10+11+12
K10+11+12K10+11+12
K10+11+12
 
đề Cương ôn tập toán học kỳ i1 toán 7
đề Cương ôn tập  toán học kỳ i1 toán 7đề Cương ôn tập  toán học kỳ i1 toán 7
đề Cương ôn tập toán học kỳ i1 toán 7
 
9 03 de thi tet 2016
9 03 de thi tet 20169 03 de thi tet 2016
9 03 de thi tet 2016
 
Bai tap menh de tap hop lop 10 co ban
Bai tap menh de tap hop lop 10 co banBai tap menh de tap hop lop 10 co ban
Bai tap menh de tap hop lop 10 co ban
 
On tap toan 7 hoc ki ii va de thi hoc ki ii
On tap toan 7 hoc ki ii va de thi hoc ki iiOn tap toan 7 hoc ki ii va de thi hoc ki ii
On tap toan 7 hoc ki ii va de thi hoc ki ii
 
CHUYÊN ĐỀ BẤT ĐẲNG THỨC ÔN THI CHUYỂN CẤP LỚP 9
CHUYÊN ĐỀ BẤT ĐẲNG THỨC ÔN THI CHUYỂN CẤP LỚP 9CHUYÊN ĐỀ BẤT ĐẲNG THỨC ÔN THI CHUYỂN CẤP LỚP 9
CHUYÊN ĐỀ BẤT ĐẲNG THỨC ÔN THI CHUYỂN CẤP LỚP 9
 
đề thi vào lớp 10
đề thi vào lớp 10đề thi vào lớp 10
đề thi vào lớp 10
 
Toan 1
Toan 1Toan 1
Toan 1
 
Đề thi vào lớp 10 THPT môn Toán tỉnh Hưng Yên năm 2015
Đề thi vào lớp 10 THPT môn Toán tỉnh Hưng Yên năm 2015Đề thi vào lớp 10 THPT môn Toán tỉnh Hưng Yên năm 2015
Đề thi vào lớp 10 THPT môn Toán tỉnh Hưng Yên năm 2015
 
DE+DAP AN (minh hoa) Thi THPT QG mon Toan 2018-Bo GD
DE+DAP AN (minh hoa) Thi THPT QG mon Toan 2018-Bo GDDE+DAP AN (minh hoa) Thi THPT QG mon Toan 2018-Bo GD
DE+DAP AN (minh hoa) Thi THPT QG mon Toan 2018-Bo GD
 

Viewers also liked

Sáng kiến kinh nghiệm : phương pháp chứng minh bài toán hình học thông qua cá...
Sáng kiến kinh nghiệm : phương pháp chứng minh bài toán hình học thông qua cá...Sáng kiến kinh nghiệm : phương pháp chứng minh bài toán hình học thông qua cá...
Sáng kiến kinh nghiệm : phương pháp chứng minh bài toán hình học thông qua cá...Học Tập Long An
 
Day con-lam-giau-tap-13
Day con-lam-giau-tap-13Day con-lam-giau-tap-13
Day con-lam-giau-tap-13Hung Duong
 
De2010&2011 chuyen
De2010&2011 chuyenDe2010&2011 chuyen
De2010&2011 chuyenToan Isi
 
De chuyen2009
De chuyen2009De chuyen2009
De chuyen2009Toan Isi
 
Cau trucvao10hanoi
Cau trucvao10hanoiCau trucvao10hanoi
Cau trucvao10hanoiToan Isi
 
150 dechuyen2008&2009
150 dechuyen2008&2009150 dechuyen2008&2009
150 dechuyen2008&2009Toan Isi
 
De dhkhtn tphcm1993-2007
De dhkhtn tphcm1993-2007De dhkhtn tphcm1993-2007
De dhkhtn tphcm1993-2007Toan Isi
 
77 de thi lop10 truong chuyen 2013
77 de thi lop10 truong chuyen 201377 de thi lop10 truong chuyen 2013
77 de thi lop10 truong chuyen 2013Toan Isi
 

Viewers also liked (8)

Sáng kiến kinh nghiệm : phương pháp chứng minh bài toán hình học thông qua cá...
Sáng kiến kinh nghiệm : phương pháp chứng minh bài toán hình học thông qua cá...Sáng kiến kinh nghiệm : phương pháp chứng minh bài toán hình học thông qua cá...
Sáng kiến kinh nghiệm : phương pháp chứng minh bài toán hình học thông qua cá...
 
Day con-lam-giau-tap-13
Day con-lam-giau-tap-13Day con-lam-giau-tap-13
Day con-lam-giau-tap-13
 
De2010&2011 chuyen
De2010&2011 chuyenDe2010&2011 chuyen
De2010&2011 chuyen
 
De chuyen2009
De chuyen2009De chuyen2009
De chuyen2009
 
Cau trucvao10hanoi
Cau trucvao10hanoiCau trucvao10hanoi
Cau trucvao10hanoi
 
150 dechuyen2008&2009
150 dechuyen2008&2009150 dechuyen2008&2009
150 dechuyen2008&2009
 
De dhkhtn tphcm1993-2007
De dhkhtn tphcm1993-2007De dhkhtn tphcm1993-2007
De dhkhtn tphcm1993-2007
 
77 de thi lop10 truong chuyen 2013
77 de thi lop10 truong chuyen 201377 de thi lop10 truong chuyen 2013
77 de thi lop10 truong chuyen 2013
 

Similar to 32 dedhkhtn hanoi1989-2005

32 de thi lop 10 dh khtn ha noi 1989 2005
32 de thi lop 10 dh khtn ha noi 1989 200532 de thi lop 10 dh khtn ha noi 1989 2005
32 de thi lop 10 dh khtn ha noi 1989 2005vutoanpvd
 
32 de thi lop 10 dh khtn ha noi 1989 2005
32 de thi lop 10 dh khtn ha noi 1989 200532 de thi lop 10 dh khtn ha noi 1989 2005
32 de thi lop 10 dh khtn ha noi 1989 2005Tam Vu Minh
 
De thi vao truong le hong phong (hcm) truonghocso.com
De thi vao truong le hong phong (hcm)   truonghocso.comDe thi vao truong le hong phong (hcm)   truonghocso.com
De thi vao truong le hong phong (hcm) truonghocso.comThế Giới Tinh Hoa
 
De cuong on tap toan 7 ca nam
De cuong on tap toan 7 ca namDe cuong on tap toan 7 ca nam
De cuong on tap toan 7 ca namcnguynthanh3
 
Xuctu.com de thi_tuyen_sinh_10_lhp_tdn_chuyen_dhsp(hcm)
Xuctu.com de thi_tuyen_sinh_10_lhp_tdn_chuyen_dhsp(hcm)Xuctu.com de thi_tuyen_sinh_10_lhp_tdn_chuyen_dhsp(hcm)
Xuctu.com de thi_tuyen_sinh_10_lhp_tdn_chuyen_dhsp(hcm)Minh Đức
 
Tuyển tập đề thi học sinh giỏi lớp 8
Tuyển tập đề thi học sinh giỏi lớp 8Tuyển tập đề thi học sinh giỏi lớp 8
Tuyển tập đề thi học sinh giỏi lớp 8Nhật Hiếu
 
De tsl10 toan amsterdam chuyen 13-14
De tsl10 toan amsterdam chuyen  13-14De tsl10 toan amsterdam chuyen  13-14
De tsl10 toan amsterdam chuyen 13-14Toan Isi
 
Mot so dang toan co ban lop 7 ki i
Mot so dang toan co ban lop 7 ki iMot so dang toan co ban lop 7 ki i
Mot so dang toan co ban lop 7 ki ilop1409ktmt
 
Chuyên đề 1 vector
Chuyên đề 1 vectorChuyên đề 1 vector
Chuyên đề 1 vectorphamchidac
 
Chuyên đề 1 vector
Chuyên đề 1 vectorChuyên đề 1 vector
Chuyên đề 1 vectorphamchidac
 
De cuong on tap hoc ki 2 toan 7201213
De cuong on tap hoc ki 2 toan 7201213De cuong on tap hoc ki 2 toan 7201213
De cuong on tap hoc ki 2 toan 7201213Khoa Tuấn
 
De thi toan_10_hanoi_2012-2013
De thi toan_10_hanoi_2012-2013De thi toan_10_hanoi_2012-2013
De thi toan_10_hanoi_2012-2013pdgnguyen
 

Similar to 32 dedhkhtn hanoi1989-2005 (20)

32 de thi lop 10 dh khtn ha noi 1989 2005
32 de thi lop 10 dh khtn ha noi 1989 200532 de thi lop 10 dh khtn ha noi 1989 2005
32 de thi lop 10 dh khtn ha noi 1989 2005
 
32 de thi lop 10 dh khtn ha noi 1989 2005
32 de thi lop 10 dh khtn ha noi 1989 200532 de thi lop 10 dh khtn ha noi 1989 2005
32 de thi lop 10 dh khtn ha noi 1989 2005
 
C2-14
C2-14C2-14
C2-14
 
De thi vao truong le hong phong (hcm) truonghocso.com
De thi vao truong le hong phong (hcm)   truonghocso.comDe thi vao truong le hong phong (hcm)   truonghocso.com
De thi vao truong le hong phong (hcm) truonghocso.com
 
Dai so to hop
Dai so to hopDai so to hop
Dai so to hop
 
40 de-thi-toan-vao-10-chon-loc
40 de-thi-toan-vao-10-chon-loc40 de-thi-toan-vao-10-chon-loc
40 de-thi-toan-vao-10-chon-loc
 
De cuong on tap toan 7 ca nam
De cuong on tap toan 7 ca namDe cuong on tap toan 7 ca nam
De cuong on tap toan 7 ca nam
 
1356763168
13567631681356763168
1356763168
 
Kimc 2014-senior-14-16
Kimc 2014-senior-14-16Kimc 2014-senior-14-16
Kimc 2014-senior-14-16
 
Xuctu.com de thi_tuyen_sinh_10_lhp_tdn_chuyen_dhsp(hcm)
Xuctu.com de thi_tuyen_sinh_10_lhp_tdn_chuyen_dhsp(hcm)Xuctu.com de thi_tuyen_sinh_10_lhp_tdn_chuyen_dhsp(hcm)
Xuctu.com de thi_tuyen_sinh_10_lhp_tdn_chuyen_dhsp(hcm)
 
Tuyển tập đề thi học sinh giỏi lớp 8
Tuyển tập đề thi học sinh giỏi lớp 8Tuyển tập đề thi học sinh giỏi lớp 8
Tuyển tập đề thi học sinh giỏi lớp 8
 
De tsl10 toan amsterdam chuyen 13-14
De tsl10 toan amsterdam chuyen  13-14De tsl10 toan amsterdam chuyen  13-14
De tsl10 toan amsterdam chuyen 13-14
 
Mot so dang toan co ban lop 7 ki i
Mot so dang toan co ban lop 7 ki iMot so dang toan co ban lop 7 ki i
Mot so dang toan co ban lop 7 ki i
 
Chuyên đề 1 vector
Chuyên đề 1 vectorChuyên đề 1 vector
Chuyên đề 1 vector
 
Chuyên đề 1 vector
Chuyên đề 1 vectorChuyên đề 1 vector
Chuyên đề 1 vector
 
Chuyen de-vecto
Chuyen de-vectoChuyen de-vecto
Chuyen de-vecto
 
Kimc 2014-senior-14-16
Kimc 2014-senior-14-16Kimc 2014-senior-14-16
Kimc 2014-senior-14-16
 
De cuong on tap hoc ki 2 toan 7201213
De cuong on tap hoc ki 2 toan 7201213De cuong on tap hoc ki 2 toan 7201213
De cuong on tap hoc ki 2 toan 7201213
 
De thi toan_10_hanoi_2012-2013
De thi toan_10_hanoi_2012-2013De thi toan_10_hanoi_2012-2013
De thi toan_10_hanoi_2012-2013
 
bai tap co loi giai xac suat thong ke
bai tap co loi giai xac suat thong kebai tap co loi giai xac suat thong ke
bai tap co loi giai xac suat thong ke
 

More from Toan Isi

77 dechuyen2013
77 dechuyen201377 dechuyen2013
77 dechuyen2013Toan Isi
 
Tuyển tập 77 đề chuyên 2014
Tuyển tập 77 đề chuyên 2014Tuyển tập 77 đề chuyên 2014
Tuyển tập 77 đề chuyên 2014Toan Isi
 
De tsl10 toan vinh phuc chuyen 13-14_giai_
De tsl10 toan vinh phuc chuyen  13-14_giai_De tsl10 toan vinh phuc chuyen  13-14_giai_
De tsl10 toan vinh phuc chuyen 13-14_giai_Toan Isi
 
De tsl10 toan thai nguyen chuyen 13-14_giai_
De tsl10 toan thai nguyen chuyen  13-14_giai_De tsl10 toan thai nguyen chuyen  13-14_giai_
De tsl10 toan thai nguyen chuyen 13-14_giai_Toan Isi
 
De tsl10 toan quang ninh chuyen 13-14_giai_
De tsl10 toan quang ninh chuyen  13-14_giai_De tsl10 toan quang ninh chuyen  13-14_giai_
De tsl10 toan quang ninh chuyen 13-14_giai_Toan Isi
 
De tsl10 toan quang nam(chuyen) 13 14
De tsl10 toan quang nam(chuyen) 13 14De tsl10 toan quang nam(chuyen) 13 14
De tsl10 toan quang nam(chuyen) 13 14Toan Isi
 
De tsl10 toan phu tho chuyen 13-14
De tsl10 toan phu tho chuyen  13-14De tsl10 toan phu tho chuyen  13-14
De tsl10 toan phu tho chuyen 13-14Toan Isi
 
De tsl10 toan nghe an chuyen 13-14
De tsl10 toan nghe an chuyen  13-14De tsl10 toan nghe an chuyen  13-14
De tsl10 toan nghe an chuyen 13-14Toan Isi
 
De tsl10 toan lang son chuyen 13-14_giai_
De tsl10 toan lang son chuyen  13-14_giai_De tsl10 toan lang son chuyen  13-14_giai_
De tsl10 toan lang son chuyen 13-14_giai_Toan Isi
 
De tsl10 toan hung yen chuyen toan 13-14_giai_
De tsl10 toan hung yen  chuyen toan  13-14_giai_De tsl10 toan hung yen  chuyen toan  13-14_giai_
De tsl10 toan hung yen chuyen toan 13-14_giai_Toan Isi
 

More from Toan Isi (20)

10.2014
10.201410.2014
10.2014
 
9.2014
9.20149.2014
9.2014
 
8.2014
8.20148.2014
8.2014
 
7.2014
7.20147.2014
7.2014
 
6.2014
6.20146.2014
6.2014
 
5.2014
5.20145.2014
5.2014
 
4.2014
4.20144.2014
4.2014
 
3.2014
3.20143.2014
3.2014
 
2.2014
2.20142.2014
2.2014
 
1.2014
1.20141.2014
1.2014
 
77 dechuyen2013
77 dechuyen201377 dechuyen2013
77 dechuyen2013
 
Tuyển tập 77 đề chuyên 2014
Tuyển tập 77 đề chuyên 2014Tuyển tập 77 đề chuyên 2014
Tuyển tập 77 đề chuyên 2014
 
De tsl10 toan vinh phuc chuyen 13-14_giai_
De tsl10 toan vinh phuc chuyen  13-14_giai_De tsl10 toan vinh phuc chuyen  13-14_giai_
De tsl10 toan vinh phuc chuyen 13-14_giai_
 
De tsl10 toan thai nguyen chuyen 13-14_giai_
De tsl10 toan thai nguyen chuyen  13-14_giai_De tsl10 toan thai nguyen chuyen  13-14_giai_
De tsl10 toan thai nguyen chuyen 13-14_giai_
 
De tsl10 toan quang ninh chuyen 13-14_giai_
De tsl10 toan quang ninh chuyen  13-14_giai_De tsl10 toan quang ninh chuyen  13-14_giai_
De tsl10 toan quang ninh chuyen 13-14_giai_
 
De tsl10 toan quang nam(chuyen) 13 14
De tsl10 toan quang nam(chuyen) 13 14De tsl10 toan quang nam(chuyen) 13 14
De tsl10 toan quang nam(chuyen) 13 14
 
De tsl10 toan phu tho chuyen 13-14
De tsl10 toan phu tho chuyen  13-14De tsl10 toan phu tho chuyen  13-14
De tsl10 toan phu tho chuyen 13-14
 
De tsl10 toan nghe an chuyen 13-14
De tsl10 toan nghe an chuyen  13-14De tsl10 toan nghe an chuyen  13-14
De tsl10 toan nghe an chuyen 13-14
 
De tsl10 toan lang son chuyen 13-14_giai_
De tsl10 toan lang son chuyen  13-14_giai_De tsl10 toan lang son chuyen  13-14_giai_
De tsl10 toan lang son chuyen 13-14_giai_
 
De tsl10 toan hung yen chuyen toan 13-14_giai_
De tsl10 toan hung yen  chuyen toan  13-14_giai_De tsl10 toan hung yen  chuyen toan  13-14_giai_
De tsl10 toan hung yen chuyen toan 13-14_giai_
 

32 dedhkhtn hanoi1989-2005

  • 1. Dàch Vö To¡n Håc 32 · thi tuyºn sinh v o lîp 10 ¤i håc KHTN H  Nëi (k±m theo ¡p ¡n) Mæn To¡n WWW.VNMATH.COM
  • 2. About VnMath.Com vnMath.com Dàch vö To¡n håc info@vnmath.com ¤i sè S¡ch Gi£i t½ch H¼nh håc C¡c lo¤i kh¡c Chuy¶n · To¡n Luy»n thi ¤i håc Bçi d÷ïng HSG · thi ¡p ¡n ¤i håc Cao håc Gi¡o ¡n c¡c mæn Olympic Thi lîp 10 1 1T i li»u ÷ñc t¼m th§y tr¶n m¤ng v  khæng rã t¡c gi£.
  • 3. Ch÷ìng 1 · thi tuyºn sinh lîp 10 1.1 · thi tuyºn sinh lîp 10 nam 1989 (cho måi thí sinh) Bài 1. Cho a thùc P(x) = ax2 + bx + c. Bi¸t r¬ng vîi måi giá trà nguyên cõa x, giá trà cõa a thùc P(x) ·u là nhúng sè chính ph÷ìng (nghia là b¬ng bình ph÷ìng cõa mët sè nguyên). Chùng minh r¬ng các h» sè a, b, c ·u là nhúng sè nguyên, và b là mët sè ch®n. Bài 2. Tìm giá trà bé nh§t cõa biºu thùc a2 + ab + b2 − 3a − 3b + 1989 Giá trà bé nh§t ó ¤t ÷ñc t¤i giá trà nào cõa a và b? Bài 3. Chùng minh r¬ng trong 52 sè nguyên d÷ìng b§t ky luôn luôn có thº tìm ÷ñc 2 sè sao cho têng ho°c hi»u cõa 2 sè ó chia h¸t cho 100. Bài 4. Cho tam giác ABC. V· phía ngoài tam giác v³ các góc[BAx = [CAy = 21. H¤ BE vuông góc vîi Ax (E n¬m trên Ax), CF vuông góc vîi Ay (F n¬m trên Ay. M là trung iºm cõa BC. 1. Chùng minh r¬ng tam giác MEF là tam giác cân 2. Tính các góc cõa tam giác MEF. Bài 5. Có 9 håc sinh vøa lîp A vøa lîp B s­p thành mët hàng dåc, ùng cách ·u. Chùng minh r¬ng có ít nh§t 1 håc sinh ùng cách hai em cùng lîp vîi mình mët kho£ng cách nh÷ nhau. 5 www.vnmath.com
  • 4. 6 Ch÷ìng 1. · thi tuyºn sinh lîp 10 1.2 · thi tuyºn sinh lîp 10 nam 1989 (cho thí sinh thí sinh chuyên lý) Bài 1. Tìm t§t c£ nhúng giá trà nguyên cõa x º biºu thùc sau là sè nguyên −2x2 + x + 36 2x + 3 Bài 2. Tìm giá trà bé nh§t cõa biºu thùc a2 + ab + b2 − 3a − 3b + 3 Giá trà bé nh§t ó ¤t ÷ñc t¤i giá trà nào cõa a và b? Bài 3. 1. Chùng minh r¬ng vîi måi m nguyên d÷ìng, biºu thùc m2 + m + 1 không ph£i là sè chính ph÷ìng (nghia là không thº b¬ng bình ph÷ìng cõa sè nguyên). 2. Chùng minh r¬ng vîi måi m nguyên d÷ìng, m(m+1) không thº b¬ng tích cõa bèn sè nguyên liên ti¸p. Bài 4. Cho tam giác ABC vuông cân, góc A = 90. CM là trung tuy¸n (M n¬m trên AB). Tø A v³ ÷íng vuông góc vîi MC c­t BC ð H. Tính t sè BH HC . Bài 5. Có 6 thành phè, trong ó cù 3 thành phè b§t ky thì có ít nh§t 2 thành phè liên l¤c ÷ñc vîi nhau. Chùng minh r¬ng trong 6 thành phè nói trên tçn t¤i 3 thành phè liên l¤c ÷ñc vîi nhau. 1.3 · thi tuyºn sinh lîp 10 nam 1989 (cho thí sinh chuyên toán - tin håc) Bài 1. Phân tích biºu thùc sau thành nhân tû a4 + b4 + c4 − 2a2b2 − ab2c2 − 2c2a2 Bài 2. 1. Cho bi¸t x x2+x+1 = −2 3 . Hãy tính giá trà cõa biºu thùc x2 x4 + x2 + 1 www.vnmath.com
  • 5. 1.4. · thi tuyºn sinh lîp 10 nam 1991 (cho måi thí sinh) 7 2. Tìm giá trà lîn nh§t cõa biºu thùc x2 x4 + x2 + 1 Giá trà lîn nh§t ó ¤t ÷ñc t¤i giá trà nào cõa x Bài 3. Cho biºu thùc P(n) = an + bn + c, trong ó a, b, c là nhúng sè nguyên d÷ìng. Chùng minh r¬ng n¸u vîi måi giá trà nguyên d÷ìng cõa n, P(n) luôn chia h¸t cho m (m là sè nguyên d÷ìng cè ành), thì b2 ph£i chia h¸t cho m. Vîi ví dö sau ây hãy chùng tä r¬ng không thº suy ra b chia h¸t cho m P(n) = 3n + 2n + 3 (xét khi m = 4) Bài 4. Cho a giác lçi sáu c¤nh ABCDEF.M, I,L,K,N,H l¦n l÷ñt là trung iºm cõa các c¤nh AB,BC,CD,DE,EF,FA. Chùng minh r¬ng các trång tâm cõa hai tam giác MNL và HIK trùng nhau. Bài 5. Gi£ sû trong mët tr÷íng có n lîp ta ký hi»u am là sè håc sinh cõa lîp thù m, dk là sè lîp trong ó méi lîp có ít nh§t k håc sinh, M là sè håc sinh cõa lîp ông nh§t. Chùng minh r¬ng: 1. a1 + a2 + · · · + an = d1 + d2 + · · · + dM 2. a21 +a22 +· · ·+a2 n = d1+3d2+5d3+· · ·+(2k−1)dk +· · ·+(2M−1)dM 1.4 · thi tuyºn sinh lîp 10 nam 1991 (cho måi thí sinh) Bài 1. 1. Gi£i và bi»n luªn ph÷ìng trình. p a + x + p a − x p a + x − p a − x = p b Trong ó a, b là các sè d÷ìng ã cho. 2. Cho ph÷ìng trình x2 + ax + b+1 = 0. Trong ó a, b 2 Z và b6= −1. Chùng minh r¬ng n¸u ph÷ìng trình có hai nghi»m ·u là nhúng sè nguyên thì a2 + b2 là hñp sè. www.vnmath.com
  • 6. 8 Ch÷ìng 1. · thi tuyºn sinh lîp 10 Bài 2. Cho a, b, c là các sè ôi mët khác nhau và khác 0. Gi£i h» 8 : a3x + a2y + az = 1 b3x + b2y + bz = 1 c3x + c2y + cz = 1 Bài 3.Tìm nghi»m nguyên, d÷ìng cõa ph÷ìng trình 7x = 3.2y + 1. Bài 4. 1. Cho hình thang ABCD(AB//CD). Gåi giao iºm cõa AD và BC là E, giao iºm cõa AC và BD là F. Chùng minh r¬ng ÷íng th¯ng EF i qua giao iºm cõa hai áy AB,CD. 2. Cho tam giác ABC. M,N,P l¦n l÷ñt là các iºm trên các c¤nh BC,CA,AB. Nèi AM,BN,CP. Chùng minh r¬ng n¸u di»n tích cõa bèn tam giác g¤ch chéo b¬ng nhau thì các di»n tích cõa ba tù giác không g¤ch chéo cung b¬ng nhau. (Xem hình v³) Bài 5. Tçn t¤i hay không 1991 iºm trên m°t ph¯ng sao cho ba iºm b§t ky trong chúng là ba ¿nh cõa mët tam giác có mët góc tù? 1.5 · thi tuyºn sinh lîp 10 nam 1991 (cho thí sinh chuyên toán và chuyên tin) Bài 1. 1. Rút gån biºu thùc A = q 3 p 3 − 4 2 p 2. q 6 p 6 44 + 16 2. Phân tích biºu thùc sau thành nhân tû P = (x − y)5 + (y − z)5 + (z − x)5 www.vnmath.com
  • 7. 1.6. · thi tuyºn sinh lîp 10 nam 1992 (cho måi thí sinh) 9 Bài 2. 1. Cho các sè a, b, c,
  • 8. , tho£ mãn các i·u ki»n 8 : a + b + c = 0 +
  • 9. + = 0 a +
  • 10. b + c = 0 Hãy tính giá trà cõa biºu thùc A = a2 +
  • 11. b2 + c2 2. Cho bèn sè a, b, c, d méi sè ·u không âm và nhä hìn ho°c b¬ng 1. Chùng minh r¬ng 0 a + b + c + d − ab − bc − cd − da 2 Khi nào thì d§u ¯ng thùc x£y ra? Bài 3. Cho tr÷îc a và d là nhúng sè nguyên d÷ìng. Xét t§t c£ các sè có d¤ng a, a + d, a + 2d, . . . , a + nd, . . . Chùng minh r¬ng trong các sè ó có ít nh§t mët sè mà 4 chú sè ¦u tiên cõa nó là 1991. Bài 4. Trong mët cuëc hëi th£o khoa håc có 100 ng÷íi tham dü. Gi£ sû méi ng÷íi ·u quen bi¸t vîi ít nh§t 67 ng÷íi. Chùng minh r¬ng có thº tìm ÷ñc mët nhóm 4 ng÷íi mà b§t ky 2 ng÷íi trong nhóm ó ·u quen bi¸t nhau. Bài 5. 1. Cho hình vuông ABCD. L§y iºm M n¬m trong hình vuông sao cho MAB =MBA = 15. Chùng minh r¬ng tam giác MCD là tam giác ·u. 2. Hãy xây düng mët tªp hñp gçm 8 iºm có tính ch§t: ÷íng trung trüc cõa o¤n nèi hai iºm b§t ky luôn i qua ít nh§t hai iºm cõa tªp hñp iºm ó. 1.6 · thi tuyºn sinh lîp 10 nam 1992 (cho måi thí sinh) Bài 1. www.vnmath.com
  • 12. 10 Ch÷ìng 1. · thi tuyºn sinh lîp 10 1. Gi£i ph÷ìng trình q p 2x − 5 + x+ 2 + 3 q p 2x − 5 = 2 x − 2 − 3 p 2 2. Gi£i h» ph÷ìng trình ( xy2 − 2y + 3x2 = 0 y2 + x2y + 2x = 0 Bài 2. Tìm t§t c£ các c°p sè nguyên không âm (m, n) º ph÷ìng trình x2 − mnx + m + n = 0 có nghi»m nguyên. Bài 3. Cho tam giác ABC có di»n tích S. Trên các c¤nh AB,BC,CA l¦n l÷ñt l§y C0,A0,B0 t÷ìng ùng, sao cho AC0 = C0B, BA0 A0C = 1 2 , CB0 B0A = 1 3 Gi£ sû AA0 c­t BB0 t¤i M, BB0 c­t CC0 t¤i N, CC0 c­t AA0 t¤i P. Tính di»n tích tam giác MNP theo S. Bài 4. Cho tam giác ABC nëi ti¸p trong mët ÷íng tròn. L§y mët iºm D trên cung BC (không chùa A) cõa ÷íng tròn ó. H¤ DH vuông góc vîi BC, DI vuông góc vîi CA và DK vuông góc vîi AB. Chùng minh r¬ng BC DH = AC DI + AB DK Bài 5. Tìm t§t c£ các c°p sè nguyên d÷ìng (m, n) sao cho 2m+ 1 chia h¸t cho n và 2n + 1 chia h¸t cho m 1.7 · thi tuyºn sinh lîp 10 nam 1992 (cho thí sinh chuyên toán và chuyên tin) Bài 1. 1. Tìm t§t c£ các sè nguyên n º n4 + 2n3 + 2n2 + n + 7 là sè chính ph÷ìng. 2. Cho a, b, c 0 và a + b + c 1. Chùng minh r¬ng 1 a2 + 2bc + 1 b2 + 2ca + 1 c2 + 2ab 9 www.vnmath.com
  • 13. 1.8. · thi tuyºn sinh lîp 10 nam 1993 (cho måi thí sinh) 11 Bài 2. Cho a là têng các chú sè cõa (29)1945, b là têng các chú sè cõa sè a. Tìm têng các chú sè cõa b. Bài 3. Cho tam giác ABC. Gi£ sû ÷íng phân giác trong và ngoài cõa góc A c­t ÷íng th¯ng BC t¤i D,K t÷ìng ùng. Chùng minh r¬ng n¸u AD = AK thì AB2+AC2 = 4R2, trong ó R là bán kính ÷íng tròn ngo¤i ti¸p tam giác ABC Bài 4. Trong m°t ph¯ng k´ 1992 ÷íng th¯ng sao cho không có 2 ÷íng nào song song và không có ba ÷íng nào çng quy. Tam giác t¤o bði ba ÷íng th¯ng trong sè các ÷íng th¯ng ã cho gåi là tam giác xanh n¸u nó không bà ÷íng th¯ng nào trong sè các ÷íng th¯ng còn l¤i c­t. 1. Chùng minh r¬ng sè tam giác xanh không ít hìn 664. 2. Chùng minh k¸t luªn m¤nh hìn: Sè tam giác xanh không ít hìn 1328. Bài 5. Có 41 thành phè ÷ñc nèi vîi nhau b¬ng các ÷íng ch¿ i ÷ñc mët chi·u. Bi¸t r¬ng tø méi thành phè có úng 16 ÷íng ¸n các thành phè khác và úng 16 ÷íng tø các thành phè khác ¸n nó. Giúa hai thành phè b§t ky không có quá mët con ÷íng cõa m¤ng ÷íng nói trên. Chùng minh r¬ng tø mët thành phè b§t ky A ·u có thº i ¸n mët thành phè b§t ky B mà ch¿ i qua nhi·u nh§t hai thành phè trung gian. 1.8 · thi tuyºn sinh lîp 10 nam 1993 (cho måi thí sinh) Bài 1. 1. Gi£i ph÷ìng trình x + s x + 1 2 + r x + 1 4 = 2 2. Gi£i h» ph÷ìng trình ( x3 + 2xy2 + 12y = 0 8y2 + x2 = 12 Bài 2. Tìm giá trà lîn nh§t và bé nh§t cõa biºu thùc A = x2y(4 − x − y) khi x và y thay êi tho£ mãn i·u ki»n: x 0, y 0, x + y 6 6 www.vnmath.com
  • 14. 12 Ch÷ìng 1. · thi tuyºn sinh lîp 10 Bài 3. Cho hình thoi ABCD. Gåi R, r l¦n l÷ñt là bán kính các ÷íng tròn ngo¤i ti¸p các tam giác ABD,ABC và a là ë dài c¤nh hình thoi. Chùng minh r¬ng: 1 R2 + 1 r2 = 4 a2 Bài 4. Cho tam giác ·u ABC nëi ti¸p ÷íng tròn tâm O bán kính R. Quay 4ABC mët góc 90 quanh tâm O ta ÷ñc 4A1B1C1. Tính di»n tích ph¦n chung cõa hai hình tam giác ABC và A1B1C1 theo R. Bài 5. Tìm t§t c£ các sè nguyên d÷ìng a, b, c ôi mët khác nhau sao cho biºu thùc A = 1 a + 1 b + 1 c + 1 ab + 1 ac + 1 bc nhªn giá trà nguyên d÷ìng. 1.9 · thi tuyºn sinh lîp 10 nam 1994 (cho måi thí sinh) Bài 1. Gi£i các ph÷ìng trình sau: 1. x4 − 2x3 − 6x2 + 16x − 8 = 0 2. x2 + 2x + 4 = 3 p x3 + 4x Bài 2. Xét các sè x, y, z, t 0 tho£ mãn h» thùc xy + 4zt + 2yz + 2xt = 9 Tìm giá trà lîn nh§t cõa biºu thùc A = p zt p xy + 2 Bài 3. Tìm t§t c£ các sè nguyên x, y, z, t tho£ mãn h» ph÷ìng trình ( xy − 3zt = 1 xz + yt = 2 Bài 4. Cho tam giác cân ABC có AB = AC và H là trung iºm cõa c¤nh BC. Mët ÷íng tròn i qua A và ti¸p xúc vîi c¤nh BC t¤i B c­t AC,AH l¦n l÷ñt t¤i D và E. Bi¸t r¬ng D là trung iºm cõa AC và bán kính ÷íng tròn b¬ng R. Tính ë dài các dây cung AE,AD theo R. Bài 5. Cho tam giác ABC có BC AC. Mët ÷íng th¯ng song song vîi c¤nh AB c­t các c¤nh BC và AC l¦n l÷ñt t¤i các iºm M và N. Chùng minh r¬ng BN AM. www.vnmath.com
  • 15. 1.10. · thi tuyºn sinh lîp 10 nam 1994(cho thí sinh chuyên toán và chuyên tin)13 1.10 · thi tuyºn sinh lîp 10 nam 1994 (cho thí sinh chuyên toán và chuyên tin) Bài 1. Gi£i h» ph÷ìng trình 8 : (x + y)(y + z) = 4xy2z (y + z)(z + x) = 4yz2x (z + x)(x + y) = 4zx2y Bài 2. Tìm t§t c£ các c°p sè nguyên (x, y) tho£ mãn ph÷ìng trình 12x2 + 6xy + 3y2 = 28(x + y) Bài 3. Xác ành các giá trà nguyên d÷ìng n(n 3) sao cho sè A = 1, 2, 3 . . .n (tích cõa n sè nguyên d÷ìng ¦u tiên) chia h¸t cho sè B = 1 + 2 + 3+· · · + n. Bài 4. Cho a, b, c 1. Chùng minh r¬ng 1 1 + a + 1 1 + b + 1 1 + c 1 1 + 4 p ab3 + 1 1 + 4 p bc3 + 1 1 + 4 p ca3 Bài 5. Cho 4ABC có AB = AC. 1. Chùng minh r¬ng n¸u BAC = 20 thì luôn tìm ÷ñc các iºm D và K trên các c¤nh AB và AC sao cho AD = DK = KC = CB. 2. Ng÷ñc l¤i, chùng minh r¬ng n¸u tçn t¤i các iºm D và K trên các c¤nh AB và AC sao cho AD = DK = KC = CB thì BAC = 20. 1.11 · thi tuyºn sinh lîp 10 nam 1995 (cho måi thí sinh) Bài 1. Gi£i h» ph÷ìng trình ( 2x2 − y2 = 1 xy + x2 = 2 Bài 2. Gi£i ph÷ìng trình p 1 − x + p 4 + x = 3 www.vnmath.com
  • 16. 14 Ch÷ìng 1. · thi tuyºn sinh lîp 10 Bài 3. Gi£ sû a, b là các sè nguyên d÷ìng sao cho: a+1 b + b+1 a là mët sè nguyên. Gåi d là ÷îc sè cõa a và b. Chùng minh r¬ng: d 6 p a + b. Bài 4. Cho hai hình chú nhªt có cùng di»n tích. Hình chú nhªt thù nh§t có các kích th÷îc a và b (a b). Hình chú nhªt thù hai có các kích th÷îc c và d (c d). Chùng minh r¬ng: n¸u a c thì chu vi cõa hình chú nhªt thù nh§t lîn hìn chu vi cõa hình chú nhªt thù hai. Bài 5. Cho ba iºm cè ành A,B,C th¯ng hàng theo thù tü §y. Gåi ( ) là mët vòng tròn qua B và C. K´ tø A các ti¸p tuy¸n AE và AF ¸n vòng tròn ( ). (E và F là các ti¸p iºm). Gåi O là tâm cõa vòng tròn ( ), I là trung iºm cõa BC, N là trung iºm cõa EF. 1. Chùng minh r¬ng: E và F n¬m trên mët vòng tròn cè ành khi vòng tròn ( ) thay êi. 2. ÷íng th¯ng FI c­t vòng tròn ( ) t¤i E0. Chùng minh r¬ng EE0 song song vîi AB. 3. Chùng minh r¬ng tâm vòng tròn ngo¤i ti¸p tam giác ONI n¬m trên mët ÷íng th¯ng cè ành khi vòng tròn ( ) thay êi. 1.12 · thi tuyºn sinh lîp 10 nam 1995 (cho thí sinh chuyên toán và chuyên tin) Bài 1. Cho x + p x2 + 3 y + = 3 p y2 + 3 Hãy tính giá trà cõa biºu thùc E = x + y Bài 2. Gi£i h» ph÷ìng trình 8 : x + xy + y = 1 y + yz + z = 3 z + zx + x = 1 Bài 3. Cho x, y 0 và x2 + y2 = 1. Chùng minh r¬ng 1 p 2 6 x3 + y3 6 1 Bài 4. Tìm sè nguyên có chín chú sè A = a1a2a3b1b2b3a1a2a3, trong 24 23 22 21 ó a16= 0 và b1b2b3 = 2a1a2a3 çng thíi A có thº vi¸t ÷ñc d÷îi d¤ng A = ppppvîi p1, p2, p3, p4 là bèn sè nguyên khác nhau. www.vnmath.com
  • 17. 1.13. · thi tuyºn sinh lîp 10 nam 1996 (cho måi thí sinh) 15 Bài 5. Cho vòng tròn ( ), v³ hai dây cung AB và CD c­t nhau ð I (I n¬m trong vòng tròn). Gåi M là trung iºm cõa BD, MI kéo dài c­t AC ð N. Chùng minh r¬ng AN NC = AI2 CI2 1.13 · thi tuyºn sinh lîp 10 nam 1996 (cho måi thí sinh) Bài 1. Cho x 0, hãy tìm giá trà nhä nh§t cõa biºu thùc P = x + 1 x 6 − x6 + 1 x6 − 2 x + 1 x 3 + x3 + 1 x3 Bài 2. Gi£i h» ph÷ìng trình 8 : p1 x + q 2 − 1 y = 2 1 p y + q 2 − 1 x = 2 Bài 3. Chùng minh r¬ng vîi måi n nguyên d÷ìng ta có n3 + 5n ...6 Bài 4. Cho a, b, c 0, chùng minh r¬ng a3 b + b3 c + c3 a ab + bc + ca Bài 5. Cho hình vuông ABCD c¤nh b¬ng a. Gåi M,N,P,Q là các iºm b§t ky l¦n l÷ñt n¬m trên các c¤nh AB,BC,CD,DA. 1. Chùng minh r¬ng 2a2 6 MN2 + NP2 + PQ2 + QM2 6 4a2 2. Gi£ sû M là mët iºm cè ành cho tr÷îc trên c¤nh AB. Hãy xác ành và trí cõa các iºm N, P,Q l¦n l÷ñt trên các c¤nh BC,CD,DA sao cho MNPQ là mët hình vuông. www.vnmath.com
  • 18. 16 Ch÷ìng 1. · thi tuyºn sinh lîp 10 1.14 · thi tuyºn sinh lîp 10 nam 1996 (cho thí sinh chuyên toán và chuyên tin) Ph¦n chung cho chuyên toán và chuyên tin Bài 1. Gi£i ph÷ìng trình ( p x − 1 + 1)3 + 2 p x − 1 = 2 − x Bài 2. Gi£i h» ph÷ìng trình 8 : x − p y = 1 y − p z = 1 z − p x = 1 Bài 3. Cho x, y là nhúng sè nguyên d÷ìng thay êi tho£ mãn i·u ki»n x + y = 201 Hãy tìm giá trà lîn nh§t và nhä nh§t cõa biºu thùc: P = x(x2+y)+y(y2+x). Bài 4. Cho o¤n th¯ng BC và ÷íng th¯ng (d) song song vîi BC. Bi¸t r¬ng kho£ng cách giúa ÷íng th¯ng (d) và ÷íng th¯ng i qua BC nhä hìn BC 2 . Gi£ sû A là mët iºm thay êi trên ÷íng th¯ng (d). 1. Hãy xác ành và trí cõa iºm A º bán kính vòng tròn ngo¤i ti¸p 4ABC nhä nh§t 2. Gåi ha, hb, hc là ë dài các ÷íng cao cõa 4ABC. Hãy xác ành và trí cõa iºm A º tích ha.hb.hc là lîn nh§t. Ph¦n dành cho chuyên toán Bài 5. Cho x, y, z 0 và x + y + z 6 3 2. Chùng minh r¬ng: r x2 − 1 x2 + r y2 − 1 y2 + r z2 − 1 z2 3 2 p 17 Ph¦n dành cho chuyên tin Câu 5. Chia mët hình tròn thành 14 hình qu¤t b¬ng nhau. Trong méi hình qu¤t °t mët viên bi (xem hình v³). Gåi T là mët phép bi¸n êi: L§y hai hình qu¤t b§t ky có bi và chuyºn tø méi hình qu¤t ó mët viên bi sang hình qu¤t li·n k· nh÷ng theo hai chi·u ng÷ñc nhau (ví dö, n¸u viên bi ð mët hình qu¤t ÷ñc chuyºn theo chi·u kim çng hç thì viên bi ð hình qu¤t kia ÷ñc chuyºn theo chi·u ng÷ñc l¤i). Häi b¬ng vi»c thüc hi»n phép bi¸n êi trên, sau mët sè húu h¤n b÷îc ta có thº chuyºn ÷ñc t§t c£ các viên bi vào mët hình qu¤t ÷ñc không. N¸u có, hãy ch¿ ra quá trình bi¸n êi.N¸u không, hãy gi£i thích t¤i sao? www.vnmath.com
  • 19. 1.15. · thi tuyºn sinh lîp 10 nam 1997 (cho måi thí sinh) 17 1.15 · thi tuyºn sinh lîp 10 nam 1997 (cho måi thí sinh) Bài 1. Cho x = 3 p p p p 10 + 6 3( 3 − 1) 6 + 2 p 5 − p 5 Tính P = (x3 − 4x + 1)1997 Bài 2. Gi£i ph÷ìng trình p x + 3+ p x + 8 = 5 p x Bài 3. Gi£i h» ph÷ìng trình 8 : 2xy = x + y + 1 2yz = y + z + 7 2xz = z + x + 2 Bài 4. Tìm t§t c£ các sè tü nhiên n º 2n + 15 là sè chính ph÷ìng. Bài 5. Cho tam giác ·u ABC c¤nh l. Bên trong tam giác ta °t 2 ÷íng tròn (O,R) và (O0,R0) ti¸p xúc ngoài vîi nhau, sao cho mët trong hai ÷íng tròn ti¸p xúc vîi các c¤nh BC và BA, ÷íng tròn kia ti¸p xúc vîi các c¤nh BC và CA. 1. Chùng minh r¬ng R + R0 p 3−1 2 . 2. Các bán kính R và R0 b¬ng bao nhiêu º têng di»n tích các hình tròn (O,R) và O0,R0 nhä nh§t và tính giá trà nhä nh§t ó. 1.16 · thi tuyºn sinh lîp 10 nam 1997 (cho thí sinh chuyên toán và chuyên tin) Bài 1. Gi£i h» ph÷ìng trình ( y3 + y2x + 3x − 6y = 0 x2 + xy = 3 www.vnmath.com
  • 20. 18 Ch÷ìng 1. · thi tuyºn sinh lîp 10 Bài 2. Có tçn t¤i hay không các sè nguyên x, y tho£ mãn i·u ki»n 1992x1993 + 1993y1994 = 1995 Bài 3. Sè 1997 vi¸t ÷ñc d÷îi d¤ng têng n hñp sè, nh÷ng không vi¸t ÷ñc d÷îi d¤ng têng n + 1 hñp sè. Häi n b¬ng bao nhiêu? Bài 4. Cho các tam giác ABC ngo¤i ti¸p vòng tròn có bán kính b¬ng 1. Gåi ha, hb, hc l¦n l÷ñt là ë dài các ÷íng cao h¤ tø ¿nh A,B,C tîi các c¤nh èi di»n. Hãy tìm giá trà lîn nh§t cõa biºu thùc. M = 1 ha + 2hb + 1 hb + 2hc + 1 hc + 2ha Bài 5. Trên ÷íng tròn cho 16 iºm và dùng 3 màu: xanh, ä, vàng º tô các iºm này (méi iºm tô b¬ng mët màu. Giúa méi c°p iºm nèi b¬ng mët o¤n th¯ng ÷ñc tô b¬ng màu tím ho°c màu nâu. Chùng minh r¬ng vîi måi cách tô màu trên các iºm (ch¿ dùng 3 màu: xanh, ä, vàng) và måi cách tô màu trên các o¤n th¯ng nèi giúa các c°p iºm (ch¿ dùng hai màu: tím ho°c nâu) ta ·u tìm ÷ñc trên hình v³ mët tam giác có ¿nh là các iºm ã cho, mà các ¿nh ÷ñc tô b¬ng cùng mët màu và các c¤nh cung ÷ñc tô b¬ng cùng mët màu (di nhiên khác màu tô trên ¿nh). 1.17 · thi tuyºn sinh lîp 10 nam 1998 (cho måi thí sinh) Bài 1. 1. Gi£i ph÷ìng trình p 2 − x2 + p x2 + 8 = 4 2. Gi£i h» ph÷ìng trình ( x2 + xy + y2 = 7 x4 + x2y2 + y4 = 21 Bài 2. Các sè a, b tho£ mãn i·u ki»n: ( a3 − 3ab2 = 19 b3 − 3a2b = 98 Hãy tính giá trà cõa biºu thùc sau: P = a2 + b2. www.vnmath.com
  • 21. 1.18. · thi tuyºn sinh lîp 10 nam 1998(cho thí sinh chuyên toán và chuyên tin)19 Bài 3. Cho các sè a, b, c 2 [0, 1]. Chùng minh r¬ng a + b2 + c3 − ab − bc − ca 6 1 Bài 4. Cho ÷íng tròn () bán kính R. A và B là hai iºm cè ành trên ÷íng tròn, (AB 2R). Gi£ sû M là mët iºm thay êi trên cung lîn AB cõa ÷íng tròn. 1. K´ tø B ÷íng th¯ng vuông góc vîi AM, ÷íng th¯ng này c­t AM t¤i I và c­t ÷íng tròn () t¤i N. Gåi J là trung iºm cõa MN. Chùng minh r¬ng khi M thay êi trên ÷íng tròn thì méi iºm I, J ·u n¬m trên mët ÷íng tròn cè ành. 2. Xác ành và trí cõa iºm M º chu vi cõa 4AMB là lîn nh§t. Bài 5. 1. Tìm t§t c£ các sè nguyên d÷ìng n sao cho méi sè n + 26 và n − 11 ·u là lªp ph÷ìng cõa mët sè nguyên d÷ìng. 2. Cho các sè x, y, z thay êi tho£ mãn i·u ki»n: x2 + y2 + z2 = 1. Hãy tìm giá trà lîn nh§t cõa biºu thùc: P = xy + yz + zx + 1 2 [x2(y − z)2 + y2(z − x)2 + z2(x − y)2] 1.18 · thi tuyºn sinh lîp 10 nam 1998 (cho thí sinh chuyên toán và chuyên tin) Bài 1. 1. Gi£i h» ph÷ìng trình ( x + x2 + x3 + x4 = y + y2 + y3 + y4 x2 + y2 = 1 2. Vîi nhúng giá trà nào cõa a thì ph÷ìng trình sau ây có nghi»m p 1 − x + p 1 + x = |1 − a| + |1 + a| Bài 2. Tìm nghi»m nguyên cõa ph÷ìng trình 19x3 − 98y2 = 1998 Bài 3. www.vnmath.com
  • 22. 20 Ch÷ìng 1. · thi tuyºn sinh lîp 10 1. Cho a, b, c là các sè tho£ mãn hai i·u ki»n sau i) 0 a b ii) Ph÷ìng trình ax2 + bx + c = 0 vô nghi»m. Chùng minh r¬ng a + b + c b − a 3 2. Cho x, y, z 0. Hãy tìm giá trà nhä nh§t cõa biºu thùc P = x2 x2 + 2yz + y2 y2 + 2zx + z2 z2 + 2xy Bài 4. Cho b£ng ô vuông kích th÷îc 1998 ×2000 (b£ng gçm 1998 hàng và 2000 cët) Ký hi»u (m, n) là ô vuông n¬m ð giao cõa hàng thù m (tính tø trên xuèng d÷îi)và cët thù n (tính tø trái qua ph£i). Cho các sè nguyên p, q vîi 1 6 p 6 1993 và 1 6 q 6 1995; Tô màu các ô vuông con cõa b£ng theo quy t­c: L¦n thù nh§t tô màu nam ô: (p, q); (p + 1, q + 1); (p + 2, q + 2); (p + 3, q + 3); (p + 4, q + 4). L¦n thù hai trð i, méi l¦n tô nam ô ch÷a có màu n¬m liên ti¸p trong cùng mët hàng ho°c cùng mët cët. Häi b¬ng cách ó ta có thº tô màu h¸t t§t c£ các ô vuông con cõa b£ng hay không? Vì sao? Bài 5. Cho tam giác ·u ABC. Trong 4ABC, v³ ba vòng tròn 1, 2, 3 có bán kính b¬ng nhau, ti¸p xúc ngoài l¨n nhau và méi vòng tròn ·u ti¸p xúc vîi hai c¤nh cõa tam giác. Gåi là vòng tròn ti¸p xúc ngoài vîi c£ ba vòng tròn 1, 2, 3. Bi¸t bán kính cõa vòng tròn là r, hãy tính ë dài c¤nh cõa 4ABC. 1.19 · thi tuyºn sinh lîp 10 nam 1999 (cho måi thí sinh) Bài 1. Cho các sè a, b, c tho£ mãn i·u ki»n ( a + b + c = 0 a2 + b2 + c2 = 14 Hãy tính giá trà cõa biºu thùc: P = 1+a4 + b4 + c4 Bài 2. www.vnmath.com
  • 23. 1.19. · thi tuyºn sinh lîp 10 nam 1999 (cho måi thí sinh) 21 1. Gi£i ph÷ìng trình p x + 3 − p 7 − x = p 2x − 8 2. Gi£i h» ph÷ìng trình ( x + y + 1 x + 1 y = 9 2 xy + 1 xy = 5 2 Bài 3. Tìm t§t c£ các sè nguyên d÷ìng n sao cho: n2 + 9n − 2 chia h¸t cho n + 11. Bài 4. Cho vòng tròn () và iºm I ð trong vòng tròn. Düng qua I hai dây cung b§t ky MIN và EIF. Gåi M0,N0,E0, F0 là các trung iºm cõa IM,IN,IE, IF. 1. Chùng minh r¬ng tù giác M0E0N0F0 là tù giác nëi ti¸p. 2. Gi£ sû I thay êi, các dây cung MIN,EIF thay êi. Chùng minh r¬ng vòng tròn ngo¤i ti¸p tù giác M0E0N0F0 có bán kính không êi. 3. Gi£ sû I cè ành, các dây cung MIN,EIF thay êi nh÷ng luôn luôn vuông góc vîi nhau. Tìm và trí cõa các dây cung MIN và EIF sao cho tù giác M0E0N0F0 có di»n tích lîn nh§t. www.vnmath.com
  • 24. 22 Ch÷ìng 1. · thi tuyºn sinh lîp 10 Bài 5. Các sè d÷ìng x và y thay êi tho£ mãn i·u ki»n: x + y = 1. Hãy tìm giá trà nhä nh§t cõa biºu thùc: P = x2 + 1 y2 y2 + 1 x2 Các thí sinh chuyên Sinh không ph£i làm bài 5 1.20 · thi tuyºn sinh lîp 10 nam 1999 (cho thí sinh chuyên toán và chuyên tin) Bài 1. Gi£i ph÷ìng trình r x + 7 x + 1 + 8 = 2x2 + p 2x − 1 Bài 2. Các sè a1, a2, . . . ÷ñc xác ành bði công thùc ak = 3k2 + 3k + 1 (k2 + k)3 vîi måi k 1 Hãy tính giá trà cõa têng: 1 + a1 + a2 + · · · + a9. Bài 3. Chùng minh r¬ng tçn t¤i mët sè chia h¸t cho 1999 và têng các chú sè cõa sè ó b¬ng1999. Bài 4. Cho vòng tròn tâm O bán kính R. Gi£ sû A và B là hai iºm cè p 3. ành trên vòng tròn vîi AB = R 1. Gi£ sû M là mët iºm thay êi trên cung lîn AB cõa ÷íng tròn. Vòng tròn nëi ti¸p 4MAB ti¸p xúc vîi MA t¤i E và ti¸p xúc vîi MB t¤i F. Chùng minh r¬ng ÷íng th¯ng EF luôn ti¸p xúc vîi mët ÷íng tròn cè ành khi M thay êi. 2. Tìm tªp hñp t§t c£ các iºm P sao cho ÷íng th¯ng 4 vuông góc vîi OP t¤i P c­t o¤n th¯ng AB. Bài 5. Cho hình tròn (C) bán kính b¬ng 1. Gi£ sû A1,A2, . . . ,A8 là 8 iºm b§t ky n¬m tròn hình tròn (kº c£ biên). Chùng minh r¬ng trong các iºm ã cho luôn tçn t¤i hai iºm ma kho£ng cách giúa chúng nhä hìn 1. 1.21 · thi tuyºn sinh lîp 10 nam 2000 (cho måi thí sinh) Bài 1. www.vnmath.com
  • 25. 1.22. · thi tuyºn sinh lîp 10 nam 2000(cho thí sinh chuyên toán và chuyên tin)23 1. Tính S = 1 1.2 + 1 2.3 + · · · + 1 1999.2000 2. Gi£i h» ph÷ìng trình ( x2 + 1 y2 + x y = 3 x + 1 y + x y = 3 Bài 2. 1. Gi£i ph÷ìng trình p x − 1 + p x3 + x2 + x+ 1 = 1+ p x4 − 1 2. Tìm t§t c£ các giá trà cõa a (a là sè thüc) º ph÷ìng trình 2x2 − 4a + 11 2 x + 4a2 + 7 = 0 có ít nh§t mët nghi»m nguyên. Bài 3. Cho ÷íng tròn tâm O nëi ti¸p trong hình thang ABCD(AB//CD), ti¸p xúc vîi c¤nh AB t¤i E và vîi c¤nh CD t¤i F (nh÷ hình v³) 1. Chùng minh r¬ng BE AE = DF CF 2. Cho bi¸t AB = a,CB = b, (a b),BE = 2AE. Tính di»n tích hình thang ABCD. Bài 4. Cho x, y là hai sè thüc b§t ky khác không. Chùng minh r¬ng 4x2y2 (x2 + y2)2 + x2 y2 + y2 x2 3 D§u ¯ng thùc x£y ra khi nào? 1.22 · thi tuyºn sinh lîp 10 nam 2000 (cho thí sinh chuyên toán và chuyên tin) Bài 1. www.vnmath.com
  • 26. 24 Ch÷ìng 1. · thi tuyºn sinh lîp 10 1. Tìm t§t c£ các c°p sè nguyên (x, y) tho£ mãn ¯ng thùc y(x − 1) = x2 + 2 2. Cho c°p sè (x, y) tho£ mãn các i·u ki»n −1 6 x + y 6 1, −1 6 xy + x + y 6 1 Chùng minh r¬ng |x| 6 2, |y| 6 2 Bài 2. 1. Gi£i ph÷ìng trình 1 x + r x − 1 x = x + r 2x − 5 x 2. Cho f(x) = ax2 + bx+c có tính ch§t f(1), f(4) và f(9) là các sè húu t. Chùng minh r¬ng khi ó a, b, c là các sè húu t. Bài 3. 1. Cho tù giác lçi ABCD. Chùng minh r¬ng n¸u các góc B và D cõa tù giác là vuông ho°c tù thì AC BD 2. Cho o¤n th¯ng AC cè ành và iºm B di ëng. Hãy tìm tªp hñp t§t c£ các iºm B º tam giác ABC là tam giác không tù và góc [BAC là góc bé nh§t cõa tam giác ABC. Bài 4. Trên m°t ph¯ng cho 6 iºm sao cho không có ba iºm nào th¯ng hàng và kho£ng cách giúa các c°p iºm là các sè khác nhau. Ta nèi méi c°p iºm bði mët o¤n th¯ng. Chùng minh r¬ng trong các o¤n th¯ng thu ÷ñc có mët o¤n th¯ng là c¤nh bé nh§t cõa mët tam giác có 3 ¿nh là 3 trong 6 iºm ã cho çng thíi là c¤nh lîn nh§t cõa mët tam giác khác cung có 3 ¿nh là 3 trong 6 iºm ã cho. 1.23 · thi tuyºn sinh lîp 10 nam 2001 (cho måi thí sinh) Bài 1. Tìm các giá trà nguyên x, y tho£ mãn ¯ng thùc (y + 2)x2 + 1 = y2 Bài 2. www.vnmath.com
  • 27. 1.24. · thi tuyºn sinh lîp 10 nam 2001(cho thí sinh chuyên toán và chuyên tin)25 1. Gi£i ph÷ìng trình p x(3x + 1) − p p x2 x(x − 1) = 2 2. Gi£i h» ph÷ìng trình ( x2 + xy + 2 = 3x + y x2 + y2 = 2 Bài 3. Cho nûa vòng tròn ÷íng kính AB = 2a. Trên o¤n AB l§y iºm M. Trong nûa m°t ph¯ng bí AB chùa nûa vòng tròn, ta k´ hai tia Mx và My sao choAMx =BMx = 300. Tia Mx c­t nûa vòng tròn ð E, tia My c­t nûa vòng tròn ð F. K´ EE0, FF0 vuông góc xuèng AB. 1. Cho AM = a 2 , tính di»n tích hình thang vuông EE0F0F theo a. 2. Khi iºm M di ëng trên AB, chùng minh r¬ng ÷íng th¯ng EF luôn ti¸p xúc vîi mët vòng tròn cè ành. Bài 4. Gi£ sû x, y, z là các sè thüc khác không tho£ mãn h» ¯ng thùc: ( x 1 y + 1 z + y 1 z + 1 x + z 1 x + 1 y = −2 x3 + y3 + z3 = 1 Hãy tính giá trà cõa biºu thùc P = 1 x + 1 y + 1 z Bài 5. Vîi x, y, z là nhúng sè thüc d÷ìng, hãy tìm giá trà lîn nh§t cõa biºu thùc M = xyz (x + y)(y + z)(z + x) 1.24 · thi tuyºn sinh lîp 10 nam 2001 (cho thí sinh chuyên toán và chuyên tin) Bài 1. 1. Cho f(x) = ax2 + bx + c có tính ch§t f(x) nhªn giá trà nguyên khi x là sè nguyên. Häi các h» sè a, b, c có nh§t thi¸t ph£i là các sè nguyên hay không? T¤i sao? www.vnmath.com
  • 28. 26 Ch÷ìng 1. · thi tuyºn sinh lîp 10 2. Tìm các sè nguyên không âm x, y tho£ mãn ¯ng thùc: x2 = y2 + p y + 1 Bài 2. Gi£i ph÷ìng trình p x + 1 = x2 − 5x + 14 4 Bài 3. Cho các sè thüc a, b, x, y tho£ mãn h» 8 : ax + by = 3 ax2 + by2 = 5 ax3 + by3 = 9 ax4 + by4 = 17 Hãy tính giá trà cõa biºu thùc A = ax5 + by5 B = ax2001 + by2001 Bài 4. Cho o¤n th¯ng AB có trung iºm là O. Gåi d1, d2 là các ÷íng th¯ng vuông góc vîi AB t÷ìng ùng t¤i A và B. Mët góc vuông ¿nh O có mët c¤nh c­t d1 ð M, còn c¤nh kia c­t d2 ð N. K´ OH vuông góc xuèng MN. Vòng tròn ngo¤i ti¸p tam giác MHB c­t d1 ð iºm thù hai E khác M, MB c­t NA ð I, ÷íng th¯ng HI c­t EB ð K. Chùng minh r¬ng K n¬m trên mët vòng tròn cè ành khi góc vuông quay xung quanh ¿nh O. Bài 5. Cho 2001 çng ti·n, méi çng ti·n ÷ñc sìn mët m°t b¬ng màu ä và m°t kia b¬ng màu xanh. X¸p 2001 çng ti·n ó theo mët vòng tròn sao cho t§t c£ các çng ti·n ·u có m°t xanh ngûa lên phía trên. Cho phép méi l¦n êi m°t çng thíi 5 çng ti·n liên ti¸p c¤nh nhau. Häi vîi cách làm nh÷ th¸, sau mët sè húu h¤n l¦n ta có thº làm cho t§t c£ các çng ti·n ·u có m°t ä ngûa lên phía trên ÷ñc hay không? T¤i sao? 1.25 · thi tuyºn sinh lîp 10 nam 2002 (cho måi thí sinh) Bài 1. 1. Gi£i ph÷ìng trình: p 8 + p x + p 5 − p x = 5 www.vnmath.com
  • 29. 1.26. · thi tuyºn sinh lîp 10 nam 2002(cho thí sinh chuyên toán và chuyên tin)27 2. Gi£i h» ph÷ìng trình ( (x + 1)(y + 1) = 8 x(x+ 1)+y(y + 1) + xy = 17 Bài 2. Cho a, b, c là ë dài ba c¤nh cõa mët tam giác. Chùng minh r¬ng ph÷ìng trình x2 + (a + b + c)x + ab + bc + ca = 0 vô nghi»m. Bài 3. Tìm t§t c£ các sè nguyên n sao cho n2 + 2002 là mët sè chính ph÷ìng. Bài 4. Tìm giá trà nhä nh§t cõa biºu thùc P = 1 1 + xy + 1 1 + yz + 1 1 + zx trong ó x, y, z là các sè d÷ìng thay êi tho£ mãn i·u ki»n x2+y2+z2 6 3. Bài 5. Cho hình vuông ABCD, M là iºm thay êi trên c¤nh BC (M không trùng vîi B) và N là iºm thay êi trên c¤nh CD (N không trùng vîi D) sao cho: MAN =MAB +NAD 1. BD c­t AN và AM t÷ìng ùng t¤i P và Q. Chùng minh r¬ng nam iºm P,Q,M,C,N cùng n¬m trên mët ÷íng tròn. 2. Chùng minh r¬ng ÷íng th¯ng MN luôn ti¸p xúc vîi mët ÷íng tròn cè ành khi M và N thay êi. 3. Ký hi»u di»n tích cõa tam giác APQ là S1 là di»n tích cõa tù giác PQMN là S2. Chùng minh r¬ng t¿ sè S1 S2 không êi khi M và N thay êi. 1.26 · thi tuyºn sinh lîp 10 nam 2002 (cho thí sinh chuyên toán và chuyên tin) Bài 1. 1. Gi£i ph÷ìng trình: p x2 − 3x + 2+ p x + 3 = p x − 2+ p x2 + 2x − 3 2. Tìm nghi»m nguyên cõa ph÷ìng trình x + xy + y = 9 www.vnmath.com
  • 30. 28 Ch÷ìng 1. · thi tuyºn sinh lîp 10 Bài 2. Gi£i h» ph÷ìng trình ( x2 + y2 + xy = 1 x3 + y3 = x+ 3y Bài 3. Cho m÷íi sè nguyên d÷ìng 1, 2, . . . , 10. S­p x¸p m÷íi sè ó mët cách tuy ý thành mët hàng. Cëng méi sè vîi sè thù tü cõa nó trong hàng, ta ÷ñc m÷íi têng. Chùng minh r¬ng trong m÷íi têng ó tçn t¤i ít nh§t hai têng có chú sè tªn cùng gièng nhau. Bài 4. Tìm giá trà nhä nh§t cõa biºu thùc P = 4a b + c − a + 9b a + c − b + 16c a + b − c trong ó a, b, c là ë dài ba c¤nh cõa mët tam giác. Bài 5. ÷íng tròn (C) tâm I nëi ti¸p tam giác ABC ti¸p xúc vîi các c¤nh BC,CA,AB t÷ìng ùng t¤i các iºm A0,B0,C0. 1. Gåi các giao iºm cõa ÷íng tròn (C) vîi các o¤n IA,IB,IC l¦n l÷ñt là M,N,P. Chùng minh r¬ng các ÷íng th¯ng A0M,B0N,C0P çng quy. 2. Kéo dài o¤n AI c­t ÷íng tròn ngo¤i ti¸p tam giác ABC t¤i D (khác A). Chùng minh r¬ng IB.IC ID = 2r trong ó r là bán kính ÷íng tròn (C). 1.27 · thi tuyºn sinh lîp 10 nam 2003 (cho måi thí sinh) Bài 1. Gi£i ph÷ìng trình ( p x + 5 − p x + 2)(1 + p x2 + 7x + 10 = 3 Bài 2. Gi£i h» ph÷ìng trình ( 2x3 + 3x2y = 5 y3 + 6xy2 = 7 Bài 3. Tìm các sè nguyên x, y tho£ mãn ¯ng thùc 2y2x + x + y + 1 = x2 + 2y2 + xy Bài 4. Cho nûa ÷íng tròn (O) ÷íng kính AB = 2R (R là mët ë dài cho tr÷îc), M,N là hai iºm trên nûa ÷íng tròn (O) sao cho M thuëc cung AN và têng các kho£ng cách tø A,B ¸n ÷íng th¯ng MN b¬ng R p 3. www.vnmath.com
  • 31. 1.28. · thi tuyºn sinh lîp 10 nam 2003(cho thí sinh chuyên toán và chuyên tin)29 1. Tính ë dài o¤n MN theo R. 2. Gåi giao iºm cõa hai dây AN và BM là I, giao iºm cõa các ÷íng th¯ng AM và BN là K. Chùng minh r¬ng bèn iºm M,N, I,K cùng n¬m trên mët ÷íng tròn. Tính bán kính cõa ÷íng tròn ó theo R. 3. Tìm giá trà lîn nh§t cõa di»n tích tam giác KAB theo R khi M,N thay êi nh÷ng v¨n tho£ mãn gi£ thi¸t cõa bài toán. Bài 5. x, y, z là các sè thüc tho£ mãn i·u ki»n x + y + z + xy + yz + zx = 6 Chùng minh r¬ng: x2 + y2 + z2 3. 1.28 · thi tuyºn sinh lîp 10 nam 2003 (cho thí sinh chuyên toán và chuyên tin) Bài 1. Cho ph÷ìng trình x4 + 2mx2 + 4 = 0 Tìm giá trà cõa tham sè m º ph÷ìng trình có 4 nghi»m phân bi»t x1, x2, x3, x4 tho£ mãn x41 + x42 + x43 + x44 = 32 Bài 2. Gi£i h» ph÷ìng trình ( 2x2 + xy − y2 − 5x + y + 2 = 0 x2 + y2 + x + y −4 = 0 Bài 3. Tìm các sè nguyên x, y tho£ mãn ¯ng thùc x2 + xy + y2 = x2y2 Bài 4. Cho ÷íng tròn tâm O nëi ti¸p tam giác ABC ti¸p xúc vîi các c¤nh BC,CA,AB t÷ìng ùng t¤i các iºm D,E,F. ÷íng tròn tâm O0 bàng ti¸p trong góc [BAC cõa tam giác ABC ti¸p xúc vîi c¤nh BC và ph¦n kéo dài cõa các c¤nh AB,AC t÷ìng ùng t¤i các iºm P,M,N. 1. Chùng minh r¬ng: BP = CD. 2. Trên ÷íng th¯ngMN ta l§y các iºm I và K sao cho CK//AB,BI//AC. Chùng minh r¬ng các tù giác BICE và BKCF là các hình binh hành. www.vnmath.com
  • 32. 30 Ch÷ìng 1. · thi tuyºn sinh lîp 10 3. Gåi (S) là ÷íng tròn i qua ba iºm I,K,P. Chùng minh r¬ng (S) ti¸p xúc vîi các ÷íng th¯ng BC,BI,CK. Bài 5. Sè thüc x thay êi và tho£ mãn i·u ki»n x2+(3−x)2 5. Tìm giá trà nhä nh§t cõa biºu thùc p = x4 + (3 − x)4 + 6x2(3 − x)2 1.29 · thi tuyºn sinh lîp 10 nam 2004 (cho måi thí sinh) Bài 1. 1. Gi£i ph÷ìng trình |x + 1| + |x − 1| = 1+|x2 − 1| 2. Tìm nghi»m nguyên cõa h» ( 2y2 − x2 − xy + 2y − 2x = 7 x3 + y3 + x − y = 8 Bài 2. Cho các sè thüc d÷ìng a và b tho£ mãn a100 + b100 = a101 + b101 = a102 + b102 Hãy tính giá trà cõa biºu thùc P = a2004 + b2004 Bài 3. Cho 4ABC có AB = 3cm,BC = 4cm,CA = 5cm. ÷íng cao, ÷íng phân giác, ÷íng trung tuy¸n cõa tam giác k´ tø ¿nh B chia tam giác thành 4 ph¦n. Tính di»n tích méi ph¦n. Bài 4. Cho tù giác ABCD nëi ti¸p trong ÷íng tròn có hai ÷íng chéo AC và BD vuông góc vîi nhau t¤i H (H không trùng vîi tâm cõa ÷íng tròn). Gåi M và N l¦n l÷ñt là chân các ÷íng vuông góc h¤ tø H xuèng các ÷íng th¯ng AB và BC; P và Q l¦n l÷ñt là giao iºm cõa ÷íng th¯ng MH và NH vîi các ÷íng th¯ng CD và DA. Chùng minh r¬ng ÷íng th¯ng PQ song song vîi ÷íng th¯ng AC và bèn iºm M,N,P,Q n¬m trên cùng mët ÷íng tròn. Bài 5. Tìm giá trà nhä nh§t cõa biºu thùc Q = 1 2 x10 y2 + y10 x2 + 1 4 (x16 + y16) − (1 + x2y2)2 www.vnmath.com
  • 33. 1.30. · thi tuyºn sinh lîp 10 nam 2004(cho thí sinh chuyên toán và chuyên tin)31 1.30 · thi tuyºn sinh lîp 10 nam 2004 (cho thí sinh chuyên toán và chuyên tin) Bài 1. Gi£i ph÷ìng trình p x+ 3+ p x − 1 = 2 Bài 2. Gi£i h» ph÷ìng trình (x + y)(x2 + y2) = 15(x − y)(x2 − y2) = 3 Bài 3. Tìm giá trà nhä nh§t cõa biºu thùc P = (x3 + y3) − (x2 + y2) (x − 1)(y − 1) trong ó, x, y là nhúng sè thüc lîn hìn 1. Bài 4. Cho hình vuông ABCD và iºm M n¬m trong hình vuông. 1. Tìm t§t c£ các và trí cõa iºm M sao choMAB =MBC =MCD = MDA 2. Xét iºm M n¬m trên ÷íng chéo AC. Gåi N là chân ÷íng vuông góc h¤ tø iºmM xuèng AB và O là trung iºm cõa o¤n AM. Chùng minh r¬ng t sè OB CN có giá trà không êi khi M di chuyºn trên ÷íng chéo AC. 3. Vîi gi£ thi¸t M n¬m trên ÷íng chéo AC, xét các ÷íng tròn (S1) và (S2) có ÷íng kính t÷ìng ùng là AM và CN. Hai ti¸p tuy¸n chung cõa (S1) và (S2) ti¸p xúc vîi (S2) t¤i P và Q. Chùng minh r¬ng ÷íng th¯ng PQ ti¸p xúc vîi (S1) Bài 5. Vîi sè thüc a, ta ành nghia ph¦n nguyên cõa sè a là sè nguyên lîn nh§t không v÷ñt quá a và ký hi»u là [a]. Dãy các sè x0, x1, x2, . . . ,xn, . . . ÷ñc xác ành bði công thùc xn = hn + 1 p 2 i − h n p 2 i Häi trong 200 sè {x0, x1, . . . ,x199} có bao nhiêu sè khác 0? (Cho bi¸t 1, 41 p 2 1, 42). www.vnmath.com
  • 34. 32 Ch÷ìng 1. · thi tuyºn sinh lîp 10 1.31 · thi tuyºn sinh lîp 10 nam 2005 (cho måi thí sinh) Bài 1. Gi£i h» ph÷ìng trình ( x + y + xy = 3 x2 + y2 = 2 Bài 2. Gi£i ph÷ìng trình p x + 3 + 2 x + 4 p 3 − 2x = 11 Bài 3. Tìm nghi»m nguyên cõa ph÷ìng trình x2 + 17y2 + 34xy + 51(x + y) = 1740 Bài 4. Cho ÷íng tròn (O), (O0) n¬m ngoài nhau có tâm t÷ìng ùng là O và O0. Mët ti¸p tuy¸n chung ngoài cõa hai ÷íng tròn ti¸p xúc vîi (O) t¤i A và (O0) t¤i B. Mët ti¸p tuy¸n chung trong cõa hai ÷íng tròn c­t AB t¤i I, ti¸p xúc vîi (O) t¤i C và (O0) t¤i D. Bi¸t C n¬m giúa I và D. 1. Hai ÷íng th¯ng OC,O0B c­t nhau t¤i M. Chùng minh r¬ng OM O0M. 2. Ký hi»u (S) là ÷íng tròn i qua A,C,B và (S0) là ÷íng tròn i qua A,D,B. ÷íng th¯ng CD c­t (S) t¤i E khác C và c­t (S0) t¤i F khác D. Chùng minh r¬ng AF vuông góc vîi BE. Bài 5. Gi£ sû x, y, z là các sè d÷ìng thay êi và tho£ mãn i·u ki»n xy2z2 + x2z + y = 3z2. Hãy tìm giá trà lîn nh§t cõa biºu thùc P = z4 1 + z4(x4 + y4) 1.32 · thi tuyºn sinh lîp 10 nam 2005 (cho thí sinh chuyên toán và chuyên tin) Bài 1. Gi£i ph÷ìng trình p 2 − x + p 2 + x + p 4 − x2 = 2 Bài 2. Gi£i h» ph÷ìng trình ( x3 + y3 − xy2 = 1 4x4 + y4 = 4x + y Bài 3. Gi£ sû x, y là nhúng sè không âm tho£ mãn i·u ki»n x2+y2 = 1 www.vnmath.com
  • 35. 1.32. · thi tuyºn sinh lîp 10 nam 2005(cho thí sinh chuyên toán và chuyên tin)33 1. Chùng minh r¬ng 1 6 x + y 6 p 2 2. Tìm giá trà lîn nh§t và giá trà nhä nh§t cõa biºu thùc P = p 1 + 2x + p 1 + 2y Bài 4. Cho hinh vuông ABCD và iºm P n¬m trong tam giác ABC. 1. Gi£ sû gócBPC = 1350. Chùng minh r¬ng 2PB2 + PC2 = PA2. 2. Các ÷íng th¯ng AP và CP c­t các c¤nh BC và BA t÷ìng ùng t¤i các iºm M và N. Gåi Q là iºm èi xùng vîi B qua trung iºm cõa o¤n MN. Chùng minh r¬ng khi P thay êi trong 4ABC, ÷íng th¯ng PQ luôn i qua D. Bài 5. 1. Cho a giác ·u (H) có 14 ¿nh. Chùng minh r¬ng trong 6 ¿nh b§t ky cõa (H) luôn có 4 ¿nh là các ¿nh cõa mët hình thang 2. Có bao nhiêu phân sè tèi gi£n m n lîn hìn 1 (m, n là các sè nguyên d÷ìng) tho£ mãn m.n = 13860. www.vnmath.com
  • 36. Ch÷ìng 2 áp án tuyºn sinh 2.1 áp án tuyºn sinh lîp 10 nam 1989 (cho måi thí sinh) Bài 1. Gåi tªp hñp các sè chính ph÷ìng là . P(x) = ax2 + bx + c Ta có P(0) = c 2 ! c = c21 , c1 2 Z P(1) = a + b + c 2 P(−1) = a − b + c 2 ! ( a + b 2 Z a − b 2 Z ! ( 2a = a1 2 Z 2a = b1 2 Z P(4) = 16a + 4b + c21 = k2, k 2 Z ! 8a1 + 2b1 = k2 − c21 Do 8a1 + 2b1 ch®n nên k2 − c21 ch®n hay k và c1 cùng tính ch®n, l´ nên k2 − c21 ... 4 ! b1 ... 2 ! b = b1 2 2 Z Do a − b 2 Z nên tø ây ta có a 2 Z P(2) = 4a+ 2b + c21 = t2, t2 Z ! 4a+2b = t2 − c21 ! t2 − c21 là ch®n suy ra t và c1 cùng tính ch®n, l´ nên ta có t2 − c21 .. .4 ! b là ch®n Vªy a, b, c 2 Z và b ch®n. Bài 2. °t P = a2 + ab + b2 − 3a − 3b + 1989 ! 4P = a2 − 2ab + b2 + 3(a2 + b2 + 4 + 2ab − 4a − 4b) + 4.1989 − 12 = (a − b)2 + 3(a + b − 2)2 + 4.1986 4.1986 Suy ra P 1986, d§u = ¤t ÷ñc khi và ch¿ khi ( a − b = 0 a + b − 2 = 0 ! a = b = 1 34 www.vnmath.com
  • 37. 2.1. áp án tuyºn sinh lîp 10 nam 1989 (cho måi thí sinh) 35 Vªy P ¤t giá trà bé nh§t b¬ng 1986, ¤t ÷ñc khi a = b = 1 Bài 3. Gåi 52 sè nguyên d÷ìng b§t ky ã cho là a1, a2, . . . ,a52. Méi sè ai ·u có d¤ng ai = 100bi +ci, trong ó bi, ci 2 N và 0 ci 99, (i = 1, 52). N¸u trong sè c1, c2, . . . , c52 có hai sè b¬ng nhau, gi£ sû ci = ck ! ai−ak = 100(bi − bk) .. .100. N¸u t§t c£ c1, c2, . . . , c52 ôi mët khác nhau thì có ít nh§t 51 sè khác 50, gi£ sû ó là c1, c2, . . . , c51. Khi ó ta °t di = 100 − ci thì d1, d2, . . . ,d51 là các sè nguyên khác nhau và 1 di 100. Nh÷ vªy 102 sè c1, c2, . . . , c52, d1, d2, . . . ,d51 ch¿ nhªn không quá 101 giá trà (tø 0 ¸n 100) và do ó có 2 sè trong chúng b¬ng nhau. Do các sè c1, c2, . . . , c51 khác nhau và d1, d2, . . . ,d51 khác nhau nên hai sè b¬ng nhau là ci và dk nào ó suy ra ci = dk = 100 − ck ! ci+ck = 100, ð ây i6= k vì ci6= 50 ! ai+ak = 100(bi+bk)+100 ... 100. Bài 4 Kéo dài BE,CF các o¤n EI = BE và FK = CF. Khi ó 4ABI,4ACK cân ð A vàBAK =CAK = 30. 1. N¸u [BAC = 150 thì B,A,K th¯ng hàng, C,A,K th¯ng hàng và BK = BA = AK = IA + AC = IC. Do E,M, F là các trung iºm cõa IB,BC,CK nên EM = 1 2IC = 1 2BK = MF )4MEF cân ð M 2. Gåi giao iºm cõa IC và BK là O thì trong måi tr÷íng hñp ta ·u có A,B,O, I cùng n¬m trên mët ÷íng tròn và góc giúa hai tia BK,CI b¬ng 150. Tø ó ta cóMEF =MFE = 15. Bài 5. Gi£ sû theo thù tü 9 b¤n håc sinh là a1, a2, a3, a4, a5, a6, a7, a8, a9. Ta chùng minh bài toán b¬ng ph£n chùng. Gi£ sû ng÷ñc l¤i: Không có b¤n nào ùng cách ·u hai b¤n cùng lîp (1). Không m§t têng quát gi£ sû a5 là håc sinh lîp A, khi ó a4 và a6 không thº cùng thuëc lîp A. Vì vªy có hai kh£ nang sau: 1. a4 và a6 cùng thuëc lîp B. Khi ó do a4 cách ·u a2 và a6, còn a6 cách ·u a4 và a8 nên a2 và a8 thuëc lîp A suy ra a5 ùng cách ·u hai b¤n cùng lîp là a2 và a8, trái vîi gi£ thi¸t (1). 2. a4 và a6 thuëc hai lîp khác nhau, không m§t têng quát gi£ sû a4 thuëc lîp A còn a6 thuëc lîp B. Do a4 cách ·u a3 và a5, nên a3 thuëc lîp B. Do a6 cách ·u a3 và a9 nên a9 thuëc lîp A. Do a5 cách ·u a1 và a9 nên a1 thuëc lîp B. Do a2 cách ·u a1, a3 nên a2 thuëc lîp A. Do a5 cách ·u a2, a8 nên a8 thuëc lîp B. Do a6, a8 thuëc lîp B nên a7 thuëc lîp A. Nh÷ vªy a7 ùng cách ·u hai b¤n cùng lîp A là a5 và a9, trái vîi gi£ thi¸t (1). Vªy c£ hai kh£ nang a) và b) ·u d¨n ¸n vô lý nên i·u gi£ sû (1) là sai. www.vnmath.com
  • 38. 36 Ch÷ìng 2. áp án tuyºn sinh 2.2 áp án tuyºn sinh lîp 10 nam 1989 (cho thí sinh chuyên lý) Bài 1. f(x) = −2x2 + x + 36 2x + 3 = −x + 2+ 30 2x + 3 Vîi x nguyên f(x) là sè nguyên khi và ch¿ khi 2x + 3 là ÷îc sè cõa 30. Do 2x + 3 là l´ nên 2x + 3 ch¿ có thº nhªn mët trong tám giá trà là ±1,±3,±5,±15, tø ó thu ÷ñc 8 giá trà c¦n tìm cõa x là −9,−4,−3,−2,−1, 0, 1, 6. Bài 2. °t P = a2 + ab + b2 − 3a − 3b + 3 ta có 4P = a2 − 2ab + b2 + 3(a2 + b2 + 4 + 2ab − 4a − 4b) = (a − b)2 + 3(a + b − 2)2 0 Suy ra P 0, d§u = ¤t ÷ñc khi và ch¿ khi ( a − b = 0 a + b − 2 = 0 ) a = b = 1 Vªy P ¤t giá trà bé nh§t b¬ng 0, ¤t ÷ñc khi và ch¿ khi a = b = 1 Bài 3 1. Vîi m nguyên d÷ìng ta có m2 m2 + m + 1 (m + 1)2. Do ó m p m2 + m + 1 m+1 ! p m2 + m + 1 không là sè nguyên hay m2 + m + 1 không là sè chính ph÷ìng. 2. Gi£ sû ng÷ñc l¤i: m(m+1) b¬ng tích cõa 4 sè nguyên liên ti¸p, tùc là tçn t¤i a 2 Z mà m(m+1) = a(a+1)(a +2)(a+3) = (a2 +3a)(a2 + 3a + 2) = n(n + 2), vîi n = a2 + 3a 2 Z. Vªy m(m+1) = n(n+2) suy ra m2 +m+1 = n2 +2n+1 = (n+1)2 hay m2 +m+1 là sè chính ph÷ìng, trái vîi k¸t luªn câu 1) nên i·u gi£ sû m(m+ 1) b¬ng tích cõa 4 sè nguyên liên ti¸p là sai. Bài 4. www.vnmath.com
  • 39. 2.3. áp án tuyºn sinh lîp 10 nam 1989 (cho thí sinh chuyên toán - tin håc) 37 Gi£ sû AH c­t MC ð I. Gåi trung iºm cõa BH là K thì MK//AM. D¹ th§y ba tam giác vuông AMC, IAC và IMA çng d¤ng mà AC = 2AM nên IC = 2IA = 4IM suy ra MK MC = IM IC = 1 4 ! BH HC = 2HK HC = 1 2 Vªy BH HC = 1 2 Bài 5. Gåi 6 thành phè ã cho là A,B,C,D,E,F. Xét thành phè A. Trong 5 thành phè còn l¤i thì có ít nh§t 3 thành phè liên l¤c ÷ñc vîi A ho°c có ít nh§t ba thành phè không liên l¤c ÷ñc vîi A (vì n¸u sè thành phè liên l¤c ÷ñc vîi A không v÷ñt quá 2 và sè thành phè không liên l¤c ÷ñc vîi A cung không v÷ñt quá 2 thì ngoài A, sè thành phè còn l¤i không v÷ñt quá 4). Ta xét c£ hai kh£ nang. a) Sè thành phè liên l¤c ÷ñc vîi A không ít hìn 3, gi£ sû B,C,D liên l¤c ÷ñc vîi A. Theo gi£ thi¸t, trong 3 thành phè B,C,D có hai thành phè liên l¤c ÷ñc vîi nhau, khi ó hai thành phè này cùng vîi A là ba thành phè (ôi mët) liên l¤c ÷ñc vîi nhau. b) Sè thành phè không liên l¤c ÷ñc vîi A không ít hìn 3, gi£ sû ba thành phè không liên l¤c ÷ñc vîi A là D,E,F. Khi ó trong bë ba thành phè (A,D,E) thì D và E liên l¤c ÷ñc vîi nhau (vì D,E không liên l¤c ÷ñc vîi A). T÷ìng tü, trong các bë ba (A,E, F), (A,F,D) thì E và F liên l¤c ÷ñc vîi nhau, F và D liên l¤c ÷ñc vîi nhau và nh÷ vªy D,E,F là ba thành phè (ôi mët) liên l¤c ÷ñc vîi nhau. 2.3 áp án tuyºn sinh lîp 10 nam 1989 (cho thí sinh chuyên toán - tin håc) Bài 1. a4 + b4 + c4 − 2a2b2 − 2b2c2 − 2c2a2 = = (a2 + b2 − c2)2 − 4a2b2 = (a2 + b2 − c2 + 2ab)(a2 + b2 − c2 − 2ab) = [(a + b)2 − c2][(a− b)2 − c2] = (a + b + c)(a + b − c)(a − b + c)(a − b − c) Bài 2. www.vnmath.com
  • 40. 38 Ch÷ìng 2. áp án tuyºn sinh 1. x x2 + x+ 1 = − 2 3 ) x2 + x + 1 x2 = − 3 2 )x + 1 x + 1 = − 3 2 ) x + 1 x = − 5 2 ) x4 + x2 + 1 x2 = x2 + 1 x2 + 1 = x + 1 x 2 −1 = 25 4 −1 = 21 4 Vªy x2 x4 + x2 + 1 = 4 21 Chú ý: Có thº gi£i ph÷ìng trình x x2+x+1 = −2 3, thu ÷ñc hai nghi»m 2 . Tø ó ta có: x2 x4+x2+1 = 4 21. x1 = 2 và x2 = 1 2. P(x) = x2 x4 + x2 + 1 Ta có P(x) 0 vîi måi x6= 0 và P(0) = 0. Vîi x6= 0 ta có 1 P(x) = x2 + 1 x2 + 1 3 (vì x2 + 1 x2 2) D§u = ¤t ÷ñc khi và ch¿ khi x2 = 1 ! x = ±1. Vªy P(x) 1 3 hay P(x) ¤t giá trà lîn nh§t b¬ng 1 3, ¤t ÷ñc khi và ch¿ khi x = ±1. Bài 3. P(n) = an + bn + c ) 8 : P(1) = a + b + c P(2) = a2 + 2b + c P(3) = a3 + 3b + c Tø ó ta có P(2) − P(1) ... .. .m m , a(a − 1) + b ! 2ba(a − 1) + 2b2... m (1) và [a(a − 1) + b]2... m , a2(a − 1)2 + 2ba(a − 1) + b2.. .m (2) P(1) + P(3) − 2P(2) ... .. .m ) a(a − 1)2... m , a3 − 2a2 + a m ) a2(a − 1)2.. .m www.vnmath.com
  • 41. 2.3. áp án tuyºn sinh lîp 10 nam 1989 (cho thí sinh chuyên toán - tin håc) 39 Do ó, tø (2) ta có 2ba(a − 1) + b2... m (3) Tø (1) và (3) ta suy ra b2... m. Có thº th§y r¬ng tø gi£ thi¸t cõa bài toán không suy ra ÷ñc b .. .m. Thªt vªy, vîi a = 3, b = 2, c = 3 thì P(n) = 3n + 2n + 3. Chån m = 4 khi ó: .. .4, do ó P(n) = 3n + 1 + 2(n + 1) Vîi n l´ thì d¹ th§y 3n + 1 ... 4 ... Vîi n ch®n thì d¹ th§y 3n + 3 4, do ó P(n) = 3n + 3 + 2n ... 4 Vªy vîi måi sè nguyên d÷ìng n ta có P(n) ... 4, nh÷ng b = 2 không chia h¸t cho 4. Bài 4. Gåi trung iºm cõa CF là O thì MIOH và KLON là các hình bình hành. Suy ra hai o¤n MO, IH có chung trung iºm P và hai o¤n KO,LN có chung trung iºm Q. Có hai kh£ nang: 1. M,O,K không th¯ng hàng. Khi ó MQ là ÷íng trung tuy¸n chung cõa 4MLN và 4MOK. Do vªy trång tâm G cõa 4MLN cung là trång tâm 4MOK. T÷ìng tü, KP là ÷íng trung tuy¸n chung cõa 4MOK và 4IKH nên trång tâm G cõa 4MOK cung là trång tâm 4IKH. Vªy trång tâm hai tam giác MLN và IKH trùng nhau. 2. M,O,K th¯ng hàng. Gåi G và G0 là trång tâm 4MLN và 4IKH t÷ìng ùng. °t MP = PO = a,OQ = QK = b. Khi ó MG = 2 3 MQ = 2 3 (2a + b) = 4 3 a + 2 3 b KG0 = 2 3 KP = 2 3 (2b + a) = 4 3 b + 2 3 a ) MG0 = MK − KG0 = 2a + 2b − 4 3 b + 2 3 = a 4 3 a + 2 3 b = MG Do G,G0 cùng thuëc o¤n MK nên tø ây ta có G0 G Bài 5. Gåi têng sè håc sinh là T thì T = a1 + a3 + · · · + an. Gåi sè lîp có úng i håc sinh là pi (i = 1,M). 1. D¹ th§y T = 1p1 + 2p2 + 3p3 + · · · + MpM. M°t khác do dk là sè lîp www.vnmath.com
  • 42. 40 Ch÷ìng 2. áp án tuyºn sinh mà trong méi lîp ó có sè håc sinh không ít hìn k nên d1 = p1 + p2 + p3 + · · · + pM−1 + pM (1) d2 = p2 + p3 + · · · + pM−1 + pM (2) d3 = p3 + · · · + pM−1 + pM (3) . . . . . . dM−1 = pM−1 + pM (M-1) dM = pM (M) Cëng các ¯ng thùc trên l¤i ta ÷ñc d1+d2+· · ·+dM = 1p1+2p2+3p3+· · ·+MpM = T = a1+a2+· · ·+an 2. Ta có a21 +a22 +· · · +a2 n = p112 + p222 + · · ·+pMM2. M°t khác, nhân các ¯ng thùc (1), (2), (3), . . . , (M) ð câu 1) vîi 1, 3, 5, . . . , (2M − 1) t÷ìng ùng rçi cëng l¤i ta ÷ñc d1 + 3d2+5d3 + · · · + (2M − 1)dM = = p1 + (1 + 3)p2 + (1+3 + 5)p3 + · · ·+ + (1 + 3 + 5+· · · + (2M − 1))pM = 12P1 + 22p2 + 33p3 + · · · +M2pM = a21 + a22 + · · · + a2 n (Chú ý r¬ng 1+3 + 5 + (2k − 1) = k2) 2.4 áp án tuyºn sinh lîp 10 nam 1991 (cho måi thí sinh) Bài 1. 1. p a + x + p a − x p a + x − p a − x = p b (1) º các can có nghia ta ph£i có −a x a. Do v¸ ph£i d÷ìng nên p p a + x − a −x 0 suy ra x 0. Vªy i·u ki»n èi vîi x là: 0 x a. Vîi i·u ki»n ó (1) t÷ìng ÷ìng vîi p a2 − x2 2a + 2 p a2 − x2 2a − 2 = b ) 2a 1 − p a2 − x2 −1 = b 2a b + 1 = a − p a2 − x2 ) p a2 − x2 = a(b − 1) b + 1 (2) www.vnmath.com
  • 43. 2.4. áp án tuyºn sinh lîp 10 nam 1991 (cho måi thí sinh) 41 Do ó N¸u b 1 thì (2) vô nghi»m do ó (1) vô nghi»m N·u b 1 thì (2) t÷ìng ÷ìng vîi a2 − x2 = a2(b − 1)2 (b+ 1)2 ) x2 = a2 4b (b + 1)2 ) x = ± 2a b + 1 p b Lo¤i nghi»m âm ta ÷ñc nghi»m x = 2a b+1 p b tho£ mãn i·u ki»n p b). 0 x a (vì b + 1 2 p b b+1 Vªy (1) có nghi»m khi và ch¿ khi b 1 và nghi»m ó là x = 2a 2. x2 + ax + b + 1 = 0 (1) Gi£ sû x1, x2 là hai nghi»m nguyên cõa (1). Do b6= −1 nên x16= 0 và x26= 0. Theo ành lý Viét ta có ( x1 + x2 = −a x1x2 = b + 1 ! a2 + b2 = (x1 + x2)2 + (x1x2 − 1)2 = x21 + x22 + x21 x22 + 1 = (x21 + 1)(x22 + 1) Do x1, x2 là các sè nguyên khác 0 nên x21 +1 và x22 +1 là các sè nguyên không bé hìn 2 và nh÷ vªy a2 + b2 là hñp sè. Bài 2. 8 : a3x + a2y + az = 1 b3x + b2y + bz = 1 c3x + c2y + cz = 1 Nhân (1) vîi b và (2) vîi a rçi trø tøng v¸ cho nhau, sau ó chia cho a − b6= 0 ta ÷ñc ph÷ìng trình ab(a + b)x + aby = −1 (4) T÷ìng tü, nhân (1) vîi c và (3) vîi a rçi trø tøng v¸ cho nhau, sau ó chia cho a − c6= 0 ta ÷ñc ac(a + c)x + acy = −1 (5) www.vnmath.com
  • 44. 42 Ch÷ìng 2. áp án tuyºn sinh Khi ó h» ã cho t÷ìng ÷ìng vîi 8 : a3x + a2y + az = 1 ab(a + b)x + aby = −1 ac(a + c)x + acy = −1 Nhân (4) vîi c, (5) vîi b rçi trø tøng v¸ cho nhau, sau ó chia cho b−c6= 0 ta ÷ñc abcx = 1 hay x = 1 abc . Thay x = 1 abc vào (4) thu ÷ñc y = −a+b+c abc . Thay x = 1 abc, y = −a+b+c abc vào (1) thu ÷ñc z = ab+bc+ca abc . Vªy nghi»m cõa h» ã cho là x = 1 abc , y= − a + b + c abc , z= ab + bc + ca abc Bài 3 D¹ th§y 7x chia 4 d÷ 3 n¸u x l´ và d÷ 1 n¸u x ch®n. Ph÷ìng trình ã cho t÷ìng ÷ìng vîi 7x − 1 = 3.2y (1) N¸u x l´ thì 7x − 1 chia 4 d÷ 2 còn vîi y 2 thì 3.2y... 4 do ó y ch¿ có thº là 1. Vîi y = 1 ta ÷ñc nghi»m là x = 1, y = 1. N¸u x ch®n tùc là x = 2z (z nguyên d÷ìng) ph÷ìng trình (1) có d¤ng (7z + 1)(7z − 1) = 3.2y (2) Vì 2,3 là các sè nguyên tè, nên (2) là d¤ng phân tích cõa (7z + 1)(7z − 1) thành tích các thøa sè nguyên tè. Do 7z +1 chia 3 d÷ 2 nên 7z +1 = 2n.(3) vîi n là sè nguyên d÷ìng nào ó. Tø ó ta có 7z −1 = 2n − 2 suy ra (2) có d¤ng 2n(2n − 2) = 3.2y ) 2n+1(2n−1 − 1) = 3.2y Do ó 2n−1 −1 không chia h¸t cho 2 nên 2n−1 −1 = 3 hay n = 3. Thay vào (3) ta thu ÷ñc z = 1 suy ra x = 2. Thay x = 2 vào (1) ta ÷ñc nghi»m là x = 2, y = 4. Vªy ph÷ìng trình ã cho có hai nghi»m nguyên d÷ìng là x = 1, y = 1 và x = 2, y = 4 Bài 4. 1. Gåi các giao iºm cõa EF vîi AB,CD t÷ìng ùng là I,K. Qua F k´ ÷íng th¯ng song song vîi AB và CD c­t AD ð M, c­t BC ð N. Ta có MF DC = AM AD = BN BC = FN DC ) MF = FN www.vnmath.com
  • 45. 2.4. áp án tuyºn sinh lîp 10 nam 1991 (cho måi thí sinh) 43 Do AB//MN,CD//MN nên ba ÷íng th¯ng çng quy EC,ED,EK c­t các ÷íng th¯ng AB,MN,DC thành các o¤n th¯ng t l» IA IB = KD KC = FM FN = 1 Suy ra EF i qua các trung iºm I,K cõa AB và CD 2. SIJK = SNJC ) SICK = SICN ) KN//IC Khi ó theo (1) thì AJ i qua trung iºm E cõa KN suy ra SAKJ = SANJ mà SAKP = SCJN nên SAPJ = SAIC ) PJ = JC ) SBPJ = SBJC Do SBIM = SIJK nên tø ây ta có SBIKP = SCJIM. T÷ìng tü ta chùng minh ÷ñc SCJIM = SAKJN. Vªy di»n tích cõa ba tù giác không g¤ch chéo b¬ng nhau. Bài 5. Trên nûa ÷íng tròn ÷íng kính AB ta l§y 1991 iºm khác nhau và khác A,B : A1,A2,A3, . . . ,A1991. Gi£ sû Ai,Aj,Ak là ba iºm b§t www.vnmath.com
  • 46. 44 Ch÷ìng 2. áp án tuyºn sinh ky trong chúng, khi ó Ai,Aj,Ak không th¯ng hàng. Gi£ sû trên nûa ÷íng tròn ã cho AJ n¬m giúa Ai và Ak (tùc Aj thuëc cung nhä AjAk) khi ó AiAjAk 90 và 4AiAjAk là tam giác tù Vªy trên m°t ph¯ng tçn t¤i 1991 iºm mà ba iºm b§t ky trong chúng là ba ¿nh cõa mët tam giác tù. 2.5 áp án tuyºn sinh lîp 10 nam 1991 (cho thí sinh chuyên toán và chuyên tin) Bài 1. 1. Rút gån A = q 3 p 3 − 4 2 p 2 q 6 44 + 16 p 6 = q 3 p 3 − 4 2 q (2 p 2 6 p 3 + 4 p 2)2 q = 3 (2 p 3 − 4 p 2)(2 p 3 + 4 p 2) = − 3 p 20 2. P = (x − y)5 + (y − z)5 + (z − x)5 °t a = x − y, b = y − z, z − x = −(a + b) Khi ó P = a5 + b5 − (a + b)5 = (a + b)[a4 − a3b + a2b2 − ab3 + b4 − (a + b)4] = (a + b)(a4 − a3b + a2b2 − ab3 + b4 − a4 − 4a3b − 6a2b2 − 4ab3 − b4) = (a + b)[−5(a3b + a2b2 + ab3)] = −5(a + b)ab(a2 + ab + b2) = 5(x − y)(y − z)(z − x)[(x − y)2 + (x − y)(y − z) + (y − z)2] = 5(x − y)(y − z)(z − x)(x2 + y2 + z2 − xy − yz − zx) Bài 2. 1. 8 : a + b + c = 0 +
  • 47. + = 0 a +
  • 48. b + c = 0 www.vnmath.com
  • 49. 2.5. áp án tuyºn sinh lîp 10 nam 1991(cho thí sinh chuyên toán và chuyên tin)45 Tø ph÷ìng trình 1 và 2 ta có c = −(a+b), = −(+
  • 50. ). Thay vào ph÷ìng trình 3 ta ÷ñc a +
  • 51. b + c = 0 ! (b +
  • 52. a)(a+ b) + ( +
  • 53. )ab = 0 ! b2 +
  • 55. ) = 0 ! b2 +
  • 56. a2 − 2ab = 0 (1) T÷ìng tü
  • 57. c2 + b2 − 2bc = 0 (2) a2 + c2 − 2ca
  • 58. = 0 (3) Cëng (1), (2), (3) ta ÷ñc (
  • 59. + )a2 + ( + )b2 + ( +
  • 60. )c2 − 2(bc + ca
  • 61. + ab ) = 0 !−(a2 +
  • 62. b2 + c2) = 2(bc + ca
  • 63. + ab ) = a = 2abc +
  • 64. b + c = 0 Vªy A = a2 +
  • 65. +
  • 66. b2 + c2 = 0 2. 0 a, b, c, d 1. °t P = a + b + c + d − ab − bc − cd − da a) Ta có a − ab = a(1 − b) 0 d§u = ¤t ÷ñc , a = 0 ho°c b = 1 (1) b − bc 0 d§u = ¤t ÷ñc , b = 0 ho°c c = 1 (2) c − cd 0 d§u = ¤t ÷ñc , c = 0 ho°c d = 1 (3) d − da 0 d§u = ¤t ÷ñc , d = 0 ho°c a = 1 (4) Cëng bèn b§t ¯ng thùc trên ta ÷ñc P 0 Gi£ sû d§u = ¤t ÷ñc, khi ó c£ bèn b§t ¯ng thùc ð (1), (2), (3), (4) ph£i là ¯ng thùc. Tø (1) ta có a = 0 ho°c b = 1. N¸u a = 0 thì tø (4) ta có d = 0, do ó tø (3) ta có c = 0 và tø (2) suy ra b = 0. Vªy a = b = c = d = 0 www.vnmath.com
  • 67. 46 Ch÷ìng 2. áp án tuyºn sinh N¸u b = 1 thì tø (2) ta có c = 1, tø (3) ta có d = 1 và tø (4) suy ra a = 1. Vªy a = b = c = d = 1 Ng÷ñc l¤i d¹ th§y vîi a = b = c = d = 0 ho°c a = b = c = d = 1 thì p = 0 Tóm l¤i: P 0, P = 0 khi và ch¿ khi a = b = c = d = 0 ho°c a = b = c = d = 1 b) Tø (1 − a)(a − b) 0 ! 1 − a − b + ab 0 ! a + b − ab 1. Vªy a + b − ab 1, d§u = ¤t ÷ñc , a = 1 ho°c b = 1 (5) T÷ìng tü b + c − bc 1, d§u = ¤t ÷ñc , b = 1 ho°c c = 1 (6) c + d − cd 1, d§u = ¤t ÷ñc , c = 1 ho°c d = 1 (7) d + a − da 1, d§u = ¤t ÷ñc , d = 1 ho°c a = 1 (8) M°t khác (a + c)(b + d) 0 ! ab + bc + cd + da 0 !−(ab + bc + cd + da) 0 D§u = ¤t ÷ñc , a + c = 0 ho°c b + d = 0 ! a = c = 0 ho°c b = d = 0 (9) Cëng các b§t ¯ng thùc ð (5),(6),(7),(8) và (9) ta ÷ñc 2p 4 hay p 2 N¸u d§u = ¤t ÷ñc thì ð c£ 5 b§t ¯ng thùc ·u có d§u =, nh÷ vªy, tø (9) ta có a = c = 0 ho°c b = d = 0. Vîi a = c = 0 thì tø (5), (7) suy ra b = d = 1. Vîi b = d = 0 thì tø (5), (7) suy ra a = c = 1 Ng÷ñc l¤i, vîi a = c = 0, b = d = 1 ho°c b = d = 0, a = c = 1 thì P = 2. Tóm l¤i P 2, P = 2 khi và ch¿ khi a = c = 0, b = d = 1 ho°c b = d = 0, a = c = 1 Chú ý: Có thº gi£i cách khác nh÷ sau: P = (a + c)(1 − b − d) + b + d P = (b + d)(1 − a − c) + a + c ! 2P = (a + c)(2 − b − d) + (b + d)(2 − a − c) 1 4 [(a + c + 2 − b − d)2 + (b + d + 2 − a − c)2] www.vnmath.com
  • 68. 2.5. áp án tuyºn sinh lîp 10 nam 1991(cho thí sinh chuyên toán và chuyên tin)47 Do a + c + 2 − b − d 0, b+ d + 2 − a − c 0 nên tø ây ta có. 2P 1 4 [(a + c + 2 − b − d) + (b + d + 2 − a − c)]2 = 4 hay P 2 D§u = ¤t ÷ñc khi và ch¿ khi a + c + 2 − b − d = 0 b + d + 2 − a − c = 0 ! a = c = 0, b= d = 1 b = d = 0, a = c = 1 (vì 0 a, b, c, d 1) Bài 3. Gi£ sû k là sè tü nhiên tho£ mãn a 10k, d 10k. °t an = a + nd và xn = an 1991 = a 1991 + n d 1991 Do a 1991 10k và d 1991 0 nên tçn t¤i sè tü nhiên m mà xm−1 10k xm (1) M°t khác xm = xm−1 + d 1991 xm−1 + 10k 1991 10k + 10k 1991 = 1992.10k 1991 (2) Tø (1) và (2) ta có 10k xm 1992.10k 1991 ! 1991.10k 1991.xm 1992.10k Hay 1991.10k am 1992.10k Do ó am = a + md là sè có bèn chú sè ¦u tiên là 1991. Bài 4. Gåi 100 ng÷íi dü hëi th£o là a1, a2, a3, . . . ,a99, a100. Gi£ sû a1 quen bi¸t vîi 67 ng÷íi là a2, a3, . . . ,a68. Khi ó không kº a1 và 32 ng÷íi là a69, a70, . . . ,a100 thì a2 quen bi¸t vîi ít nh§t 34 ng÷íi trong sè a3, a4, . . . ,a68. Không m§t têng quát, gi£ sû a2 quen bi¸t vîi a3, a4, . . . ,a36. T÷ìng tü, không kº a1, a2 và 64 ng÷íi là a37, a38, . . . ,a100 thì a3 quen bi¸t vîi ít nh§t mët ng÷íi trong sè a4, a5, . . . ,a36, ch¯ng h¤n a3 quen bi¸t vîi a4. Tø ó ta có 4 ng÷íi là a1, a2, a3, a4 ôi mët quen bi¸t nhau. Bài 5. 1) Cùng phía vîi hình vuông èi vîi CD düng tam giác ·u M0CD. Khi ó 4ADM cân ð D và 4BCM cân ð C nên ta có ADM0 =BCM0 = 90 − 60 = 30 !DAM0 =CBM0 = 75 www.vnmath.com
  • 69. 48 Ch÷ìng 2. áp án tuyºn sinh M°t khác, d¹ th§y M0 n¬m cùng phía vîi hình vuông ABCD èi vîi AB. Theo gi£ thi¸t thì M n¬m trong hình vuông vàABM =BAM = 15 suy ra M0 M hay 4MCD ·u Chú ý: Có thº chùng minh b¬ng ph£n chùng nh÷ sau: Gi£ sû 4MCD không ·u. D¹ th§y 4AMD = 4BMC (c-g-c) suy ra MC = MD,MCD =MDC. Có hai kh£ nang a) MDC = MCD 60 suy ra MD CD và ADM 30. Mà DAM = 75 nênDMA 75 ! AD MD CD vô lý (Vì ABCD là hình vuông). b)MDC =MCD 60, lý luªn t÷ìng tü d¨n ¸n i·u vô lý, do ó gi£ thi¸t 4MCD không ·u là sai. 2) Trong hình vuông ABCD düng bèn iºm M,N,P,Q tho£ mãnMAB = MBA =NBC =NCB =PCD =PDC =QDA =QAD = 15 thì theo chùng minh trên, bèn tam giác MCD,NDA,PAB và QBC là ·u. Ta chùng minh tªp hñp 8 iºm {A,B,C,D,M,N, P,Q} tho£ mãn i·u ki»n bài toán. Có t§t c£ là 8.7 2 = 28 o¤n th¯ng nèi hai trong tám iºm trên. Ta chia chúng thành 6 nhóm sau: a) 4 c¤nh AB,BC,CD,DA cõa hình vuông ABCD b) 2 ÷íng chéo AC,BD cõa hình vuông ABCD c) 4 c¤nh MN,NP,PQ,QM cõa hình vuông MNPQ (d¹ th§y MNPQ là hình vuông) d) 2 ÷íng chéo MP,NQ cõa hình vuông MNPQ www.vnmath.com
  • 70. 2.6. áp án tuyºn sinh lîp 10 nam 1992 (cho måi thí sinh) 49 e) 8 o¤n MA,MB,NB,NC,PC,PD,QD,QA f) 8 o¤n MC,MD,ND,NA, PA, PB,QB,QC Ta chùng minh các o¤n ð nhóm e) và f) tho£ mãn i·u ki»n bài toán. (Vi»c chùng minh các o¤n ð các nhóm còn l¤i tho£ mãn i·u ki»n bài toán ìn gi£n hìn, b¤n åc tü chùng minh) Do 4AMD cân ð D vàADQ =MDQ = 15 nên DQ là trung trüc cõa AM hay trung trüc cõa AM i qua hai iºm D,Q, èi vîi các o¤n khác ð nhóm e) chùng minh t÷ìng tü. Do 4MCD ·u còn 4MQD cân ð Q nên trung trüc cõa MD i qua hai iºm C,Q. èi vîi các o¤n khác ð nhóm f) chùng minh t÷ìng tü. 2.6 áp án tuyºn sinh lîp 10 nam 1992 (cho måi thí sinh) Bài 1. 1. q p 2x − 5 + x + 2 + 3 q p 2x − 5 = 2 x − 2 − 3 p 2 (1) i·u ki»n: x 5 2 Nhân hai v¸ vîi p 2 ta có (1) t÷ìng ÷ìng vîi q p 2x − 5 + 9+ (2x − 5) + 6 q p 2x − 5+1 = 4 (2x − 5) − 2 q ) ( p 2x − 5 + 3)2 + q ( p 2x − 5 − 1)2 = 4 p 2x − 5 + 3+| ) p 2x − 5 − 1| = 4 (2) Vîi x 3 thì (2) có d¤ng p 2x − 5 + 3+ p 2x − 5 −1 = 4 ) p 2x − 5 = 1 )2x − 5 = 1 ) x = 3 (tho£ mãn x 3) 2 6 x 3 thì (2) có d¤ng Vîi 5 p 2x − 5 + 3 + 1 − p 2x −5 = 4 luôn tho£ mãn. Vªy nghi»m cõa (1) là 5 2 6 x 6 3. 2. ( xy2 − 2y + 3x2 = 0 y2 + x2y + 2x = 0 ) ( xy2 − 2y = −3x2 (1) x2y + 2x = −y2 (2) www.vnmath.com
  • 71. 50 Ch÷ìng 2. áp án tuyºn sinh D¹ th§y x = 0, y = 0 là mët nghi»m cõa h» và ng÷ñc l¤i n¸u (x, y) là nghi»m mà x ho°c y b¬ng 0 thì sè kia cung b¬ng 0. Ta tìm nghi»m tho£ mãn x6= 0, y6= 0. Vîi i·u ki»n x6= 0, y6= 0, chia (1) cho y2 và (2) cho x2 ta ÷ñc h» (1), (2) t÷ìng ÷ìng vîi ( x − 2 y = −3x2 y2 (3) x = −y2 x2 (4) y + 2 Nhân hai ph÷ìng trình vîi nhau ta ÷ñc h» (3), (4) t÷ìng ÷ìng vîi ( x − 2 y = −3x2 y2 x − 2 y y + 2 x = 3 ) ( xy2 − 2y = −3x2 (1) xy − 4 xy (5) Ta có (5) t÷ìng ÷ìng vîi (xy)2 − 3xy − 4 = 0 ) xy = −1 4 Thay xy = −1 hay y = −1 x vào (1) ta ÷ñc 1 x + 2 x = −3x2 ) x3 = −1 ) x = −1 ) y = 1 Thay xy = 4 hay y = 4 x vào (1) ta ÷ñc 16 x − 8 x = −3x2 ) x3 = − 8 3 ) x = − 2 3 p 3 ) y = −2 [ 3 ]3 Vªy h» ã cho có ba nghi»m là x = y = 0;x = −1, y = 1 và x = − 2 3 p 3 , y = −2 3 p 3 Bài 2. x2 − mnx + m + n = 0 (1) Vîi m = 0 ph÷ìng trình (1) có d¤ng x2 + n = 0. N¸u ph÷ìng trình có nghi»m nguyên a vîi n 0 nào ó thì a2 + n = 0 ) n = −a2 0 do ó n = 0 và nghi»m là x = a = 0. Lý luªn t÷ìng tü èi vîi tr÷íng hñp n = 0. Nh÷ vªy n¸u mët trong hai sè m, n b¬ng 0 thì (1) có nghi»m nguyên suy ra sè còn l¤i cung b¬ng 0 và nghi»m là x = 0. www.vnmath.com
  • 72. 2.6. áp án tuyºn sinh lîp 10 nam 1992 (cho måi thí sinh) 51 Ta xét tr÷íng hñp m, n 1. Khi ó gi£ sû a, b là các nghi»m cõa (1), theo ành lý Viét ta có ( a + b = mn ab = m + n (2) Do m, n 1 nên tø ây suy ra a, b 0 ) a, b 1 ) (a−1)(b−1) 0 hay ab − a − b + 1 0 ) m + n − mn+ 1 0 ) mn − m − n + 1 6 2 ) (m − 1)(n − 1) 6 2 (3) 1. Vîi m = 1 thì (1) có d¤ng x2 − nx + 1+n = 0 (2) x2 + 1 = n(x − 1) ) n = x2 + 1 x − 1 ) n = x+ 1+ 2 x − 1 N¸u ph÷ìng trình có nghi»m nguyên x thì x − 1 là ÷îc cõa 2, do ó x − 1 2 {1,−1, 2,−2}. Vîi x−1 = 1 tùc x = 2 thì n = 5, ph÷ìng trình (2) có d¤ng x2−5x+ 6 = 0 (có hai nghi»m x = 2, x = 3). Vîi x −1 = −1 tùc x = 0 thì n = −1 (lo¤i). Vîi x − 1 = 2 tùc x = 3 thì n = 5, ph÷ìng trình x = 2, x = 3. Vîi x −1 = −2 tùc x = −1 thì n = −1 (lo¤i). Vªy vîi m = 1 thì ph÷ìng trình (1) có nghi»m nguyên khi n = 5. Lý luªn t÷ìng tü ta có vîi n = 1 thì ph÷ìng trình (1) có nghi»m nguyên khi m = 5. Ta xét tr÷íng hñp m, n 2. 2. Vîi m = 2 thì tø (3) ta có n = 2 ho°c n = 3. Khi n = 2 thì (1) có d¤ng x2 − 4x + 4 = 0 có nghi»m nguyên x = 2. Khi n = 3 thì (1) có d¤ng x2 −6x+5 = 0 có nghi»m nguyên là x = 1 và x = 5. Nh÷ vªy vîi m = 2 thì (1) có nghi»m nguyên khi n = 2 ho°c n = 3. Lý luªn t÷ìng tü ta có vîi n = 2 thì (1) có nghi»m nguyên khi m = 2 ho°c m = 3. 3. Vîi m, n 3 ¯ng thùc (3) không tho£ mãn. Tóm l¤i: Ph÷ìng trình (1) có nghi»m nguyên (vîi m, n không âm) khi (m, n) là mët trong các c°p sè sau: (0, 0); (1, 5); (5, 1); (2, 2); (2, 3); (3, 2). www.vnmath.com
  • 73. 52 Ch÷ìng 2. áp án tuyºn sinh Bài 3. SMNP = S − SABM − SBCN − SCAP Qua A0 k´ ÷íng th¯ng song song vîi BB0 c­t CB0 t¤i I, ta có: AM MA0 = AB0 B0I = AB0 B0C . B0C B0I = 3. BC BA0 = 3.3 = 9 AM AA0 = AM AM +MA0 = 9 10 ) SABM SABA0 = AM AA0 = 9 10 mà SABA0 = S 3 ) SABM = 9 10 SABA0 = 3 10 S T÷ìng tü: Qua B0 k´ ÷íng th¯ng song song vîi CC0 c­t AC0 t¤i H, ta có BN NB0 = BC0 C0K = BC0 C0A . C0A C0K = 1. CA CB0 = 4 BN BB0 = 4 5 ) SBCN SBCB0 = BN BB0 = 4 5 mà SBCB0 = S 4 ) SBCN = 4 5 SBCB0 = S 5 Hoàn toàn t÷ìng tü ta tính ÷ñc SCAP = 2 5 S ) SMNP = S − 3 10 S + S 5 + 2 5 S = S 10 Bài 4. Trên c¤nh BC l§y iºm M sao cho BDM = ADC ) CDM = ADB. D¹ th§y 4BDM v 4ADC ) BM AC = DH DI (t sè hai ÷íng cao t÷ìng ùng cõa hai tam giác çng d¤ng b¬ng t sè çng d¤ng), suy ra BM DH = AC DI (1) T÷ìng tü 4CDM v 4ADB ) CM AB = DH DK ) CM DH = AB DK (2) Cëng (1) và (2) ta ÷ñc BC DM = AC DI + AB DK www.vnmath.com
  • 74. 2.6. áp án tuyºn sinh lîp 10 nam 1992 (cho måi thí sinh) 53 Bài 5. Gi£ sû m, n nguyên d÷ìng và 8 : ... 2m + 1 n ... 2n + 1 m ... ) (2m + 1)(2n + 1) mn ... ) 2m + 2n + 1 mn ) 2m + 2n + 1 = kmn (k 2 N) (1) Suy ra: k,m, n ·u l´. Vì m 1, n 1 ) (m − 1)(n − 1) 0 ) m+ n 6 mn + 1 2m + 2n + 1 6 2mn + 3 6 5mn (2) Do ó tø (1) suy ra k 6 5. 1. Vîi k = 5 ) 2m + 2n+1 = 5mn, tø (2) ta suy ra 3mn = 3 ) m = ... n = 1. Rõ ràng m = n = 1 tho£ mãn 2m + 1 ...m. n và 2n + 1 2. Vîi k = 3, gi£ sû m n, ta có 3mn = 2m + 2n + 1 6 5m ) 3n 6 5 ) n = 1 ) 3m = 2m + 3 ) m = 3. Rõ ràng n = 1,m = 3 tho£ mãn 2m + 1 ... ... n và 2n + 1 m. Vîi m 6 n ta ÷ñc n = 3,m = 1 tho£ mãn. 3. Vîi k = 1, gi£ sû m n, ta có mn = 2m + 2n + 1 6 5m ) n 6 5 ) n = 1 ho°c n = 3 ho°c n = 5. Vîi n = 1 ¯ng thùc mn = 2m + 2n + 1 trð thành m = 2m + 3 vô lý Vîi n = 3 ta có 3m = 2m + 6 + 1 ) m = 7. Rõ ràng n = 3,m = 7 tho£ mãn i·u ki»n 2m + 1 ... ... n; 2n + 1 m. Vîi n = 5 ta có 5m = 2m + 10 + 1 ) m = 11 3 (lo¤i). Vîi m 6 n lý luªn t÷ìng tü ta ÷ñc m = 3, n = 7 tho£ mãn i·u ki»n ... 2m + 1 ... n; 2n + 1 m. Tóm l¤i: Có 5 c°p sè (m, n) tho£ mãn yêu c¦u bài toán là (1, 1); (1, 3); (3, 1); (3, 7); (7, 3) . www.vnmath.com
  • 75. 54 Ch÷ìng 2. áp án tuyºn sinh 2.7 áp án tuyºn sinh lîp 10 nam 1992 (cho thí sinh chuyên toán và chuyên tin) Bài 1. 1. P = n4 + 2n3 + 2n2 + n + 7 = (n2 + n)2 + (n2 + n) + 7 Do n nguyên nên n2+n 0. °t k = n2+n ta có P = k2+k+7 k2. Ta có: P (k + 3)2 (vì(k + 3)2 − P = 5k + 2 0) Do ó k2 P (k + 3)2 ) P = (k + 1)2 ho°c p = (k + 2)2. Vîi P = (k + 1)2 tùc k2 + k + 7 = k2 + 2k + 1 ta thu ÷ñc k = 6 ) n2 + n = 6 ) n = 2 và n = −3. Vîi P = (k + 2)2 tùc k2 + k + 7 = k2 + 4k + 4 ta thu ÷ñc k = 1 ) n2 + n = 1 ) n = 2 không có nghi»m nguyên. Vªy P là sè chính ph÷ìng khi n = 2 và n = −3 (khi ó P = 49). 2. A = 1 a2 + 2bc + 1 b2 + 2ca + 1 c2 + 2ab °t x = a2 + 2bc, y = b2 + 2ac, z = c2 + 2ab thì x, y, z 0 và x+y+z = (a+b+c)2 6 1, do (x+y+z)A = (x+y+z) 1 x+1 y +1 z 9 mà x + y + z 6 1 nên A 9. Bài 2. Vîi n 2 N ta ký hi»u têng các chú sè cõa n là S(n). Ta có N = (29)1945 = (23)3.1945 = 85835 105835 Nên N có không quá 5835 chú sè mà a = S(N) 6 5835.9 = 52515 suy ra a có không quá 5 chú sè, b = S(a) 6 5.9 = 45. Trong các sè tü nhiên tø 0 ¸n 45 thì sè có têng các chú sè lîn nh§t là 39 và têng các chú sè cõa nó là 12. Suy ra S(b) 6 12. Ta bi¸t r¬ng vîi måi n 2 N thì S(n) n (mod 9), do ó S(b) b = S(a) a = S(N) (mod 9). Mà N = 85835 suy ra N −1 (mod 9) hay S(b) −1 (mod 9). Do S(b) 6 12 nên S(b) = 8. Vªy têng các chú sè cõa b là 8. Bài 3. Gi£ sû K n¬m ngoài o¤n BC v· phía C (tr÷íng hñp AB AC), khi ó d¹ th§y ACK nhån và ACB tù suy ra cung AB không chùa C lîn hìn cung ACE. Trên cung AB không chùa C l§y iºm E sao cho cung BE b¬ng cung AC, khi ó ACBE là hình thang cân. www.vnmath.com
  • 76. 2.7. áp án tuyºn sinh lîp 10 nam 1992(cho thí sinh chuyên toán và chuyên tin)55 Ta có AEB + EAB = EAC + EAB = 2(EAB + BAD) = 2EAD = 90 suy ra ABE = 90 nên AE là ÷íng kính cõa ÷íng tròn (O) ngo¤i ti¸p 4ABC. Do ó AB2 + AC2 = AB2 + EB2 = AE2 = 4R2 Chú ý: Có thº chùng minh b¬ng cách l§y iºm M trên cung AB không chùa C sao cho cung AM b¬ng cung AC, sau ó chùng minh BAM = 90, tø ó suy ra ¯ng thùc AB2 + AC2 = 4R2 Bài 4. Gåi các ÷íng th¯ng ã cho là d1, d2, . . . ,d1992 và giao iºm cõa ÷íng th¯ng di, dk là Aik ho°c Aki. a) Xét ÷íng th¯ng di b§t ky trong 1992 ÷íng th¯ng ã cho. Do không có 3 ÷íng th¯ng nào çng quy nên các giao iºm Akl cõa các c°p ÷íng th¯ng dk, dl(k6= i, l6= i) ·u n¬m ngoài di. Do sè giao iºm ó là húu h¤n nên có 1 giao iºm g¦n nó nh§t, gi£ sû ó là Akl. Ta chùng minh tam giác AklAkiAli là tam giác xanh. Thªt vªy n¸u tam giác ó bà ÷íng th¯ng dm nào ó trong 1989 ÷íng th¯ng còn l¤i c­t thì dm ph£i c­t mët trong hai o¤n AklAki ho°c AklAli, gi£ sû dm c­t o¤n AklAki t¤i Akm thì Akm g¦n di hìn Akl, trái vîi gi£ thi¸t Akl là iºm g¦n di nh§t. Nh÷ vªy vîi méi ÷íng th¯ng di luôn tçn t¤i mët tam giác xanh có c¤nh n¬m trên nó. Trên méi di ta chån mët c¤nh cõa mët tam giác xanh thì ta thu ÷ñc 1992 c¤nh khác nhau cõa các tam giác xanh. Tø ó suy ra sè tam giác xanh không ít hìn 1992 : 3 = 664. b) Xét ÷íng th¯ng di trong sè 1992 ÷íng th¯ng ã cho. N¸u trong méi nûa m°t ph¯ng có bí là di ·u có các giao iºm cõa các c°p ÷íng th¯ng còn l¤i thì trong méi nûa m°t ph¯ng ta l§y giao iºm g¦n di nh§t và lý luªn nh÷ câu a) ta ÷ñc hai tam giác xanh n¬m v· hai phía cõa di. Hai tam giác ó có hai c¤nh n¬m trên di và hai c¤nh ó là khác nhau (không có ba ÷íng nào çng quy). Ta chùng minh r¬ng sè ÷íng th¯ng mà các giao iºm cõa các c°p ÷íng th¯ng còn l¤i n¬m v· cùng mët phía cõa nó không v÷ñt quá 2. Thªt vªy, gi£ sû có 3 ÷íng th¯ng nh÷ vªy, ch¯ng h¤n ó là di, dk, dl. Khi ó xét ÷íng th¯ng dn khác, dn c­t di, dk, dl t¤i 3 iºm phân bi»t Ani,Ank,Anl. Trong 3 iºm ó có 1 iºm n¬m giúa hai iºm kia, gi£ sû Ank. Khi ó hai giao iºm Ani và Anl n¬m v· hai phía cõa dk trái vîi gi£ thi¸t. Vªy có ít nh§t 1990 ÷íng th¯ng mà v· hai phía cõa méi ÷íng ·u có các giao iºm cõa các ÷íng th¯ng còn l¤i. Theo lý luªn ð trên thì có hai tam giác xanh n¬m v· hai phía cõa méi ÷íng th¯ng ó và có hai c¤nh khác nhau n¬m trên nó. Trong hai ÷íng th¯ng còn l¤i, trên méi ÷íng th¯ng có ít nh§t mët c¤nh cõa mët tam giác xanh. Nh÷ vªy sè c¤nh khác nhau cõa các tam giác xanh không ít hìn 1990 × 2 + 2 = 3982 = 1327.3 +1. Suy ra sè tam giác xanh không ít hìn 1327 + 1 = 1328. Bài 5. www.vnmath.com
  • 77. 56 Ch÷ìng 2. áp án tuyºn sinh 2.8 áp án tuyºn sinh lîp 10 nam 1993 (cho måi thí sinh) Bài 1. 1. Gi£i ph÷ìng trình x + s x + 1 2 + r x + 1 4 = 2 (1) i·u ki»n: x −1 2 (1) ) x + s r x + 1 4 + 1 2 2 = 2 ) x + r x + 1 4 + 1 2 = 2 ) r x + 1 4 + 1 2 2 = 2 ) r x + 1 4 = p 2 − 1 2 ) x + 1 4 = 2+ 1 4 − p 2 ) x = 2− p 2(tho£ mãn i·u ki»n x − 1 2 ) 2. Gi£i h» ph÷ìng trình ( x3 + 2xy2 + 12y = 0 8y2 + x2 = 12 ) ( x3 + 2xy2 + (8y2 + x2)y = 0 (1) 8y2 + x2 = 12 (2) Ta có (1) t÷ìng ÷ìng vîi x3 + x2y + 2xy2 + 8y3 = 0 (3) D¹ th§y h» không có nghi»m vîi y = 0, vì n¸u y = 0 thì tø (3) suy ra x = 0 không tho£ mãn (2). Vîi y6= 0 t÷ìng ÷ìng vîi x y 3 + x y 2 + 2 x y + 8 = 0 (4) °t x y = t thì (4) có d¤ng t3 + t2 + 2t + 8 = 0 ) (t + 2)(t2 − t+4) = 0 ) t = −2 Tø ó x y = −2 ) x = −2y. Thay vào (2) ta ÷ñc 12y2 = 12 ) y = ±1 ) x = 2 Vªy h» ã cho có hai nghi»m x = 2, y = −1 và x = −2, y = 1 www.vnmath.com
  • 78. 2.8. áp án tuyºn sinh lîp 10 nam 1993 (cho måi thí sinh) 57 Bài 2. Tr÷îc h¸t ta chùng minh r¬ng: Vîi a, b, c, d 0 thì abcd 6 a + b + c + d 4 4 (1) D§u = ¤t ÷ñc khi và ch¿ khi a = b = c = d. Ta có vîi a, b 0 thì ab 6 a+b 2 2 , d§u = ¤t ÷ñc khi và ch¿ khi a = b. Do ó vîi a, b, c, d 0 thì abcd 6 a + b 2 2c + d 2 2 = a + b 2 . c + d 2 2 6 ha + b + c + d 4 2i2 = a + b + c + d 4 4 ) abcd 6 a + b + c + d 4 4 N¸u trong a, b, c, d có mët sè b¬ng 0 thì d§u = ¤t ÷ñc khi và ch¿ khi a = b = c = d = 0. N¸u c£ 4 sè a, b, c, d ·u d÷ìng thì d§u = ¤t ÷ñc khi và ch¿ khi 8 : a = b c = d a + b = c + d ) a = b = c = d Tóm l¤i abcd 6 a+b+c+d 4 4 d§u = ¤t ÷ñc khi và ch¿ khi a = b = c = d. Xét A = x2y(4 − x − y) vîi x 0, y 0, x + y 6 6 (1) a) Giá trà lîn nh§t: Vîi x + y 4 thì A 6 0 Vîi x +y 4 ta có A = 4. x 2 . x 2 x .y(4 − x − y) 6 4 2 + x 2 + y + 4 − x − y 4 4 = 4 D§u = ¤t ÷ñc khi và ch¿ khi x 2 = y = 4 − x − y ) x = 2, y = 1. Vªy A ¤t giá trà lîn nh§t b¬ng 4 ¤t ÷ñc khi x = 2, y = 1 (tho£ mãn i·u ki»n (1)). b) Giá trà bé nh§t. Vîi x + y 6 4 thì A 0. Vîi 4 x+ y 6 6 ta có − A 4 = x 2 . x 2 .y(x + y − 4) 6 x 2 + x 2 + y + x + y − 4 4 4 = h2(x + y) − 4 4 i4 6 2.6 − 4 4 4 = 16 www.vnmath.com
  • 79. 58 Ch÷ìng 2. áp án tuyºn sinh Vì x+y 6 6 nên A −64. D§u = ¤t ÷ñc khi và ch¿ khi x 2 = y = x+y−4 và x + y = 6 ) x = 4, y = 2. Vªy A ¤t giá trà bé nh§t b¬ng −64, ¤t ÷ñc khi x = 4, y = 2. Bài 3. K´ ÷íng trung trüc cõa AB c­t AC ð O1, c­t BD ð O2 thì O1,O2 là tâm các ÷íng tròn ngo¤i ti¸p 4ABD và 4ABC suy ra R = O1A, r = O2B. 4AIO1 v 4AOB ) O1A AB = AI AO ) R = O1A = AB.AI AO = a2 AC ) 1 R2 = AC2 a4 T÷ìng tü: 1 r2 = BD2 a4 ) 1 R2 + 1 r2 = AC2 + BD2 a4 = 4AB2 a4 = 4 a2 Bài 4. Gi£ sû trên ÷íng tròn (O) chi·u i tø A ! B ! C ! A là ng÷ñc chi·u kim çng hç và gi£ sû quay 4ABC mët góc 90 thuªn chi·u kim çng hç quanh (O) ta thu ÷ñc 4A1B1C1. Khi ó A1,B1,C1 thuëc các cung nhä AC,AB,BC t÷ìng ùng. Do ó c¤nh A1B1 ph£i c­t các c¤nh AB,AC, gi£ sû l¦n l÷ñt t¤i M,N. T÷ìng tü ta có c¤nh A1C1 c­t các c¤nh AC,BC l¦n l÷ñt t¤i P,Q và c¤nh B1C1 c­t các c¤nh BC,BA l¦n l÷ñt t¤i T,K, suy ra ph¦n chung cõa hai hình tam giác ABC và A1B1C1 là löc giác MNPQTK. Gåi di»n tích löc giác ó là S thì S = SABC − SAMN − SBKT − SCPQ Ta có: SABC = p 3 4 BC2 = p 3R2 4 3 Khi quay mët góc 90 thì OA1?OA,OB1?OB,OC1?OC mà BC?OA nên OA1 k BC, t÷ìng tü: OB1 k CA,OC1 k AB. Gåi giao iºm cõa OA1 vîi AC là E. Do các cung AA1,BB1,CC1 có sè o b¬ng 90 nên AMN = 90 mà MAN = 60 nên suy ra AN = 2AM. D¹ th§y 4NEA1 cân ð E (các góc ð áy b¬ng 30). Do OE k BC còn AO b¬ng 2 3 trung tuy¸n AA0 cõa 4ABC nên AE = 2 3 AC = 2 p 3 R,OE = BC 3 = R p 3 ) EN = EA1 = OA1 − OE = R − R p 3 ) AN = AE − EN = 2 p 3 R − R − R p 3 = ( p 3 − 1)R ) SAMN = AM.MN 2 = 1 2 AN 2 p 3 2 AN = p 3 8 AN2 = = p 3 p 3(4 − 2 8 R2 = p 3 − 3 2 4 R2 www.vnmath.com
  • 80. 2.8. áp án tuyºn sinh lîp 10 nam 1993 (cho måi thí sinh) 59 T÷ìng tü ta có SBKT = SCPQ = 2 p 3−3 4 R2. Vªy S = p 3 4 9 − 3 R2 . Bài 5. A = 1 a + 1 b + 1 c + 1 ab + 1 bc + 1 ca = ab + bc + ca + a + b + c abc (1) Ta chùng minh ba sè a, b, c cùng ch®n ho°c cùng l´. N¸u abc là l´ thì méi sè a, b, c ·u l´. N¸u abc là ch®n thì mët trong ba sè ph£i ch®n, ch¯ng h¤n a ch®n. Vì tû sè ð (1) chia h¸t cho abc nên tû sè ph£i ch®n suy ra bc + b + c ch®n hay (b + 1)(c + 1) − 1 ch®n ) (b + 1)(c + 1) l´. Vªy b + 1 và c+ 1 là l´ hay b, c ch®n. Vªy a, b, c cùng ch®n ho°c cùng l´. Vì a, b, c ôi mët khác nhau nên ta có thº gi£ sû r¬ng a b c. Khi ó, a 6 2, vì n¸u a 3 thì b 5, c 7, do ó A 6 1 3 + 1 5 + 1 7 + 1 15 + 1 35 + 1 21 1 Suy ra A không nguyên. a) Vîi a = 2 thì b 4, c 6 và A = 1 2 + 1 b + 1 c + 1 2b + 1 2c + 1 bc 6 1 2 + 1 4 + 1 6 + 1 8 + 1 12 + + 1 16 + 1 24 = 28 24 2 )A = 1. Vîi b 6 thì c 8 khi ó A 6 1 2 + 1 6 + 1 8 + 1 12 + 1 16 + 1 48 = 46 48 1 )A không nguyên Do ó b = 4. Vªy a = 2 thì b = 4 suy ra 1 = 1 2 + 1 4 + 1 c + 1 8 + 1 4c + 1 2c . Tø ây ta thu ÷ñc c = 14. Vªy vîi a = 2 ta ÷ñc nghi»m là a = 2, b = 4, c = 14. Chú ý: Tø i·u ki»n a = 2 và A = 1 ta ÷ñc ph÷ìng trình 1 = 1 2 + 1 b + 1 c + 1 2b + 1 2c + 1 bc ) 2b + 2c + b + c + 2 2bc = 1 2 )3b + 3c + 2 = bc ) (b − 3)(c − 3) = 11 ) ( b − 3 = 1 c −3 = 11 ) ( b = 4 c = 14 www.vnmath.com
  • 81. 60 Ch÷ìng 2. áp án tuyºn sinh Do ó ta cung tìm ÷ñc nghi»m trên. b) a = 1, khi ó b 3, c 5 và A = 1+ 2 b + 2 c + 1 bc = 32 15 3 mà A 1 ) A = 2 ) 2 b + 2 c + 1 bc = 1 Khi ó n¸u b 5 thì 2 b + 2 c + 1 bc 1, do ó b ch¿ có thº là 3. Vîi b = 3 ta ÷ñc 2 3 + 2 c + 1 3c = 1 ) c = 7. Vªy vîi a = 1 ta ÷ñc nghi»m là a = 1, b = 3, c = 7. Tóm l¤i, vîi gi£ thi¸t a b c ta có hai nghi»m là (2, 4, 14) và (1, 3, 7). Thay êi vai trò a, b, c ta thu ÷ñc 12 nghi»m là các cách s­p thù tü cõa ba sè 2, 4, 14 và ba sè 1, 3, 7. 2.9 áp án tuyºn sinh lîp 10 nam 1994 (cho måi thí sinh) Bài 1. 1. x4 − 2x3 − 6x2 + 16x − 8 = 0 (1) Phân tích v¸ trái thành các nhân tû ta ÷ñc (1) (x − 2)2(x2 + 2x − 2) = 0 , x = 2 −1 ± p 3 Vªy ph÷ìng trình có ba nghi»m: x = 1, x= −1 + p 3, và x = −1 − p 3 2. x2 + 2x + 4 = 3 p x3 + 4x p x2 + 4 + 2x − 3 (x2 + 4)x = 0 i·u ki»n: x 0 °t: p x2 + 4 = u, p x = v thì ph÷ìng trình có d¤ng: u2 + 2v2 − 3uv = 0 (u − v)(u − 2v) = 0 u = v 2v Vîi u = v ta ÷ñc p x2 + 4 = p x , x2 + 4 = x vô nghi»m. Vîi u = 2v ta ÷ñc p x2 + 4 = 2 p x , x2 + 4 = 4x , x = 2. www.vnmath.com
  • 82. 2.9. áp án tuyºn sinh lîp 10 nam 1994 (cho måi thí sinh) 61 Vªy ph÷ìng trình có nghi»m duy nh§t: x = 2. Bài 2. A = p zt p xy + 2 p xyzt = xy + 4zt + 2(2 A2 = xy + 4zt + 4 p xt) p yz. 6 xy + 4zt + 2yz + 2xt = 9 Tø gi£ thi¸t suy ra A 6 3. D§u = ¤t ch¯ng h¤n khi x = y = z = t = 1, (tho£ mãn xy + 4zt + 2yz + 2xt = 9). Vªy A ¤t giá trà lîn nh§t b¬ng 3. Bài 3. ( xy − 3zt = 1 (1) xz + yt = 2 (2) ) ( x2y2 − 6xyzt+ 9z2t2 = 1 (3) x2z2 + 2xyzt + y2t2 = 4 (4) Nhân (4) vîi 3 và cëng tøng v¸ vîi (3) ta có x2y2 + 9z2t2 + 3x2z2 + 3y2t2 = 13 Vì x, y, z, t là các sè nguyên mà têng các h» sè cõa v¸ trái là 16 nên n¸u x, y, z, t tho£ mãn h» ã cho thì ph£i có mët sè b¬ng 0. N¸u x = 0 ho°c y = 0 thì tø (1) ta có: −3zt = 1, vô lý. N¸u z = 0 h» có d¤ng ( xy = 1 yt = 2 H» này có hai nghi»m nguyên là: x = y = 1, t = 2 và x = y = −1, t = −2 N¸u t = 0 h» có d¤ng ( xy = 1 xz = 2 H» này có hai nghi»m nguyên là: x = y = 1, z = 2 và x = y = −1, z = −2 Vªy h» ã cho có 4 nghi»m nguyên là x = y = 1, z = 0, t = 2; x = y = −1, z = 0, t = −2 x = y = 1, t = 0, z = 2; x = y = −1, t = 0, z = −2 Bài 4. Gåi tâm ÷íng tròn ã cho là O và trung iºm cõa AB là I thì OI?AB và AI = BI = AD = DC. °t CD = x. www.vnmath.com
  • 83. 62 Ch÷ìng 2. áp án tuyºn sinh Do AH k OB (cùng k BC), nên OBI = BAH. Tø ó suy ra 4OBI v 4BAH ) AI AH = OA AB ) AH = AI.AB OA = 2x2 R (2.1) M°t khác do ÷íng tròn (O) ti¸p xúc vîi BC t¤i B nên CD.CA = BC2 = 4BH2 = 4(AB2 − AH2) = 16x2 − 4AH2 ) 2x2 = 16x2 − 4AH2 ) AH2 = 7 2 x2 (2.2) Tø (2.1) và (2.2) ta suy ra 4x4 R2 = 7 2 x2 ) x2 = 7 8 R2 ) x = R 2 r 7 2 Vªy AD = x = R 2 r 7 2 (2.3) Tø (2.1) và (2.3) suy ra AH = 7 4 R ) HB2 = AB2 − AH2 = 4x2 − 49 16 R2 = 28 8 R2 − 49 16 R2 = 7 16 R2 Do HE.HA = HB2 nên ta có HE = HB2 HA = R 4 ) AE = AH − HE = 3 2 R Bài 5. Do BC AC nên BAC ABC. Trong nûa m°t ph¯ng có bí là AB và chùa C k´ tia Bx sao cho ABx = BAC. Bx c­t ÷íng th¯ng MN t¤i P thì M n¬m giúa N và P (vì ABP ABM). Khi ó ABPN là hình thang cân nên APN = BNP. Xét 4AMP ta có: AMP ANM BNM BNP = APN APM. Do ó: AM AP = BN. 2.10 áp án tuyºn sinh lîp 10 nam 1994 (cho thí sinh chuyên toán và chuyên tin) Bài 1. 8 : (x + y)(y + z) = 4xy2z (1) (y + z)(z + x) = 4yz2x (2) (z + x)(x + y) = 4zx2y (3) www.vnmath.com
  • 84. 2.10. áp án tuyºn sinh lîp 10 nam 1994(cho thí sinh chuyên toán và chuyên tin)63 Rõ ràng x = y = z = 0 là mët nghi»m cõa h». Ng÷ñc lai, d¹ th§y n¸u (x, y, z) là nghi»m cõa h» mà môt trong ba sè x, y, z b¬ng 0 thì hai sè kia cung b¬ng 0. Ta tìm nghi»m tho£ mãn x6= 0, y6= 0, z6= 0. Ta chùng minh n¸u (x, y, z) là nghi»m mà x6= 0, y6= 0, z6= 0 thì x = y = z. Thªt vªy, n¸u (x, y, z) tho£ mãn (1), (2), (3) thì x + y6= 0, y + z6= 0, z + x6= 0, do ó chia (1) cho (2) ta ÷ñc x + y z + x = y z ) xz + yz = yz + xy ) x(y − z) = 0 ) y = z T÷ìng tü, chia (2) cho (3) ta thu ÷ñc z = x. Vªy vîi i·u ki»n x6= 0, y6= 0, z6= 0 h» (1),(2),(3) t÷ìng ÷ìng vîi ( x = y = z 4x2 = 4x4 ) x = y = z = ±1 Vªy h» ã cho có ba nghi»m là (x, y, z) = 2 4 (0, 0, 0) (1, 1, 1) (−1,−1,−1) Bài 2. 12x2 + 6xy + 3y2 = 28(x + y) ) 3(4x2 + 2xy + y2) = 28(x + y) (1) Do 3 và 28 nguyên tè cùng nhau nên x + y .. .3 hay x + y = 3k vîi k 2 Z. Tø (1) suy ra 3x2 + (x + y)2 = 28k ) 3x2 + 9k2 = 28k ) k .. .3 hay k = 3n(k 2 Z) )x2 + 3k2 = 28n mà k = 3n ) x2 + 27n2 = 28n ) x2 = n(28 − 27n) 0 )n 28 27 − n 0 ) 0 6 n 6 28 27 ) n = 0 ho°c n = 1 Vîi n = 0 ) k = 0 ) ( x = 0 x + y = 0 ) x = y = 0 Vîi n = 1 ) k = 3 ) ( x2 = 1 x + y = 9 ) x = 1; y = 8 x = −1; y = 10 Vªy ph÷ìng trình ã cho có ba nghi»m nguyên là x = y = 0; x = 1, y = 8 và x = −1, y = 10 www.vnmath.com
  • 85. 64 Ch÷ìng 2. áp án tuyºn sinh Bài 3. Ký hi»u A = 1.2.3 . . . n = n! (åc là n giai thøa). Ta có B = 1 + 2 + 3+· · · + n = n(n + 1) 2 (n 3) ... Vîi n = 3 thì rõ ràng A = B = 6 suy ra A B. Ta xét n 4. Khi ó có hai kh£ nang sau: a) n+ 1 là sè nguyên tè. Ta chùng minh A không chia h¸t cho B. Thªt ... vªy, n¸u A B thì n! = k n(n + 1) 2 ) 2(n − 1)! = k(n + 1) i·u này vô lý vì n+1 là sè nguyên tè nên (n+1) và các sè 1, 2, . . . ,n −1 là nguyên tè cùng nhau. b) n + 1 là hñp sè. Khi ó n + 1 = p.q (p, q 2 N, p, q 2) (1) Suy ra n + 1 2p hay p 6 n+1 2 . Do n 3 ta có 2n n+ 3 suy ra 2n − 2 n + 1 hay n+1 2 n− 1 nên p n− 1. T÷ìng tü q n− 1. Do ó n¸u n +1 có thº vi¸t ÷ñc d÷îi d¤ng (1) vîi p6= q thì p, q là các sè tü nhiên nhä hìn n − 1 nên trong tích (n − 1)! = 1.2.3 . . . (n − 1) có hai thøa sè là p và q suy ra (n − 1)! ... (p − q) = n + 1 ) n! .. .B .. .n(n − 1) ) A N¸u n+1 có d¤ng (1) vîi p = q tùc n+1 = p2(p 2) và p là hñp sè thì .. .B. n + 1 cung có d¤ng (1) vîi p6= q do ó A Ta xét tr÷íng hñp n + 1 = p2 vîi p sè nguyên tè. Khi ó, do n + 1 5 nên p 3 suy ra p2 9 hay n 8. Ta chùng minh p2 = n + 1 n − 1 2 2 (2) Ta có (2) t÷ìng ÷ìng vîi 4n + 4 n2 − 2n + 1 ) n2 − 6n − 3 0 ) (n2 − 8n) + (2n − 3) 0 B§t ¯ng thùc này úng vì n 8, do vªy (2) úng. Tø (2) suy ra p n−1 2 . Khi ó (n − 1)! = 1.2 . . . p.(p + 1. . . (n − 1). Do (n − 1) 2p nên www.vnmath.com
  • 86. 2.10. áp án tuyºn sinh lîp 10 nam 1994(cho thí sinh chuyên toán và chuyên tin)65 tích (p+1) . . . (n−1) có nhi·u hìn p thøa sè do ó có mët thøa sè chia h¸t cho p nên ta có (n − 1)! ... p2 = n + 1 ) n! ... .. .B n(n + 1) ) A K¸t hñp vîi tr÷íng hñp n = 3 ta có k¸t luªn: Vîi n + 1 là sè nguyên tè thì A không chia h¸t cho B. Vîi n + 1 là hñp sè thì A .. .B. Bài 4. Ta chùng minh r¬ng vîi x, y 1 ta có 1 1 + x + 1 1 + y 1 1 + p xy (1) Ta có (1) t÷ìng ÷ìng vîi (1 + y)(1 + p xy) + (1 + x)(1 + p xy) − 2(1 + x)(1 + y) 0 )1 + p xy + y + y p xy + 1+ p xy + x + x p xy − 2 − 2x − 2y − 2xy 0 )x p xy + y p xy − 2xy − x − y + 2 p xy 0 p xy( ) p x − p y)2 − ( p x − p y)2 0 )( p xy − 1)( p x − p y)2 0 B§t ¯ng thùc này úng vì x, y 1 hay (1) úng (có thº th§y r¬ng d§u = ¤t ÷ñc khi và ch¿ khi x = y ho°c xy = 1). Áp döng vîi a, b, c 1 ta có 1 1 + a + 3 1 + b = 1 1 + a + 1 1 + b + 1 1 + 2 1 + b 2 p ab + 1 1 + b 4 1 + p b p ab = 4 1 + 4 p ab3 Vªy 1 1 + a + 3 1 + b 4 1 + 4 p ab3 (2) T÷ìng tü 1 1 + b + 3 1 + c 4 1 + 4 p bc3 (3) www.vnmath.com
  • 87. 66 Ch÷ìng 2. áp án tuyºn sinh và 1 1 + c + 3 1 + a 4 1 + 4 p ca3 (4) Cëng (2), (3), (4) rçi chia cho 4 ta ÷ñc b§t ¯ng thùc ph£i chùng minh. Bài 5. 1. Gi£ sû BAC = 20. Trên các c¤nh AB,AC l§y các iºm D,K t÷ìng ùng sao cho AD = KC = BC (chú ý AB = AC BC). Ta chùng minh AD = DK = KC Phía trong 4ABC düng tam giác ·u BCI thì A, I n¬m trên trung trüc cõa BC suy ra AI là phân giác cõa góc BAC. Khi ó ACB = 80 ) ACI = 20 = CAD, mà AD = BC = CI nên d¹ th§y ACID là hình thang cân (áy là AC và ID), tø ó ta có AC k ID ) DIA = IAC = IAD = 10 ) 4ADI cân ð D. Suy ra ID = AD = CK nên CIDK là hình bình hành suy ra DK = BC. Vªy AD = DK = KC = CB. 2. Ng÷ñc l¤i, gi£ sû tçn t¤i các iºm D và K trên các c¤nh AB,AC t÷ìng ùng sao cho AD = DK = KC = CB. K´ o¤n th¯ng CI song song, cùng chi·u và b¬ng KD thì CKDI là hình bình hành và là hình thoi. Do CI = DK = AD và DAC = AKD = ACI nên ACID là hình thang cân. Vì CD là phân giác cõa ACI (CKDI là hình thoi) nên d¹ th§y AI là phân giác cõa góc DAC tø ó ta có AI là trung trüc cõa BC suy ra IB = IC = BC hay 4IBC ·u. °t BAC = x, ta có ABI = ACI = BAC = x ) IBC + ICB + 3x = 180 hay 120 + 3x = 180 ) BAC = x = 20 2.11 áp án tuyºn sinh lîp 10 nam 1995 (cho måi thí sinh) Bài 1. ( 2x2 − y2 = 1 xy + x2 = 2 ) ( 2x2 − y2 = 1 (1) xy + x2 = 2(2x2 − y2) (2) www.vnmath.com
  • 88. 2.11. áp án tuyºn sinh lîp 10 nam 1995 (cho måi thí sinh) 67 Tø (2) suy ra 2y2 + xy − 3x2 = 0 ) (y − x)(2y + 3x) = 0 ) y = x y = −3 2x Vîi y = x thay vào (1) ra ÷ñc ph÷ìng trình: x2 = 1 Do ó h» có nghi»m là x = y = ±1 Vîi y = −3 2x thay vào (1) ta ÷ñc ph÷ìng trình: −x2 2 = 1 (vô nghi»m) Vªy h» ã cho có hai nghi»m: x = y = 1 và x = y = −1. Bài 2. p 1 − x + p 4 + x = 3 (2.4) i·u ki»n: −4 6 x 6 1. Khi ó (2.4) t÷ìng ÷ìng vîi p 1 − x + 4+x + 2 (1 − x)(4 + x) = 9 p 4 − 3x − x2 = 2 ) )4 − 3x − x2 = 4 )x2 + 3x = 0 ) x = 0 x = −3 C£ hai nghi»m ·u tho£ mãn i·u ki»n. Vªy ph÷ìng trình (2.4) có hai nghi»m: x = 0 và x = −3. Bài 3. a + 1 b + b + 1 a = a2 + b2 + a + b ab = (a + b)2 + (a + b) ab − 2 Do a+1 b + b+1 a 2 N nên (a + b)2 + (a + b) ab 2 N ) (a + b)2 + (a + b) = ka (1) vîi k 2 N. N¸u d 0 là ÷îc sè chung cõa a, b thì a = md, b = nd, (n,m 2 N) ) a + b = (m + n)d, ab = mnd2. Do ó (1) có d¤ng (m+ n)2d2 + (m + n)d = kmnd2 )m+ n = [kmn − (m + n)2]d = ld (l 2 N) p )a + b = ld2 d2 ) d 6 a + b Bài 4. Gåi di»n tích cõa hai hình chú nhªt là S thì ta có: ab = cd = S ) b = S a , d = S c và a + b − (c + d) = a + S a − (c + S c ) = a − c − ( S c − S a ) = (a − c)(1 − S ac 0 www.vnmath.com
  • 89. 68 Ch÷ìng 2. áp án tuyºn sinh (vì a c d nên a − c 0 và S ac S dc = 1). Vªy a+b c + d và chu vi hình chú nhªt thù nh§t lîn hìn chu vi hình chú nhªt thù hai. Bài 5. 1. D¹ th§y 4ABE v 4AEC nên ta có: AE AB = AC AE ) AE2 = AB.AC ) AF = AE = p AB.AC không êi Vªy E,F luôn ch¤y trên ÷íng tròn cè ành tâm A, bán kính p AB.AC. 2. Gi£ sû O /2 BC và ÷íng th©ng OI c­t cung BC không chùa F t¤i M. Khi ó do A,E, F,O, I cùng thuëc ÷íng tròn tâm AO nên trong måi tr÷íng hñp ta có EOM = EFI EFE0 = 1 2EOE0 Vªy OM là ÷íng phân giác cõa góc EOE0 suy ra OM?EE0 hay OI?EE0. Mà OI?BC nên EE0 k BC AB. Tr÷íng hñp O 2 BC khi ó O I thì FEE0 = 900 mà FE?BC nên EE0 k BC. 3. Gi£ sû O6= I. Gåi giao iºm cõa BC và EF là P thì ÷íng tròn ngo¤i ti¸p 4ONI là ÷íng tròn ÷íng kính OP(PNO = PIO = 90). D¹ th§y AP.AI = AN.AO = AE2 = AB.AC ) AP = AB.AC AI không êi, mà P thuëc tia AB cè ành nên P cè ành. Gåi trung iºm cõa PI là K thì K cè ành và tâm O0 cõa ÷íng tròn ÷íng kính OP (tùc ÷íng tròn ngo¤i ti¸p 4ONI) luôn n¬m trên ÷íng th¯ng d cè ành vuông góc vîi BC ð K. 2.12 áp án tuyºn sinh lîp 10 nam 1995 (cho thí sinh chuyên toán và chuyên tin) Bài 1. x + p x2 + 3 y + = 3 (1) p y2 + 3 Ta có: x + p x2 + 3 − x + = 3 (2) p x2 + 3 y + p y2 + 3 − y + = 3 (3) p y2 + 3 www.vnmath.com
  • 90. 2.12. áp án tuyºn sinh lîp 10 nam 1995(cho thí sinh chuyên toán và chuyên tin)69 Nhân (2) vîi (3) và chia cho (1) ta ÷ñc: − x + p x2 + 3 − y + = 3 (4) p y2 + 3 (1) ) xy + x p y2 + 3+y p x2 + 3+ p x2 + 3. p y2 + 3 = 3 (5) (4) ) xy − x p y2 + 3 − y p x2 + 3+ p x2 + 3. p y2 + 3 = 3 (6) Trø (5) cho (6) ta ÷ñc x p y2 + 3+y p x2 + 3 = 0 Suy ra x, y trái d§u ho°c cùng b¬ng 0 và x p y2 + 3 = −y p x2 + 3 ) x2(y2 + 3) = y2(x2 + 3) ) 3x2 = 3y2 ) |x| = |y| ) x = −y (vì x, y trái d§u ho°c cùng b¬ng 0) nên E = x + y = 0 Bài 2. 8 : x + xy + y = 1 y + yz + z = 3 z + zx + x = 7 ) 8 : (x + 1)(y + 1) = 2 (1) (y + 1)(z + 1) = 4 (2) (z + 1)(x + 1) = 8 (3) 8 : (x + 1)(y+1) = 2 (y + 1)(z + 1) = 4 [(x + 1)(y + 1)(z + 1)]2 = 64 ) 8 : (x + 1)(y+1) = 2 (y + 1)(z + 1) = 4 (x + 1)(y + 1)(z + 1) = ±8 Vîi (x + 1)(y + 1)(z+1) = 8 ta có: x + 1 = 2, y + 1 = 1, z + 1 = 4 ) x = 1, y = 0, z = 3 Vîi (x + 1)(y + 1)(z + 1) = −8 ta có: x + 1 = −2, y + 1 = −1, z + 1 = −4 ) x = −3, y = −2, z = −5 Vªy h» có hai nghi»m x = 1, y = 0, z = 3 x = −3, y = −2, z = −5 Chú ý: Có thº gi£i b¬ng cách nhân (1) vîi (3) rçi chia cho (2) ta ÷ñc (x + 1)2 = 4 ) x + 1 = ±2 tø ây d¹ dàng tìm ÷ñc các nghi»m. www.vnmath.com
  • 91. 70 Ch÷ìng 2. áp án tuyºn sinh Bài 3. Tø gi£ thi¸t x, y 0, x2 + y2 = 1 suy ra vîi 0 6 x, y 6 1 ta có ( x3 6 x2 y3 6 y2 ) x3 + y3 6 x2 + y2 = 1 (D¹ th§y d§u = ¤t ÷ñc khi và ch¿ khi x = 1, y = 0 ho°c x = 0, y = 1) Ta có: 1 = (x2 + y2)3 = x6 + 3x4y2 + 3x2y4 + y6 2(x3 + y3)2 = 2x6 + 4x3y3 + 2y6 Trø hai ¯ng thùc cuèi ta ÷ñc 2(x3 + y3)2 −1 = x6 + y6 + 4x3y3 − 3x4y2 − 3x2y4 = (x3 − y3)2 − 3x2y2(x − y)2 = (x − y)2[(x2 + xy + y2)2 − 3x2y2] = (x − y)2[x4 + y4 + 2x3y + 2xy3] 0 (vì x4 + y4 + 2x3y + 2xy3 0) Vªy 2(x3 + y3)2 1 hay x3 + y3 p1 2 . D§u = ¤t ÷ñc khi và ch¿ khi x = y = p1 2 ). Chú ý: Có thº chùng minh b§t ¯ng thùc cuèi mët cách ng­n gån hìn (nh÷ng v÷ñt ngoài ch÷ìng trình!) b¬ng cách sû döng b§t ¯ng thùc Cauchy- Buniacovski nh÷ sau: Ta có: (x + y)2 2(x2 + y2) 2 ) x + y p 2 1 = (x2 + y2)2 = ( p x3 + p x p y p y3)2 (x + y)(x3 + y3) ) x3 + y3 1 x + y 1 p 2 Bài 4. °t a = a1a2a3 thì b1b2b3 = 2a. Khi ó A = a1a2a3b1b2b3a1a2a3 = 106a + 2.103a + a = (103 + 1)2a = 10012.a = 72.112.132.a Vì A vi¸t ÷ñc d÷îi d¤ng A = p21 p22 p23 p24 , trong ó p1, p2, p3, p4 là bèn sè nguyên tè khác nhau, nên ba trong bèn sè p1, p2, p3 ph£i là 7, 11, 13 còn sè thù t÷ có bình ph÷ìng b¬ng a. Do ó a là bình ph÷ìng cõa mët sè nguyên tè khác 7, 11, 13. Chú ý r¬ng: 100 a = 1 2b 1000 2 500 suy ra a = 172 ho°c a = 192. Vªy có hai sè tho£ mãn i·u ki»n bài toán là A = 289.578.289, và A = 361.722.361 www.vnmath.com